NCLEX Pediatrics

Lakukan tugas rumah & ujian kamu dengan baik sekarang menggunakan Quizwiz!

2. Schedule an immediate appointment with their health care provider. RATIONALE: Because more information needs to be obtained from the child and family, an immediate appointment is most appropriate. It's unclear what type of abuse the parents are concerned about. Calling child protective services is appropriate but isn't the first action to take; neither is talking to an attorney or the day-care provider.

Parents of a 5-year-old call the clinic to tell the nurse that they think their child has been abused by her day-care provider. What should the nurse advise them to do? 1. Make an appointment to speak with the day-care provider. 2. Schedule an immediate appointment with their health care provider. 3. Call the child protective services to file a complaint. 4. Talk to their attorney to file charges against the accused.

1. "Pancreatic enzymes promote absorption of nutrients and fat." RATIONALE: Pancreatic enzymes are given to a child with cystic fibrosis to aid fat and protein digestion. They don't promote rest or prevent mucus accumulation or meconium ileus.

Parents of a child with cystic fibrosis ask the nurse why their child must receive supplemental pancreatic enzymes. Which response by the nurse is most appropriate? 1. "Pancreatic enzymes promote absorption of nutrients and fat." 2. "Pancreatic enzymes promote adequate rest." 3. "Pancreatic enzymes prevent intestinal mucus accumulation." 4. "Pancreatic enzymes help prevent meconium ileus."

3. is highly sensitive to criticism. RATIONALE: A nurse should explain that a 6-year-old child has a precarious sense of self that can cause overreaction to criticism and a sense of inferiority. By age 6, most children no longer depend on the parents for daily tasks and actually love the routine of a schedule. Tattling is more common at age 4 or 5; by age 6, the child wants to make friends and be a friend.

Parents of a child, age 6, who will begin school in the fall ask the nurse for anticipatory guidance. The nurse should explain that a child of this age: 1. still depends on the parents. 2. rebels against scheduled activities. 3. is highly sensitive to criticism. 4. loves to tattle.

3. The quality of food that a preschooler consumes is more important than the quantity. RATIONALE: Stating that food quality is more important than quantity is most accurate because a high caloric intake may include many empty calories. The preschooler's caloric requirement is slightly lower than the toddler's. Overall, however, the preschooler's nutritional requirements are similar to a toddler's. The preschooler requires 1.5 g/kg of protein daily, satisfied by two meat servings, three milk servings, four bread servings, and four fruit and vegetable servings.

Parents of a preschool-age child ask the nurse about nutrition. Which statement about a preschooler's nutritional requirements is accurate? 1. Caloric requirements per kilogram of body weight increase slightly during the preschool-age period. 2. The preschooler's nutritional requirements differ greatly from those of a toddler. 3. The quality of food that a preschooler consumes is more important than the quantity. 4. Protein should account for 25% of the preschooler's total caloric intake.

4. Diarrhea RATIONALE: Diarrhea is the most common cause of dehydration and acidosis in infants. Early introduction of solid foods may cause loose stools but not dehydration or acidosis. Poor perianal hygiene may cause diaper dermatitis. Tachypnea is a sign — not a cause — of acidosis.

A 10-month-old infant is admitted to the facility with dehydration and metabolic acidosis. What is the most common cause of dehydration and acidosis in infants? 1. Early introduction of solid foods 2. Inadequate perianal hygiene 3. Tachypnea 4. Diarrhea

4. "Friends will be very important to her, and she'll develop an interest in the opposite sex." RATIONALE: At age 10, friends become very important. Also, children usually begin having an interest in the opposite sex around this age, although they aren't always willing to admit it. Her physical development towards maturity continues, but it isn't as rapid at this stage as in previous years. Although independence increases at this stage, children continue to need parental supervision. Growth and development slow down but gradual changes continue to occur.

A 10-year-old girl visits the clinic for a checkup before entering school. The child's mother questions the nurse about what to expect of her daughter's growth and development at this stage. Which response is appropriate? 1. "Her physical development will be rapid at this stage, and rapid development will continue from now on." 2. "She'll become more independent and won't require parental supervision." 3. "Don't anticipate any changes at this stage in her growth and development." 4. "Friends will be very important to her, and she'll develop an interest in the opposite sex."

3. developmental delays. RATIONALE: Children with AIDS commonly exhibit developmental delays or regression. To plan developmentally appropriate care and establish realistic goals, the nurse must obtain information about the child's developmental status. Unlike adults with AIDS, children with this disease rarely develop Kaposi's sarcoma. AIDS isn't associated with congenital heart anomalies. Clinical manifestations of Wiskott-Aldrich syndrome, an X-linked recessive disorder characterized by immunodeficiency, resemble those of AIDS; however, the two syndromes aren't related.

A 14-month-old child with acquired immunodeficiency syndrome (AIDS) is admitted to the facility with an infection. When developing a care plan, the nurse must keep in mind that AIDS in children commonly is associated with: 1. Kaposi's sarcoma. 2. congenital heart anomalies. 3. developmental delays. 4. Wiskott-Aldrich syndrome.

RATIONALE: The nurse should place the V1 lead in the fourth intercostal space to the right of the sternum.

A 15-year-old adolescent is admitted to the telemetry unit because of suspected cardiac arrhythmia. A nurse applies five electrodes to his chest and then attaches the lead wires. Identify the area where the nurse should place the chest lead (V1).

4. HIB, DTaP, HepB, IPV, and pneumococcal conjugate vaccine (PCV) RATIONALE: The current immunizations recommended for a 2-month-old who hasn't received any immunizations are HIB, DTaP, HepB, PCV, and IPV. The first immunizations for MMR and varicella are recommended when a child is age 12 months.

A 2-month-old infant hasn't received any immunizations. Which immunizations should the nurse prepare to administer? 1. Measles, mumps, rubella (MMR); diphtheria, tetanus toxoids, and acellular pertussis (DTaP); and hepatitis B (HepB) 2. Polio (IPV), DTaP, MMR 3. Varicella, Haemophilus influenzae type b (HIB), IPV, and DTaP 4. HIB, DTaP, HepB, IPV, and pneumococcal conjugate vaccine (PCV)

2. Palpate his abdomen to monitor tumor growth. RATIONALE: The abdomen of a child with Wilms' tumor should never be palpated because it may increase the risk of metastasis. All children and their parents require preoperative teaching when surgery is planned. Assessing vital signs and monitoring urine are appropriate interventions because a child with Wilms' tumor may be hypertensive as a result of excessive renin production and may have hematuria.

A 4-year-old boy is scheduled for a nephrectomy to remove a Wilms' tumor. Which intervention listed in the care plan should the nurse question? 1. Provide preoperative teaching to the child and his parents. 2. Palpate his abdomen to monitor tumor growth. 3. Assess vital signs and report hypertension. 4. Monitor urine for hematuria.

4. an abdominal mass. RATIONALE: The most common sign of Wilms' tumor is a painless, palpable abdominal mass, sometimes accompanied by an increase in abdominal girth. Gross hematuria is uncommon, although microscopic hematuria may be present. Dysuria isn't associated with Wilms' tumor. Nausea and vomiting are rare in children with Wilms' tumor.

A child is diagnosed with Wilms' tumor. During assessment, the nurse expects to detect: 1. gross hematuria. 2. dysuria. 3. nausea and vomiting. 4. an abdominal mass.

2. Somatrem (Protropin) RATIONALE: Somatrem is used to treat linear growth failure stemming from hormonal deficiency. Corticotropin zinc hydroxide is used to treat adrenal insufficiency and a variety of other conditions; desmopressin acetate and vasopressin are used to treat diabetes insipidus.

A child is diagnosed with pituitary dwarfism. Which pituitary agent will the physician most likely order to treat this condition? 1. Corticotropin zinc hydroxide (Cortrophin-Zinc) 2. Somatrem (Protropin) 3. Desmopressin acetate (DDAVP) 4. Vasopressin (Pitressin)

4. Ways to prevent infection RATIONALE: Because overwhelming infection is the most common cause of death in clients with leukemia, preventing infection is the most important teaching topic. Although promoting adjustment to an altered body image and increasing peer interactions are important, they don't address life-threatening concerns and therefore take lower priority. The nurse should advise the parents to let the child's desire and tolerance for activity determine the child's activity level.

A child is receiving chemotherapy for treatment of acute lymphocytic leukemia. During discharge preparation, which topic is most important for the nurse to discuss with the child and parents? 1. How to help the child adjust to an altered body image 2. How to increase the child's interactions with peers 3. The need to decrease the child's activity level 4. Ways to prevent infection

2. abdominal tenderness. RATIONALE: The nurse should stay alert for abdominal tenderness because it's an early sign of peritonitis. Redness at the catheter site indicates a skin infection. Abdominal fullness is expected during dialysate infusion. Headache isn't associated with peritonitis.

A child is receiving peritoneal dialysis to treat renal failure. To detect early signs of peritonitis, the nurse should stay alert for: 1. redness at the catheter site. 2. abdominal tenderness. 3. abdominal fullness. 4. headache.

3. implement pain control measures. RATIONALE: Because hydrotherapy is painful, the nurse should implement pain control measures before this treatment begins. Fluids and nutritional supplements can be given at any time and aren't required specifically before hydrotherapy. Antibiotics should be administered according to a specified schedule without regard to any treatment.

A child with a full-thickness burn is scheduled for debridement using hydrotherapy. Before hydrotherapy begins, the nurse should: 1. administer fluids as ordered. 2. administer antibiotics as ordered. 3. implement pain control measures. 4. provide nutritional supplements.

2. "I can't wiggle my fingers." RATIONALE: Signs and symptoms of compartment syndrome, such as motor weakness, reflect a deficit or deterioration of neuromuscular status in the involved area. Inability to wiggle fingers indicates an immediate risk for compartment syndrome because it could suggest neurovascular pressure or damage caused by edema following the injury. The other statements don't indicate risk for compartment syndrome.

A child, age 4, fell and broke his arm and had a cast applied. Which of these statements by the child indicates an immediate risk for compartment syndrome? 1. "My arm hurts." 2. "I can't wiggle my fingers." 3. "I need to go home." 4. "Don't touch me."

4. Iron-rich formula alone RATIONALE: The American Academy of Pediatrics recommends iron-rich formula for 5-month-old infants and cautions against giving infants solid food — even baby food — until age 6 months. The Academy doesn't recommend whole milk before age 12 months or skim milk before age 2 years.

A mother is discontinuing breast-feeding after 5 months. What should the nurse advise the mother to include in her infant's diet? 1. Iron-rich formula and baby food 2. Whole milk and baby food 3. Skim milk and baby food 4. Iron-rich formula alone

4. 10-year-old girl RATIONALE: The 10-year-old girl is most at risk because scoliosis is five times more common in girls than boys, and its peak age of incidence is between ages 8 and 15. The 8-year-old boy or a teenage boy may develop scoliosis but it's more common in females. A 6-year-old girl is typically too young to be diagnosed with scoliosis.

A nurse in a well-child clinic is assessing children for scoliosis. Which child is most at risk for scoliosis? 1. 8-year-old boy 2. Teenage boy 3. 6-year-old girl 4. 10-year-old girl

2. fluid overload. RATIONALE: Infants, small children, and children with compromised cardiopulmonary status receiving I.V. therapy are particularly vulnerable to fluid overload. To prevent fluid overload, the nurse should use a volume-control set and an infusion pump or syringe and place no more than 2 hours' worth of I.V. fluid in the volume-control set at a time. Hypotension, cardiac arrhythmias, and pulmonary emboli aren't problems associated with I.V. therapy in infants.

A nurse is administering I.V. fluids to an infant. Infants receiving I.V. therapy are particularly vulnerable to: 1. hypotension. 2. fluid overload. 3. cardiac arrhythmias. 4. pulmonary emboli.

4. amenorrhea. RATIONALE: Amenorrhea is common finding in girls and women with anorexia nervosa. Researchers don't know whether the condition results from starvation or from an underlying metabolic disturbance. Insomnia isn't associated with anorexia nervosa. Clients with anorexia nervosa are capable of eating and rarely have dysphagia (difficulty swallowing). Anorexia nervosa is more likely to cause constipation than diarrhea because limited oral intake decreases GI motility.

A nurse is assessing a 15-year-old girl who has lost 30 lb (13.6 kg) over 3 months. What other finding is the nurse likely to assess? 1. insomnia. 2. dysphagia. 3. diarrhea. 4. amenorrhea.

2. "No, it isn't necessary because you aren't sexually active." RATIONALE: A 16-year-old girl who isn't sexually active doesn't need a Pap test. When a girl is sexually active or reaches age 18, she should have a Pap test.

A nurse is caring for a 16-year-old girl who isn't sexually active. The girl asks if she needs a Papanicolaou (Pap) test. The nurse should reply: 1. "Yes, a girl should have a Pap test after she begins to menstruate." 2. "No, it isn't necessary because you aren't sexually active." 3. "Yes, you should have a Pap test because you're 16 years old." 4. "No, it isn't necessary because you aren't yet 21 years old."

2. Inform the father that the procedure won't be performed because the mother didn't consent. RATIONALE: The parent who has legal custody of a child has medical decision-making rights for that child. The other parent could contest the decision but would need to seek legal counsel. After informing the father that the procedure won't be performed at this time, the nurse should make the physician and social services aware of the situation in case additional problems arise.

A nurse is caring for a 4-year-old child with end-stage leukemia. The child's physician has ordered a lumbar puncture. His mother, who has legal custody, has refused to give consent for the child to undergo the procedure. However, the child's father is demanding that the procedure be performed. What should the nurse do first? 1. Prepare the child for the lumbar puncture because the father wants the procedure to be performed. 2. Inform the father that the procedure won't be performed because the mother didn't consent. 3. Ask the child if he would like to have the procedure. 4. Contact social services and the child's physician.

4. cricoid cartilage is the narrowest part of the larynx. RATIONALE: An uncuffed endotracheal tube is used because the cricoid cartilage in the toddler is the narrowest part of the larynx and provides a natural seal. This aspect keeps the endotracheal tube in place without requiring a cuff. The vocal cords are narrower in an adult. Although the trachea is shorter and the larynx is anterior and cephalad, these aren't reasons to choose an uncuffed tube.

A nurse is caring for a toddler in respiratory distress. She is gathering supplies to help with endotracheal intubation. The nurse knows the physician will use an uncuffed endotracheal tube because the: 1. vocal cords provide a natural seal. 2. trachea is shorter. 3. larynx is anterior and cephalad. 4. cricoid cartilage is the narrowest part of the larynx.

3. hold and rock him and give him a security object. RATIONALE: The child with Down syndrome may have difficulty coping with painful procedures and may regress during his illness. Holding, rocking, and giving the child a security object is helpful because it may be comforting to the child. An older child or a child without Down syndrome may benefit from positive self-talk, time limits, and diversionary tactics, such as counting and singing; however, the success of these tactics depends on the child.

A nurse is caring for a toddler with Down syndrome. To help the toddler cope with painful procedures, the nurse can: 1. prepare the child by positive self-talk. 2. establish a time limit to get ready for the procedure. 3. hold and rock him and give him a security object. 4. count and sing with the child.

2. Place locks on cabinets containing toxic substances. RATIONALE: The nurse should tell parents to place locks on cabinets containing toxic substances because a toddler's curiosity and the ability to climb and open doors and drawers make poisoning a concern in this age-group. Roller blading isn't an appropriate activity for toddlers even if the toddler wears a helmet. Toddlers lack the cognitive development to understand water safety. Pillows shouldn't be placed in the crib of an infant to avoid suffocation; however, toddlers may use them.

A nurse is teaching accident prevention to the parents of a toddler. Which instruction is appropriate for the nurse to tell the parents? 1. The toddler should wear a helmet when roller blading. 2. Place locks on cabinets containing toxic substances. 3. Teach the toddler water safety. 4. Don't allow the toddler to use pillows when sleeping.

2. leukemia. RATIONALE: The nurse should tell the mother that live virus vaccines shouldn't be administered to children with leukemia because they cause immunosuppression. Inactivated — rather than live — viruses should be administered. Children with diabetes mellitus, asthma, or cystic fibrosis can receive live virus vaccines because they aren't immunosuppressed.

A nurse is teaching the mother of an ill child about childhood immunizations. The nurse should tell the mother that live virus vaccines are contraindicated in children with: 1. diabetes mellitus. 2. leukemia. 3. asthma. 4. cystic fibrosis.

3. The nurse should follow facility procedures for reporting an error. RATIONALE: Although no harm came to the child, this scenario is an example of a medication error. The nurse should follow the facility's procedure for reporting the error because it allows the facility to adequately assess the causes of medication errors, and isn't meant to place blame on any one person. The nurse in this instance doesn't need to notify the physician because there was no harm to the child. Also, the nurse shouldn't document that an error took place in the child's chart; doing so may place her at risk in the event of a lawsuit.

A nurse realizes she is 1 hour late in administering a dose of medication for a 4-year-old child. She gives the medication immediately, and assesses the child. The child isn't harmed by the delay. Which action should the nurse take next? 1. No further action is necessary. 2. The nurse should notify the physician of the error. 3. The nurse should follow facility procedures for reporting an error. 4. The nurse should document a medication error in the client's chart.

3. a nomogram. RATIONALE: The method for determining body surface area is a three-column chart called a nomogram. The nurse marks the child's height in the first column and weight in the third column, then draws a line between the two marks. The point at which the line intersects the vertical scale in the second column indicates the estimated body surface area of the child in square meters. Using height or weight alone isn't sufficient, and the difference between weight and height isn't a measurement of body surface area.

A nurse should determine a child's body surface area by using: 1. weight. 2. height. 3. a nomogram. 4. the difference between weight and height.

2. being fed formula that isn't mixed according to the manufacturer's instructions. RATIONALE: Incorrectly mixed formula can cause an infant to develop severe electrolyte and nutrition imbalances. This safety hazard necessitates immediate attention. Placing a 3-month-old infant in a rear-facing car seat is appropriate. Although an infant sleeping in a cardboard box on the floor may be a concern, it isn't an immediate safety hazard. An infant being put to sleep with a pacifier isn't a safety concern.

A nurse should take action when a healthy 3-month-old infant is: 1. placed in a convertible car seat in a rear-facing position. 2. being fed formula that isn't mixed according to the manufacturer's instructions. 3. sleeping in a cardboard box on the floor of his mother's bedroom. 4. being put to sleep with a pacifier.

3. "What did your child eat for breakfast?" RATIONALE: The nurse should ask what the child ate for breakfast in order to obtain objective information about the child's nutritional intake. Asking if the child has always been so thin, if he's a picky eater, or if he eats enough would elicit subjective replies that would be open to interpretation.

A nurse suspects that a child, age 4, is being neglected physically. To best assess the child's nutritional status, the nurse should ask the parents which question? 1. "Has your child always been so thin?" 2. "Is your child a picky eater?" 3. "What did your child eat for breakfast?" 4. "Do you think your child eats enough?"

4. The child eats finger foods by himself. RATIONALE: The child eating finger foods by himself indicates effective teaching because a child with cerebral palsy should be encouraged to be as independent as possible. Finger foods allow the toddler to feed himself. Because spasticity affects coordinated chewing and swallowing as well as the ability to bring food to the mouth, it's difficult for the child with cerebral palsy to eat neatly. In terms of a specified period of time, the child with cerebral palsy may require more time to bring food to the mouth; thus, chewing and swallowing shouldn't be rushed. A child shouldn't lie down to rest after eating because doing so may cause the child to vomit from a hyperactive gag reflex. Therefore, the child should remain in an upright position after eating to prevent aspiration and choking.

A nurse teaches a mother how to provide adequate nutrition for her toddler, who has cerebral palsy. Which observation indicates that teaching has been effective? 1. The toddler stays neat while eating. 2. The toddler finishes the meal within a specified period of time. 3. The child lies down to rest after eating. 4. The child eats finger foods by himself.

3. Closed anterior and posterior fontanels RATIONALE: By age 18 months, the anterior and posterior fontanels should be closed. The diamond-shaped anterior fontanel normally closes between ages 9 and 18 months. The triangular posterior fontanel normally closes between ages 2 and 3 months.

A parent brings a toddler, age 19 months, to the clinic for a regular checkup. When palpating the toddler's fontanels, what should the nurse expect to find? 1. Closed anterior fontanel and open posterior fontanel 2. Open anterior fontanel and closed posterior fontanel 3. Closed anterior and posterior fontanels 4. Open anterior and posterior fontanels

3. 23G RATIONALE: For an infant, the nurse should use a needle with the smallest appropriate gauge for the medication to be given. For an I.M. injection of meperidine, a 25G to 22G needle is appropriate. An 18G or 20G needle is too large, and the 27G needle too small.

A physician orders meperidine (Demerol), 1.1 mg/kg I.M., for a 16-month-old child who has just had abdominal surgery. When administering this drug, the nurse should use a needle of which size? 1. 18G 2. 20G 3. 23G 4. 27G

3. 0.6 ml RATIONALE: By using the fraction method and cross-multiplying to solve for X, the nurse can determine that 0.6 ml should be administered: X ml/30 mg = 1 ml/50 mg X ml × 50 mg = 30 mg × 1 ml X = 0.6 ml.

A physician orders meperidine (Demerol), 30 mg I.M., as preoperative medication for a school-age child who weighs 66 lb (30 kg). The meperidine is supplied as 50 mg/ml. How much meperidine should the nurse administer? 1. 0.3 ml 2. 0.5 ml 3. 0.6 ml 4. 0.8 ml

2. Vastus lateralis muscle RATIONALE: For a child younger than age 3, the thigh (vastus lateralis muscle) is the best site for I.M. injections because it has few major nerves and blood vessels. The deltoid, dorsogluteal, and ventrogluteal sites aren't recommended for a child younger than age 3 because of the lack of muscle development and the risk of nerve injury during injection. Before the dorsogluteal or ventrogluteal sites can be used safely, the child should have been walking for at least 1 year to ensure sufficient muscle development.

A physician orders penicillin G, 300,000 units I.M., for an 18-month-old child. Where should the nurse administer this injection? 1. Deltoid muscle 2. Vastus lateralis muscle 3. Dorsogluteal muscle 4. Ventrogluteal muscle

3. Hypokalemia RATIONALE: The nurse should monitor the client receiving an I.V. infusion of terbutaline for hypokalemia, lactic acidosis, chest pain, arrhythmias, dyspnea, bloating, chills, or anaphylactic shock. Terbutaline doesn't cause calcium imbalances.

A physician orders terbutaline 2.5 mg by mouth four times a day, for a child with bronchitis. If the child receives an I.V. infusion of terbutaline, which serious adverse reaction is possible? 1. Hypocalcemia 2. Hypercalcemia 3. Hypokalemia 4. Hyperkalemia

1. Magnesium sulfate RATIONALE: Magnesium sulfate is an electrolyte that's used as an adjunct to treat acute nephritis. It also is used to treat seizures and severe toxemia. Calcium glubionate, potassium chloride, and sodium lactate aren't therapeutic in acute nephritis and, in fact, may worsen the condition.

A preschool child is admitted to the pediatric unit with acute nephritis. Which electrolyte replacement agent is used as an adjunct to treatment for this condition? 1. Magnesium sulfate 2. Calcium glubionate 3. Potassium chloride 4. Sodium lactate

3. Remove objects from the child's surroundings. RATIONALE: During a seizure, the nurse's first priority is to protect the child from injury caused by uncontrolled movements. Therefore, the nurse must first remove objects from the child's surroundings and pad objects that can't be removed. Restraining the child or placing an object in the child's mouth during a seizure isn't appropriate because it may cause injury. When the seizure stops, the nurse should then check for breathing and, if indicated, initiate rescue breathing.

A toddler is having a tonic-clonic seizure. What should the nurse do first? 1. Restrain the child. 2. Place a tongue blade in the child's mouth. 3. Remove objects from the child's surroundings. 4. Check the child's breathing.

4. When to administer prophylactic antibiotics RATIONALE: In CHD, areas of turbulent blood flow provide an optimal environment for bacterial growth. Therefore, a child with CHD is at increased risk for bacterial endocarditis, an infection of the heart valves and lining, and requires prophylactic antibiotics before dental work and invasive procedures. These children should receive all childhood immunizations. They don't require postural drainage or dietary fat restriction.

A toddler is hospitalized for evaluation and management of congenital heart disease (CHD). During discharge preparation, the nurse should discuss which topic with the parents? 1. The need to withhold childhood immunizations 2. The importance of restricting the child's fat intake 3. How to perform postural drainage 4. When to administer prophylactic antibiotics

1. Bradycardia RATIONALE: Digoxin enhances cardiac efficiency by increasing the force of contraction and decreasing the heart rate. An early sign of digoxin toxicity is bradycardia (an abnormally slow heart rate). To help detect digoxin toxicity, the nurse always should measure the apical heart rate before administering each digoxin dose. Other signs and symptoms of digoxin toxicity include arrhythmias, vomiting, hypotension, fatigue, drowsiness, and visual halos around objects. Tachycardia, hypertension, and hyperactivity aren't associated with digoxin toxicity.

A toddler with a ventricular septal defect is receiving digoxin (Lanoxin) to treat heart failure. Which assessment finding should be the nurse's priority concern? 1. Bradycardia 2. Tachycardia 3. Hypertension 4. Hyperactivity

3. Industry versus inferiority RATIONALE: According to Erikson, an 11-year-old child is working through the stage of industry versus inferiority. Chronic illness may interfere with this stage of development in an 11-year-old child because the child may not be able to accomplish tasks, which prevents him from achieving a sense of industry. Intimacy is the developmental task of a young adult. Trust is the developmental task to be achieved during infancy. Identity is the developmental task of adolescence.

According to Erikson's theory of development, chronic illness can interfere with which stage of development in an 11-year-old child? 1. Intimacy versus isolation 2. Trust versus mistrust 3. Industry versus inferiority 4. Identity versus role confusion

4. encouraging peer visitation. RATIONALE: Peer visitation gives the adolescent an opportunity to continue along his path toward independence and identity. Knowledge of the facility regimen prepares the adolescent for upcoming procedures but doesn't affect his development. To achieve a sense of identity, the adolescent must gain independence from his family. Tutoring may help him maintain a positive self-image relative to his schoolwork but doesn't affect his development.

According to Erikson, the psychosocial task of adolescence is the development of a sense of identity. A nurse can best promote the development of a hospitalized adolescent by: 1. emphasizing the need to follow the facility regimen. 2. allowing parents and siblings to visit frequently. 3. arranging for tutoring in school work. 4. encouraging peer visitation.

2. Holding the infant semi-upright during feedings RATIONALE: Holding the infant semi-upright during feedings is appropriate because it helps prevent aspiration. The Logan bow must be kept in place at all times to protect the suture line. The infant should be burped more frequently to prevent regurgitation and aspiration. Placing the infant on the abdomen could lead to disruption of the suture line if the infant rubs the face.

After surgery to repair a cleft lip, an infant has a Logan bow in place. Which postoperative nursing action is appropriate? 1. Removing the Logan bow during feedings 2. Holding the infant semi-upright during feedings 3. Burping the infant less frequently 4. Placing the infant on the abdomen after feedings

3. cerebral hyperemia. RATIONALE: Cerebral hyperemia (excess blood in the brain) causes an initial increase in intracranial pressure in the head of an injured child. The brain is less myelinated in a child and more easily injured than an adult brain. Intracranial hypertension — not hypotension — places the child at greater risk for secondary brain injury. A child's cranium is thinner and more pliable than an adult's, causing the child to receive a more severe injury.

An 8-month-old infant is admitted to the pediatric unit following a fall from his high chair. The child is awake, alert, and crying. The nurse should know that a brain injury is more severe in children because of: 1. increased myelination. 2. intracranial hypotension. 3. cerebral hyperemia. 4. a slightly thicker cranium.

187.5 milligrams RATIONALE: The nurse should calculate the correct dose using the following equation: 25 mg/kg × 7.5 kg = 187.5 mg

An infant who weighs 7.5 kg is to receive ampicillin (Omnipen) 25 mg/kg I.V. every 6 hours. How many milligrams should the nurse administer per dose? Record your answer using one decimal place. Answer: milligrams

3. Ineffective sexuality patterns related to parent's expectations RATIONALE: This girl is expressing concerns about the conflict between her parent's expectations and her own desires. Sexual activity is a normal experimental pattern for many adolescents, but she verbalizes parental expectations against this behavior. No evidence suggests she's having a conflict with her boyfriend, delayed growth, or problems with social interactions.

At the health clinic, a sexually active 15-year-old girl tells a nurse she's worried that her parents may find out about her sexual activity. "They would never approve," she says. The nurse should formulate which nursing diagnosis? 1. Delayed growth and development related to sexual activity 2. Impaired social interaction related to boyfriend's expectations 3. Ineffective sexuality patterns related to parent's expectations 4. Fear related to boyfriend's expectations

4. "Let your child eat any food he wants." RATIONALE: The nurse should instruct the parents to let the child eat any food he wants because any form of intake is better than none. Dry crackers or toast would be appropriate for a child experiencing nausea. Withholding all foods and fluids or ignoring lack of food intake would be inappropriate.

During chemotherapy, a boy, age 10, loses his appetite. When teaching the parents about his food intake, the nurse should include which instruction? 1. "Offer dry toast and crackers." 2. "Withhold all food and fluids." 3. "Ignore your child's lack of food intake." 4. "Let your child eat any food he wants."

3. Avoiding abdominal palpation RATIONALE: Because manipulating the abdominal mass may disseminate cancer cells to adjacent and distant sites, the most important intervention for a child with a Wilms' tumor is to avoid palpating the abdomen. Restricting oral intake and monitoring acid-base balance are routine interventions for all preoperative clients; they have no higher priority in one with a Wilms' tumor. Isolation isn't required because a Wilms' tumor isn't infectious.

For a child with a Wilms' tumor, which preoperative nursing intervention takes highest priority? 1. Restricting oral intake 2. Monitoring acid-base balance 3. Avoiding abdominal palpation 4. Maintaining strict isolation

3. Breathing pattern RATIONALE: Breathing pattern is the most important factor to assess because eschar impedes chest expansion in a child with a circumferential chest burn, causing breathing difficulty. Wound characteristics, body temperature, and heart rate are also factors that should be assessed, but they aren't as important as breathing pattern.

For a child with a circumferential chest burn, what is the most important factor for the nurse to assess? 1. Wound characteristics 2. Body temperature 3. Breathing pattern 4. Heart rate

2. Preventing bleeding episodes RATIONALE: A child with hemophilia is prone to bleeding episodes stemming from coagulatory problems. Therefore, the primary nursing goal is to prevent bleeding episodes and possible hemorrhage. A secondary effect of preventing bleeding episodes is maintenance of tissue perfusion and oxygenation. Hemophilia rarely causes pain.

For a child with hemophilia, what is the most important nursing goal? 1. Enhancing tissue perfusion 2. Preventing bleeding episodes 3. Promoting tissue oxygenation 4. Controlling pain

2. Preschool age RATIONALE: Preschool-age children are most likely to view illness as a punishment for misdeeds. Separation anxiety, although seen in all age-groups, is most common in older infants. Fear of death is typical of older school-age children and adolescents. Adolescents also fear mutilation.

When developing a care plan for a hospitalized child, the nurse knows that children in which age-group are most likely to view illness as a punishment for misdeeds? 1. Infancy 2. Preschool age 3. School age 4. Adolescence

3. Safety guidelines RATIONALE: Reinforcing safety guidelines is appropriate because such anticipatory guidance helps prevent many accidental injuries. For parents of a 9-month-old infant, it's too early to discuss nursery schools or toilet training. Because surgery isn't used to treat gastroenteritis, this topic is inappropriate.

While preparing to discharge a 9-month-old infant who's recovering from gastroenteritis and dehydration, the nurse teaches the parents about their infant's dietary and fluid requirements. The nurse should include which other topic in the teaching session? 1. Nursery schools 2. Toilet training 3. Safety guidelines 4. Preparation for surgery

3. 6 months RATIONALE: Solid foods are typically introduced around age 6 months. They aren't recommended at an earlier age because of the protrusion and sucking reflexes and the immaturity of the infant's GI tract and immune system. By age 8 months, the infant usually has been introduced to iron-fortified infant cereal and vegetables and will begin to try fruits.

A nurse is teaching the mother of an infant. The nurse should instruct the mother to introduce her infant to solid foods at what age? 1. 2 months 2. 4 months 3. 6 months 4. 8 months

1. cerebrospinal fluid otorrhea. RATIONALE: Basilar skull fracture is a fracture in any bone of the base of the skull — frontal, ethmoid, sphenoid, temporal, or occipital. Therefore, cerebrospinal fluid otorrhea would be observed. Deafness doesn't commonly occur as a result of skull fracture. Battle sign and raccoon eyes occur primarily in orbital, not basilar, fractures.

A 12-month-old child fell down the stairs. A basilar skull fracture is suspected. The nurse should look for: 1. cerebrospinal fluid otorrhea. 2. deafness. 3. raccoon eyes. 4. Battle sign.

1. Measuring the infant's weight RATIONALE: Frequent weight measurement provides the most important information about fluid balance and the infant's response to fluid replacement. Although stool or urine analysis may provide some information, the results typically aren't available for at least 24 hours, making the tests less useful than measuring weight. The posterior fontanel usually closes from ages 6 to 8 weeks and therefore doesn't reflect fluid balance in a 9-month-old infant.

A 9-month-old infant is admitted with diarrhea and dehydration. The nurse plans to assess the child's vital signs frequently. Which other action provides important assessment information? 1. Measuring the infant's weight 2. Obtaining a stool specimen for analysis 3. Obtaining a urine specimen for analysis 4. Inspecting the infant's posterior fontanel

2. "Tell him it isn't acceptable and he'll be disciplined if he continues to do it."

A boy, age 4, begins to use curse words. Concerned about this behavior, his parents ask the nurse how to discourage it. Which advice should the nurse offer? 1. "Just ignore it. He'll grow out of it." 2. "Tell him it isn't acceptable and he'll be disciplined if he continues to do it." 3. "Tell him that good little boys don't use curse words." 4. "Tell him that his behavior makes you angry."

3. Elevating the foot of the bed RATIONALE: To relieve edema of the toes, the most appropriate reaction is to raise the affected extremity above heart level such as by elevating the foot of the bed. Applying ice, massaging the toes, and placing the child on his right side wouldn't reduce swelling.

A child with a fractured left femur receives a cast. A short time later, the nurse notices that the toes on the child's left foot are edematous. Which nursing action would be most appropriate? 1. Applying ice to the foot 2. Massaging the toes 3. Elevating the foot of the bed 4. Placing the child on his right side

RATIONALE: In junctional tachycardia, the atrioventricular node fires rapidly.

A critically ill 4-year-old child is in the pediatric intensive care unit. Telemetry monitoring reveals junctional tachycardia. Identify where this arrhythmia originates.

4. "You seem upset. Having your child hospitalized must be difficult." RATIONALE: Acknowledging the mother's feelings and recognizing that it's difficult to cope with a hospitalized child allows the mother to express her feelings. Telling the mother that other staff members don't want to talk to her isn't therapeutic. Asking her to explain her behavior places the mother on the defensive and also isn't therapeutic.

A mother of a hospitalized infant appears anxious and displays anger with the staff. Which response is most appropriate? 1. "Some of the staff members don't want to talk to you because you might yell at them." 2. "Why do you seem so angry today? It makes it hard for us to help you." 3. "Is this your normal behavior or are you acting out because your child is hospitalized?" 4. "You seem upset. Having your child hospitalized must be difficult."

4. With the heel of one hand RATIONALE: When performing CPR on a child between ages 1 and 8, the nurse should use the heel of one hand to compress the chest one-third to one-half the depth of the chest. Using only the fingers of one hand isn't appropriate for CPR. The use of two fingertips is appropriate for infant CPR but this method can't compress the chest sufficiently on an older child. The palm is never used for chest compressions in CPR.

A preschooler goes into cardiac arrest. When performing cardiopulmonary resuscitation (CPR) on a child, how should the nurse deliver chest compressions? 1. With the fingers of one hand 2. With two fingertips 3. With the palm of one hand 4. With the heel of one hand

3. A protuberant abdomen RATIONALE: The nurse would expect to find a protuberant abdomen caused by the presence of fat, bulky stools; undigested food; and flatus, which are associated with celiac disease. A concave abdomen, bulges in the groin area, and a palpable abdominal mass aren't associated with celiac disease.

An 18-month-old boy is admitted to the pediatric unit with a diagnosis of celiac disease. What finding would the nurse expect in this child? 1. A concave abdomen 2. Bulges in the groin area 3. A protuberant abdomen 4. A palpable abdominal mass

4. hypokalemia. RATIONALE: Enuresis, polydipsia, and weight loss suggest diabetes insipidus, a disorder that may result from a head injury that damages the neurohypophyseal structures. Diabetes insipidus places the child at risk for fluid volume depletion and hypokalemia. Diabetes insipidus doesn't cause hypercalcemia, hyperglycemia, or hyponatremia.

After a head injury, a child experiences enuresis, polydipsia, and weight loss. Based on these findings, the nurse should monitor closely for signs and symptoms of: 1. hypercalcemia. 2. hyperglycemia. 3. hyponatremia. 4. hypokalemia.

1. Anxiety related to separation from parents and an unfamiliar environment RATIONALE: The nature of the accident, the child's pain, and the unfamiliar facility environment support a nursing diagnosis of Anxiety related to separation from parents and an unfamiliar environment. A diagnosis of Hypothermia related to head injury isn't appropriate because the child is alert and oriented, indicating that a head injury, if present, isn't severe and is unlikely to cause hypothermia. Unlike the homecoming of a new baby or riding a bicycle for the first time, a car accident isn't a maturational crisis. Risk for infection related to sepsis isn't a plausible nursing diagnosis at this time.

After a car accident, a child, age 10, is treated in the emergency department for a fractured clavicle and evaluated for a possible head injury. Alert and oriented, she keeps asking what will happen to her. Which nursing diagnosis is most appropriate? 1. Anxiety related to separation from parents and an unfamiliar environment 2. Hypothermia related to head injury 3. Interrupted family processes related to maturational crisis 4. Risk for infection related to sepsis

RATIONALE: After successful resuscitation, dopamine would be given as an infusion to increase cardiac output and maintain blood pressure. Epinephrine, sodium bicarbonate, and atropine are first-round drugs that are used during a cardiopulmonary arrest.

After a child has a cardiopulmonary arrest, which drug would the nurse expect to administer? 1. Dopamine (Inocor) 2. Epinephrine 3. Sodium bicarbonate 4. Atropine

3. "Our child must maintain these dietary restrictions for life." RATIONALE: Teaching is effective if the parents say their child must maintain the dietary restrictions for life because the child needs to avoid recurrence of the disease's clinical manifestations. Signs and symptoms will reappear if the client eats prohibited foods later in life.

After a nurse explains dietary restrictions to the parents of a child with celiac disease, which statement by the parents indicates effective teaching? 1. "We'll follow these instructions until our child's symptoms disappear." 2. "Our child must maintain these dietary restrictions until adulthood." 3. "Our child must maintain these dietary restrictions for life." 4. "We'll follow these instructions until our child has completely grown and developed."

3. Sitting in an infant seat RATIONALE: Because the infant's assessment findings suggest that respiratory distress is developing, the nurse should position the infant with the head elevated at a 45-degree angle to promote maximum chest expansion; an infant seat maintains this position. Placing an infant flat on the back or abdomen or in high Fowler's position could increase respiratory distress by preventing maximal chest expansion.

An infant, age 8 months, has a tentative diagnosis of congenital heart disease. During physical assessment, the nurse measures a heart rate of 170 beats/minute and a respiratory rate of 70 breaths/minute. How should the nurse position the infant? 1. Lying on the back 2. Lying on the abdomen 3. Sitting in an infant seat 4. Sitting in high Fowler's position

1. cerebral edema. RATIONALE: Quick fluid replacement or fluid overload would make the adolescent vulnerable to developing cerebral edema and increased intracranial pressure. Quick fluid replacement or fluid overload won't cause dehydration. It would be unusual for an adolescent to develop heart failure unless overhydration was extreme. Hypovolemic shock would occur with an extreme loss of fluid or blood, not a fluid overload.

An otherwise-healthy adolescent is hospitalized for diabetic ketoacidosis and is receiving I.V. and oral fluids. The nurse should monitor his fluid intake because quick fluid replacement or fluid overload may cause: 1. cerebral edema. 2. dehydration. 3. heart failure. 4. hypovolemic shock.

1. Fatty stools RATIONALE: Pancreatic enzymes normally aid in food digestion in the intestine. In a child with cystic fibrosis, however, these natural enzymes cannot reach the intestine because mucus blocks the pancreatic duct. Without these enzymes, undigested fats and proteins produce fatty stools. If the parents were administering the pancreatic enzymes correctly, the child would have stools of normal consistency. Noncompliance doesn't cause liquid or bloody stools.

At a previous visit, the parents of an infant with cystic fibrosis received instruction in the administration of pancrelipase (Pancrease). At a follow-up visit, which finding in the infant suggests that the parents require more teaching about administering the pancreatic enzymes? 1. Fatty stools 2. Liquid stools 3. Bloody stools 4. Normal stools

2. 12 years RATIONALE: Testicular cancer occurs most frequently between the ages of 15 and 34; therefore, boys should begin doing testicular self-examinations at age 12, which will help them become familiar with the normal contours and consistency of their genital structures.

At what age should a boy be taught how to do a monthly testicular self-examination? 1. 8 years 2. 12 years 3. 16 years 4. When he becomes sexually active

2. Auscultate the heart and lungs. RATIONALE: The nurse should first ausculate the heart and lungs because this assessment rarely distresses an infant. Placing a tape measure on the infant's head, shining a light in the eyes, or undressing the infant before weighing him may cause distress, making the rest of the examination more difficult.

Before a routine checkup, an 8-month-old infant sits contentedly on the mother's lap, chewing on a toy. When preparing to examine this infant, what should the nurse plan to do first? 1. Measure the head circumference. 2. Auscultate the heart and lungs. 3. Elicit the pupillary reaction. 4. Weigh the child.

3. Greater portions of a child's blood volume flows to the head. RATIONALE: If hemorrhage is associated with a head injury and it goes undetected, a child may experience hypovolemic shock because a large portion of a child's blood volume goes to the head. In children, cerebral tissues are softer, thinner, and more flexible — conditions that actually permit diffusion of the impact. Because a child's skull can expand more than an adult's can, a greater amount of posttraumatic edema can occur without evidence of neurologic deficits. Subdural, epidural, and intracerebral hematomas are the different types of head injury that can occur in children and adults.

Craniocerebral injury in a child differs substantially from craniocerebral trauma in an adult. Which statement identifies a difference between children and adults that could produce a life-threatening complication for a child? 1. Cerebral tissues in children are softer, thinner, and more flexible. 2. A child's skull can expand more than an adult's can. 3. Greater portions of a child's blood volume flows to the head. 4. Hematomas in children can include subdural, epidural, and intracerebral.

1. Preschool age (3 to 5 years) RATIONALE: Children in the preschool age-group have a rich fantasy life. Combined with their strong concept of self, fantasy play and participation in care can minimize the trauma of being hospitalized. Adolescents should be allowed choices and control. School-age children are modest and need to have their privacy respected. Procedures should be explained to them. Toddlers should be examined in the presence of their parents because they fear separation. Allow choices when possible.

Encouraging fantasy play and participation by children in their own care is a useful developmental approach for which pediatric age-group? 1. Preschool age (3 to 5 years) 2. Adolescence (10 to 19 years) 3. School age (5 to 10 years) 4. Toddler (1 to 3 years)

3. is motivated to enter an alcohol rehabilitation program. RATIONALE: Willingness to enter a rehabilitation program indicates that the adolescent is motivated to change. An intervention is an emotionally charged meeting; crying may be an indication of manipulation, rather than a sign that the intervention has succeeded. Relapses are common among alcoholics who simply stop drinking; success in overcoming alcoholism is more likely when a structured program is part of the rehabilitation process. Talking with friends doesn't indicate a successful intervention.

Family members and friends stage an intervention for an alcoholic adolescent. The intervention is successful when the adolescent: 1. breaks down and cries. 2. says, "I'm sorry. I'll never drink again." 3. is motivated to enter an alcohol rehabilitation program. 4. is willing to talk with his friends.

3. By shaking it so that all the drug particles are dispersed uniformly RATIONALE: The nurse should shake a suspension before administration to disperse drug particles uniformly. Diluting the suspension and crushing particles aren't recommended for this drug form.

How should a nurse prepare a suspension before administration? 1. By diluting it with normal saline solution 2. By diluting it with 5% dextrose solution 3. By shaking it so that all the drug particles are dispersed uniformly 4. By crushing remaining particles with a mortar and pestle

2. cervical cancer. RATIONALE: Anogenital warts associated with HPV increase an adolescent female's risk of cervical cancer. This risk mandates treatment of all external lesions. HPV doesn't increase the risk of gonorrhea, chlamydia, or UTIs.

Human papillomavirus (HPV) causes anogenital warts. Without proper treatment, anogenital warts increase an adolescent female's risk of: 1. gonorrhea. 2. cervical cancer. 3. chlamydial infections. 4. urinary tract infections (UTIs).

1. use a padded board to secure the extremity. RATIONALE: The nurse should use a padded board because it's adequate to secure the extremity. Restraining all four extremities can be harmful and uncomfortable for the child. Restraining the extremity to the bed's side rail limits the child's movement; the child may bang against the rail and cause injury. Allowing the extremity to be loose increases the risk that the I.V. will infiltrate or be dislodged by the infant

If an infant's I.V. access site is in an extremity, the nurse should: 1. use a padded board to secure the extremity. 2. restrain all four extremities. 3. restrain the extremity to the bed's side rail. 4. allow the extremity to be loose.

2. A 2-year-old child with a ruptured spleen and hypovolemia RATIONALE: In an emergency, intraosseous drug administration is typically used when a child is critically ill and younger than age 3. The 2-year-old child with a ruptured spleen and hypovolemia meets these criteria.

Intraosseous infusion of a medication would be most appropriate for which child? 1. An 18-month-old child with cystic fibrosis 2. A 2-year-old child with a ruptured spleen and hypovolemia 3. A 4-year-old child with celiac disease 4. A 5-year-old child with status asthmaticus

4. Compensation for hypoxia RATIONALE: A congenital heart defect with decreased pulmonary blood flow alters blood flow through the heart and lungs, resulting in hypoxia. To compensate, the body increases the oxygen-carrying capacity of RBCs by increasing RBC production, which causes the Hb level and Hct to rise. In anemia, the Hb level and Hct typically decrease. Altered electrolyte levels and other laboratory values are better indicators of dehydration. An elevated Hb level and HCT aren't associated with jaundice.

Laboratory results for a child with a congenital heart defect with decreased pulmonary blood flow reveal an elevated hemoglobin (Hb) level, hematocrit (HCT), and red blood cell (RBC) count. These data suggest which condition? 1. Anemia 2. Dehydration 3. Jaundice 4. Compensation for hypoxia

2. The child rates pain at 4 out of 5. Administered pain medication as ordered. RATIONALE: Pain is what the child says it is, and the nurse must document what the child reports. If a child's behavior appears to differ from the child's rating of pain, believe the pain rating. A child who uses passive coping behaviors (such as distraction and cooperative) may rate pain as more intense than children who use active coping behaviors (such as crying and kicking). Nurses frequently make judgments about pain based on behavior, which can result in children being inadequately medicated for pain.

One day after an appendectomy, a 9-year-old child rates his pain at 4 out of 5 on the pain scale but is playing video games and laughing with his friend. What should the nurse document on the child's chart? 1. The child is in no apparent distress, and no pain medication is needed at this time. 2. The child rates pain at 4 out of 5. Administered pain medication as ordered. 3. The child doesn't understand the pain scale. Performed teaching to help child match his pain rating to how he appears to be feeling. 4. The child rates his pain at 4 out of 5; however, he appears to be in no distress. Reassess when he's visibly showing signs of pain.

1. A sunken fontanel RATIONALE: In an infant, signs of fluid volume deficit (dehydration) include sunken fontanels, increased pulse rate, and decreased blood pressure. They occur when the body can no longer maintain sufficient intravascular fluid volume. When this happens, the kidneys conserve water to minimize fluid loss, which results in concentrated urine with a high specific gravity.

Parents bring their infant to the clinic, seeking treatment for vomiting and diarrhea that has lasted for 2 days. On assessment, the nurse detects dry mucous membranes and lethargy. What other finding suggests a fluid volume deficit? 1. A sunken fontanel 2. Decreased pulse rate 3. Increased blood pressure 4. Low urine specific gravity

2. 1 week to 1 year, peaking at 2 to 4 months RATIONALE: SIDS can occur anytime between ages 1 week and 1 year. The incidence peaks at ages 2 to 4 months.

Sudden infant death syndrome (SIDS) is one of the most common causes of death in infants. At what age is SIDS most likely to occur? 1. 1 to 2 years 2. 1 week to 1 year, peaking at 2 to 4 months 3. 6 months to 1 year, peaking at 10 months 4. 6 to 8 weeks

2. observe for behavioral changes. RATIONALE: Behavioral changes are common signs of pain and are especially valuable indicators in an 18-month-old child, who has limited verbal skills. Evaluating pupillary response isn't an appropriate technique for assessing pain. Requesting a parental report of a child's pain isn't a reliable assessment technique.

The best way for a nurse to assess pain in an 18-month-old child is to: 1. check the child's pupils. 2. observe for behavioral changes. 3. ask the child, "Are you feeling any pain?" 4. tell the parents to call if the child has pain.

3. have thinner skin than adults. RATIONALE: Children are more susceptible to the effects of chemical and biological attacks because they have thinner skin than adults, increasing their risk of absorbing a chemical. They also have a larger, not smaller, body surface area in relation to their weight than do adults, which increases the chance of chemical absorption. Children breathe at a faster, not slower, rate than adults, allowing them to inhale greater amounts of a toxic agent. Additionally, some chemical agents are heavier than air and accumulate close to the ground, which is closer to a child's breathing zone than an adult's. Because they have less fluid reserve than adults, children are at greater risk of developing rapid dehydration from agents that cause vomiting or diarrhea.

The development of disaster plans should take into consideration that children are more susceptible to the effects of a chemical attack than adults because children: 1. have smaller body surface areas than adults. 2. breathe at a slower rate than adults. 3. have thinner skin than adults. 4. have a low risk of developing rapid dehydration.

1. ventrogluteal muscle. RATIONALE: When administering an I.M. injection to a 2-year-old child, the nurse might select the ventrogluteal muscle if the muscle is well developed. However, the preferred site is the vastus lateralis. The pectoral, femoral, and deltoid muscles aren't appropriate injection sites for a child.

The most appropriate site for a nurse to use to administer an I.M. injection to a 2-year-old child is the: 1. ventrogluteal muscle. 2. pectoral muscle. 3. femoral muscle. 4. deltoid muscle.

2. weight in kilograms. RATIONALE: To calculate drug dosages for a child, the physician might use a formula that involves the child's weight in kilograms. A second recommended method involves the child's body surface area. Using weight in pound and ounces or height for dosage calculation isn't recommended.

To calculate drug dosages for a 4-year-old child, the physician might use a formula that involves the child's: 1. weight in pounds and ounces. 2. weight in kilograms. 3. height in inches. 4. height in centimeters.

4. Supine RATIONALE: The nurse should place the infant in the supine position on the caregiver's lap because it hyperextends the infant's neck, promoting thyroid palpation. A prone position wouldn't allow an adequate area for palpation. A sitting position is appropriate when assessing the thyroid gland of an older child or an adult. An infant can't stand, so this position is inappropriate.

To examine an infant's thyroid gland, the nurse should place the infant in which position? 1. Prone 2. Sitting 3. Standing 4. Supine

1. Hirschsprung's disease RATIONALE: Failure to pass meconium is an important diagnostic indicator for Hirschsprung's disease. Hirschsprung's disease is a potentially life-threatening congenital large-bowel disorder characterized by the absence or marked reduction of parasympathetic ganglion cells in a segment of the colorectal wall; narrowing impairs intestinal motility and causes severe, intractable constipation leading to partial or complete colonic obstruction. Celiac disease, intussusception, and abdominal wall defects aren't associated with failure to pass meconium.

Twenty-four hours after birth, a neonate hasn't passed meconium. The nurse suspects which condition? 1. Hirschsprung's disease 2. Celiac disease 3. Intussusception 4. Abdominal wall defect

3. Ignore the behavior when it happens. RATIONALE: Ignoring tantrums is the best advice because paying attention to the undesirable behavior can reinforce it. Changing settings can actually increase the tantrum behavior. Allowing the toddler more choices may also increase tantrum behavior if the toddler is unable to follow through with choices. It's ill-advised to give into the toddler's demands because doing so only promotes tantrum behavior.

What advice should a nurse give to the parents of a 2-year-old child who frequently throws temper tantrums? 1. Move the toddler to a different setting. 2. Allow the toddler more choices. 3. Ignore the behavior when it happens. 4. Give into the toddler's demands.

2. 93 mm Hg RATIONALE: The normal range for systolic blood pressure in preschoolers is 82 to 110 mm Hg. The normal range for diastolic blood pressure is 50 to 78 mm Hg.

What is a normal systolic blood pressure for a 3-year-old child? 1. 60 mm Hg 2. 93 mm Hg 3. 120 mm Hg 4. 150 mm Hg

3. Decreased blood pressure RATIONALE: The nurse should immediately report decreased blood pressure because it's a late sign of severe dehydration. This delayed decrease occurs because compensatory mechanisms in children are able to sustain blood pressure in the low-normal range for some time. Irritability, capillary refill less than 2 seconds, tachycardia, dry skin, and dry mucous membranes are all early signs of dehydration.

When a nurse assesses a 2-year-old child with suspected dehydration, which condition should be reported to the physician immediately? 1. Irritability for the past 12 hours 2. Capillary refill less than 2 seconds 3. Decreased blood pressure 4. Tachycardia, dry skin, and dry mucous membranes

4. tachypnea. RATIONALE: The nurse would expect to see tachypnea because the body compensates for metabolic acidosis via the respiratory system, which tries to eliminate the buffered acids by increasing alveolar ventilation through deep, rapid respirations. Altered WBC and platelet counts aren't specific signs of metabolic imbalance.

When caring for a 12-month-old infant with dehydration and metabolic acidosis, the nurse expects to see: 1. a reduced white blood cell (WBC) count. 2. a decreased platelet count. 3. shallow respirations. 4. tachypnea.

2. The lower conjunctival sac RATIONALE: Ophthalmic ointment is best instilled in the lower conjunctival sac.

Where should a nurse instill an ophthalmic ointment in a 6-year-old child? 1. The sclera 2. The lower conjunctival sac 3. The upper conjunctival sac 4. The outer canthus

3. Nausea and vomiting RATIONALE: Digoxin toxicity in infants and children may present with nausea, vomiting, anorexia, or a slow, irregular heart rate. Weight gain, tachycardia, and seizures aren't findings in digoxin toxicity.

Which toxic adverse reaction should the nurse monitor for in a toddler taking digoxin (Lanoxin)? 1. Weight gain 2. Tachycardia 3. Nausea and vomiting 4. Seizures

2. "This behavior is normal for a 6-month-old infant." RATIONALE: The nurse should say this behavior is normally seen because an infant typically transfers objects from one hand to another between ages 6 and 7 months, so the infant is demonstrating normal developmental behavior. Placing objects in a container occurs by age 12 months so the nurse doesn't need to ask about this milestone. Telling the mother not to worry and that the baby will catch up is not only ineffective communication, it's also unnecessary because the infant is exhibiting normal behavior.

While performing an assessment, a nurse observes a 6-month-old infant transferring an object from one hand to another. The mother tells the nurse this is a new behavior and asks if it is normal. What is the best response by the nurse? 1. "Your baby has very advanced motor skills." 2. "This behavior is normal for a 6-month-old infant." 3. "Can your baby move the object into a container?" 4. "Don't worry. Your baby will catch up soon."

1. production of thick, sticky mucus RATIONALE: Cystic fibrosis is associated with the production of thick, sticky mucus. Cystic fibrosis isn't associated with harsh, nonproductive coughing or with stridor or unilateral decrease in breath sounds.

A 9-year-old child is admitted to the pediatric unit for treatment of cystic fibrosis. A nurse assessing the child's respiratory status should expect to identify: 1. production of thick, sticky mucus 2. harsh, nonproductive cough 3. stridor 4. unilateral decrease in breath sounds

1. Sitting without support RATIONALE: According to the Denver Developmental Screening Test, most infants should be able to sit unsupported by age 7 months. Saying two words is expected of a 15-month-old infant. By 17 months, the toddler should be able to feed himself with a spoon. A 10-month-old infant should be able to play patty-cake.

A nurse is assessing an 8-month-old infant during a wellness checkup. Which action is a normal developmental task for an infant this age? 1. Sitting without support 2. Saying two words 3. Feeding himself with a spoon 4. Playing patty-cake

3. A lack of blood return RATIONALE: Infants and children have small, fragile veins, making a lack of a blood return normal. Erythema, pain, edema at the site or around it, blanching, and streaking are signs of infiltration. The infusion should be discontinued immediately if any of these signs are observed

A nurse is assessing an I.V. in an infant. Which assessment finding is considered normal? 1. Erythema and pain 2. Edema 3. A lack of blood return 4. Blanching or streaking along the vein

4. Irritability RATIONALE: An infant with increased ICP is commonly fussy, irritable, and restless at first as a result of a headache cause by the ICP. Vomiting occurs later. Papilledema is a late sign of increased ICP that may not be evident. Changes in vital signs occur later; pressure on the brainstem slows pulse and respiration.

A nurse is assessing an infant for signs of increased intracranial pressure (ICP). What is the earliest sign of increased ICP in an infant? 1. Vomiting 2. Papilledema 3. Vital sign changes 4. Irritability

1. providing an analgesic during the last days and hours is an ethically appropriate nursing action. RATIONALE: The nurse's action should be based on the fact that all clients, regardless of age, have the right to die with dignity and to be free of pain. Assisted suicide requires some action on the part of the client, which isn't possible in the case a 1-month-old infant. The parent's decision doesn't eliminate the nurse's ethical obligation to the infant and to the nursing profession. Withholding the opioid analgesic isn't appropriate because it isn't known that administering the drug would hasten death in this case.

A 1-month-old infant in the neonatal intensive care unit is dying. His parents request that a nurse give the infant an opioid analgesic. The infant's heart rate is 68 beats/minute and his respiratory rate is 18 breaths/minute. He is on room air; oxygen saturation is 92%. The nurse's response to the parents' request should be based on the fact that: 1. providing an analgesic during the last days and hours is an ethically appropriate nursing action. 2. withholding the opioid analgesic during the last days and hours is an ethical duty; administering it would represent assisted suicide. 3. administering an analgesic during the last days and hours is the parents' ethical decision. 4. withholding the opioid analgesic is clinically appropriate because administering it would hasten the infant's death.

2. they contain high levels of phenylalanine. RATIONALE: PKU is an inherited disorder characterized by the inability to metabolize phenylalanine, an essential amino acid. Phenylalanine accumulation in the blood results in central nervous system damage and progressive mental retardation. However, early detection of PKU and dietary restriction of phenylalanine can prevent disease progression. Intake of high-protein foods, such as meats and dairy products, must be restricted throughout life because they contain large amounts of phenylalanine.

A 10-month-old child with phenylketonuria (PKU) is being weaned from breast-feeding. When teaching the parents about the proper diet for their child, the nurse should stress the importance of restricting meats and dairy products because: 1. they're difficult for clients with PKU to digest. 2. they contain high levels of phenylalanine. 3. they aren't well tolerated in children with PKU until after age 2. 4. they lack phenylalanine, which stimulates muscle growth.

2. Disturbed body image RATIONALE: A client with anorexia nervosa has a body image disturbance and views herself as fat despite physical evidence to the contrary. One goal of nursing care is to help her develop realistic perceptions of her body. Although this adolescent has expressed concern about weight gain from I.V. fluids, no information suggests she'll refuse treatment; therefore, a nursing diagnosis of Noncompliance isn't warranted. Likewise, no evidence supports the nursing diagnoses of Complicated grieving and Grieving.

A 13-year-old with anorexia nervosa is admitted to the facility for I.V. fluid therapy and nutritional management. She says she's worried that the I.V. fluids will make her gain weight. Which nursing diagnosis is most appropriate? 1. Noncompliance (dietary regimen) 2. Disturbed body image 3. Complicated grieving 4. Grieving

2. "Do you give the baby a bottle to take to bed?" RATIONALE: In a young child, the eustachian tube is relatively short, wide, and horizontal, promoting drainage of secretions from the nasopharynx into the middle ear. Therefore, asking if the child takes a bottle to bed is appropriate because drinking while lying down may cause fluids to pool in the pharyngeal cavity, increasing the risk of otitis media. Asking if the parent noticed earwax, or cerumen, in the external ear canal is incorrect because wax doesn't promote the development of otitis media. During shampooing, water may become trapped in the external ear canal by large amounts of cerumen, possibly causing otitis external (external ear inflammation) as opposed to internal ear inflammation. Asking if the infant can combine two words is incorrect because a 10-month-old child isn't expected to do so.

A 10-month-old child with recurrent otitis media is brought to the clinic for evaluation. To help determine the cause of the child's condition, the nurse should ask the parents: 1. "Does water ever get into the baby's ears during shampooing?" 2. "Do you give the baby a bottle to take to bed?" 3. "Have you noticed a lot of wax in the baby's ears?" 4. "Can the baby combine two words when speaking?"

3. Keep him in a comfortable position and apply ice to the injured shoulder. RATIONALE: Ice should be applied first to reduce swelling and pain. The client should also be helped into a comfortable position. The nurse shouldn't apply warm compresses because it may increase swelling and cause bleeding into the injured tissue. Demonstrating full range of motion of the left arm may cause further damage to the injured area. In the emergency department, the nurse must have a physician's order to administer an analgesic.

A 10-year-old boy falls, injures his left shoulder, and is taken to the emergency department. While the client waits to be seen by the physician, what intervention should the nurse perform first? 1. Apply a warm compress to the injured shoulder. 2. Ask him to demonstrate full range of motion of his left arm. 3. Keep him in a comfortable position and apply ice to the injured shoulder. 4. Give him a nonopioid analgesic for pain.

2. After obtaining blood cultures, the nurse should insert an I.V. catheter and begin antibiotic and I.V. therapy as ordered. RATIONALE: Immediate antibiotic administration has been found to lower mortality rates from inhalation anthrax. Supportive care is essential to successful treatment, so the nurse should obtain blood cultures and immediately start an I.V. and antibiotic therapy. Inhalation anthrax is caused by inhalation of aerosolized anthrax spores, and isn't transmitted from human-to-human contact. Although standard precautions should be upheld, the health care team doesn't need special protective equipment, such as an N-95 respirator mask, and the client doesn't require special isolation, such as a negative-pressure isolation room. Because the client's condition may deteriorate rapidly as anthrax toxins are released into the systemic circulation, he'll most likely require admission to an intensive care unit (not a medical-surgical unit) for monitoring.

A 10-year-old child arrives in the emergency department with suspected inhalation anthrax. Which intervention should the nurse perform first? 1. The nurse and other members of the health care team should put on N-95 respirator masks. 2. After obtaining blood cultures, the nurse should insert an I.V. catheter and begin antibiotic and I.V. therapy as ordered. 3. The nurse should move the client to a negative-pressure isolation room. 4. The nurse should prepare to admit the client to a medical-surgical unit.

1. Taking vital signs every 4 hours and obtaining daily weight RATIONALE: Because major complications — such as hypertensive encephalopathy, acute renal failure, and cardiac decompensation — can occur, monitoring vital signs (including blood pressure) is an important measure for a child with acute glomerulonephritis. Obtaining daily weight and monitoring intake and output also provide evidence of the child's fluid balance status. Sodium and water restrictions may be ordered depending on the severity of the edema and the extent of impaired renal function. Typically, protein intake remains normal for the child's age and is only increased if the child is losing large amounts of protein in the urine. Checking urine specimens for protein and specific gravity and daily monitoring of serum electrolyte levels may be done, but their frequency is determined by the child's status. These actions are less important nursing measures in this situation.

A 10-year-old child diagnosed with acute glomerulonephritis is admitted to the pediatric unit. The nurse should ensure that which action is a part of the child's care? 1. Taking vital signs every 4 hours and obtaining daily weight 2. Obtaining a blood sample for electrolyte analysis every morning 3. Checking every urine specimen for protein and specific gravity 4. Ensuring that the child has accurate intake and output and eats a high-protein diet

4. The parents choose to leave to let the child build a relationship with the staff. RATIONALE: The parents leaving indicates more teaching is needed. The parents of an adolescent might leave to help the teen maintain a fragile identity, but a 10-year-old child would prefer to have his parents with him. Expected outcomes of support and teaching for a child and parents new to the hospital would include the parents' relating readily to the staff and calmly with the child, the child's accepting and responding positively to comforting measures, and the child's discussing procedures and activities without evidence of anxiety.

A 10-year-old child is in the hospital for the first time. The nurse has provided support and teaching to help the family and child adjust and to reduce their anxiety related to the child's hospitalization. Which situation indicates more teaching is needed? 1. The parents relate readily with the staff and calmly with the child. 2. The child accepts and responds positively to comforting measures. 3. The child discusses procedures and activities without evidence of anxiety. 4. The parents choose to leave to let the child build a relationship with the staff.

3. The child must be informed of the procedure and concur with his mother, who is giving written consent. RATIONALE: Assent, not consent, must be obtained from any child who is in the concrete operations thought stage of development (usually a child older than age 7). Assent involves knowledge of the procedure and agreement with the person authorized to give written informed consent. A child should always be notified of the treatment plan but he is too young to authorize consent. Careful ethical consideration should be given when using any person younger than age 18 in a research protocol.

A 10-year-old child must undergo a surgical procedure. Does the nurse need to obtain consent from the child? 1. The child doesn't need to know about the procedure because he is a minor. 2. The child must sign the form giving written informed consent. 3. The child must be informed of the procedure and concur with his mother, who is giving written consent. 4. The child only needs to know if the procedure is part of a research protocol.

3. Take the pump out of commission and locate a pump with a valid inspection sticker. RATIONALE: The nurse shouldn't use any equipment that doesn't have current inspection information. The pump could malfunction, causing harm to the patient. The nurse should remove the pump from service and locate a pump with the proper inspection information.

A 10-year-old child presents to the emergency department with dehydration. A physician orders 1 L of normal saline solution be administered at a rate of 60 ml/hour. While preparing the infusion, a nurse notices that the I.V. pump's safety inspection sticker has expired. Which action should the nurse take next? 1. After starting the fluids, contact the maintenance department and request a pump inspection. 2. Hang the fluids without the pump, carefully calculating the drip rate by visual inspection. 3. Take the pump out of commission and locate a pump with a valid inspection sticker. 4. Begin the infusion of the fluids while looking for a pump with a valid inspection sticker.

`1. "Once you die you never come back." 3. "My grandmother's death has been hard to understand." 4. "My grandmother died because she was sick and nothing could make her better." RATIONALE: By age 10, most children know that death is irreversible and final. However, a child may still have difficulty understanding the specific death of a loved one. School-age children should be able to identify cause-and-effect relationships, such as when a terminal illness causes someone to die. Adolescents, not school-age children, understand that death is a universal process. Preschoolers see death as temporary and may think of death as a punishment.

A 10-year-old child visits the pediatrician's office for his annual physical examination. When the nurse asks how he's doing, he becomes quiet and states that his grandmother died last week. A child this age is likely to make which statements about the concept of death? Select all that apply. 1. "Once you die you never come back." 2. "All people must die." 3. "My grandmother's death has been hard to understand." 4. "My grandmother died because she was sick and nothing could make her better." 5. "My grandmother is dead, but she'll come back." 6. "My grandmother died because someone in the family did something bad."

1. what the child knows about the disease. RATIONALE: When discussing a child's wishes for future care, a nurse must first identify what the child knows about the disease. How severe he perceives the illness to be will significantly affect his thoughts about realistic outcomes. A care plan proposed by a child who doesn't understand his disease process or prognosis won't effectively or realistically reflect his actual health status. A child who doesn't understand his disease process or prognosis might feel frightened or threatened by questions about what interventions he'd like to have implemented in the event of cardiac or respiratory arrest. Although exploring the child's experiences with death would be important, it shouldn't be the initial area of discussion.

A 12-year-old child has been receiving aggressive treatment for leukemia for the past year. His prognosis is poor and his parents would like to implement a do-not-resuscitate order. They ask the nurse to discuss their decision with their child because they can't bring themselves to talk with him about it. When approaching this subject with the child, the nurse must first assess: 1. what the child knows about the disease. 2. how the child would like to handle the care plan. 3. what interventions the child would like implemented in the event of cardiac or respiratory arrest. 4. the child's experiences with death.

1. The severity, location, and movement of pain RATIONALE: The pattern of pain is a reliable indicator of acute appendicitis. It begins with a severe colicky abdominal pain that gets progressively worse. The pain starts in the midabdominal (periumbilical) region and moves to the right lower quadrant after 6 to 12 hours. The degree of fever, a history of vomiting and diarrhea, and a history of irritability and lethargy are also clinical manifestations of acute appendicitis; however, these conditions can also be present in a number of other childhood illnesses so they aren't as reliable as the pattern of pain.

A 13-year-old adolescent may have appendicitis. Which finding is a reliable indicator of appendicitis? 1. The severity, location, and movement of pain 2. Fever 3. A history of vomiting and diarrhea, if present 4. A history of irritability and lethargy

2. inspect the girl for uneven shoulder height or uneven hip height. RATIONALE: Before deciding on any specific intervention, the school nurse should perform a basic assessment for scoliosis, including inspecting for uneven shoulder or hip height. The nurse will then have more specific information to give to the girl's parents.

A 13-year-old girl visits the school nurse because she's experiencing back pain, fatigue, and dyspnea. The nurse suspects that the girl may have scoliosis. The nurse should first: 1. send the girl home to recover. 2. inspect the girl for uneven shoulder height or uneven hip height. 3. arrange for the girl to have spinal X-rays as soon as possible. 4. ask the girl's parents to take her to a physician immediately.

1. Bryant's traction RATIONALE: Anticipating Bryant's traction is correct because this type of traction is used to treat femoral fractures or congenital hip dislocation in children younger than age 2 who weigh less than 30 lb (13.6 kg). Buck's extension traction is skin traction used for short-term immobilization or to correct bone deformities or contractures. Overhead suspension traction is used to treat fractures of the humerus; and 90-90 traction is used to treat femoral fractures in children older than age 2.

A 14-month-old child weighing 26 lb (11.8 kg) is admitted for traction to treat congenital hip dislocation. When preparing the child's room, the nurse anticipates using which traction system? 1. Bryant's traction 2. Buck's extension traction 3. Overhead suspension traction 4. 90-90 traction

4. Identity RATIONALE: According to Erikson's theory of personal development, the adolescent is in the stage of identity versus role confusion. During this stage, the body is changing as secondary sex characteristics emerge. The adolescent is trying to develop a sense of identity, and peer groups take on more importance. When an adolescent is hospitalized, she is separated from her peer group and her body image may be altered. This alteration in body image may interfere with the ongoing development of her identity. Toddlers are in the developmental stage of autonomy versus shame and doubt. Preschool children are in the stage of initiative versus guilt. School-age children are in the stage of industry versus inferiority.

A 14-year-old girl in skeletal traction for treatment of a fractured femur is expected to be hospitalized for several weeks. When planning care, the nurse should take into account the girl's need to achieve what developmental milestone? 1. Autonomy 2. Initiative 3. Industry 4. Identity

3. "We'll go to the physician if our child pulls on the ears or won't lie down." RATIONALE: The parents indicate full understanding of discharge teaching by repeating the specific, common signs of otitis media in toddlers, such as pulling on the ears and refusing to lie down, and by verbalizing the need for immediate follow-up care if these signs arise. Expressing that they should have gone to the physician sooner doesn't indicate effective teaching because it implies a sense of guilt — a feeling not promoted through teaching. Stating that they'll take the child to the physician's office every week addresses only weekly follow-up care and expressing that they're happy the problem is behind them is unrealistic because the child's condition may recur.

A 15-month-old child is being discharged after treatment for severe otitis media and bacterial meningitis. Which statement by the parents indicates effective discharge teaching? 1. "We should have gone to the physician sooner. Next time, we will." 2. "We'll take our child to the physician's office every week until everything is okay." 3. "We'll go to the physician if our child pulls on the ears or won't lie down." 4. "We're just so glad this is all behind us."

1. Instituting droplet precautions RATIONALE: Instituting droplet precautions is the priority for a newly admitted infant with meningococcal meningitis. Acetaminophen may be ordered but administering it doesn't take priority over instituting droplet precautions. Obtaining history information and orienting the parents to the unit don't take priority.

A 3-month-old infant with meningococcal meningitis has just been admitted to the pediatric unit. Which nursing intervention has the highest priority? 1. Instituting droplet precautions 2. Administering acetaminophen (Tylenol) 3. Obtaining history information from the parents 4. Orienting the parents to the pediatric unit

3. "For your protection, I can't keep this secret. After I notify the physician, we will need to involve your family. We want you to be safe." RATIONALE: In situations in which a client is a threat to himself, the nurse can't honor confidentiality. Because this adolescent has said he has a specific plan to commit suicide, the nurse must take immediate action to ensure his safety. The physician and mental health professionals should be notified as well as the client's family. The nurse should inform the adolescent that she must do this, while at the same time conveying a sense of caring and understanding. The local authorities needn't be notified in this situation.

A 15-year-old adolescent confides in the nurse that he has been contemplating suicide. He says he has developed a specific plan to carry it out and pleads with the nurse not to tell anyone. What is the nurse's best response? 1. "We can keep this between you and me, but promise me you won't try anything." 2. "I need to protect you. I will tell your physician, but we don't need to involve your parents. We want you to be safe." 3. "For your protection, I can't keep this secret. After I notify the physician, we will need to involve your family. We want you to be safe." 4. "I will need to notify the local authorities of your intentions."

4. "Can you tell me about the precautions you're taking now?" RATIONALE: An attitude that requests only the information the girl is willing to give is nonthreatening and nonjudgmental. This may enhance the girl's willingness to talk about her experiences, thus enabling the nurse to better assess her needs. Asking what the girl would like to know assumes the girl knows what information she needs. The precautions her friends are taking are irrelevant at this time. Referencing the girl's parents may make her defensive and fearful.

A 15-year-old girl visits the neighborhood clinic seeking information on "how to keep from getting pregnant." What should the nurse say in response to her request? 1. "What would you like to know?" 2. "Let's discuss what your friends are doing to keep from getting pregnant." 3. "Can you tell me if you've told your parents you're having sex?" 4. "Can you tell me about the precautions you're taking now?"

2. "Tell me what being pregnant would mean to you." RATIONALE: When talking with adolescents, it's best to get their viewpoints and thoughts before offering suggestions or giving advice. Doing so promotes therapeutic communication. Asking whether the girl's mother knows about her condition and desire to be pregnant or asking about the baby's father focuses attention away from the adolescent. A statement about the girl being too young to be pregnant is a value judgment and inappropriate for the nurse to make.

A 15-year-old girl with a urinary tract infection is admitted to the facility. She tells the nurse she hopes she's pregnant. How should the nurse respond? 1. "Does your mother know about this?" 2. "Tell me what being pregnant would mean to you." 3. "Congratulations. Does the baby's father know?" 4. "I hope you aren't pregnant; you're too young."

4. 4 hours RATIONALE: The duration of action for vasopressin administered subQ is 2 to 8 hours.

A 16-year-old adolescent sustains a severe head injury in a motor vehicle accident. He's admitted to the neurologic unit and subsequently develops neurogenic diabetes insipidus. The physician orders vasopressin (Pitressin), 5 units subcutaneously (subQ) twice per day. How long will the effects of the vasopressin last? 1. 5 minutes 2. 30 minutes 3. 1 hour 4. 4 hours

4. burning or tingling on vulva, perineum, or vagina. RATIONALE: Genital burning and tingling is the most common initial finding with primary genital or Type 2 herpes simplex. This symptom will advance to vesicular lesions rupturing into ulcerations, which then dry into a crusty erosion. Fever, headache, malaise, myalgia, regional lymphadenopathy, and dysuria, and urine retention are later findings in Type 2 herpes.

A 16-year-old girl visits the clinic for the first time. She tells the nurse that she has been exposed to herpes. Initially, with primary genital or Type 2 herpes simplex, the nurse should expect the girl to have: 1. dysuria and urine retention. 2. perineal ulcers and erosions. 3. bilateral inguinal lymphadenopathy. 4. burning or tingling on vulva, perineum, or vagina.

4. "He is only 17. He doesn't need an advance directive." RATIONALE: The parents stating that their son is too young for an advanced directive suggests that the parents don't fully understand the seriousness of their son's medical condition. Advance directives can be used for any client who has an irreversible condition. Stating that they shouldn't have allowed their son to go to a party shows a lack of knowledge about acquiring aspiration pneumonia. Being concerned about the need for a feeding tube and having an advance directive show an understanding of their son's condition.

A 17-year-old adolescent with a history of muscular dystrophy is admitted with aspiration pneumonia. The nurse asks the parents if the client has an advance directive. Which response by the parents leads the nurse to believe that the parents don't understand the severity of the client's medical condition? 1. "He has pneumonia; I shouldn't have let him go to that party last week." 2. "This is the third time he's had pneumonia in the past 6 months. I'm afraid he needs a feeding tube." 3. "Yes, he has an advance directive." 4. "He is only 17. He doesn't need an advance directive."

3. Letting the adolescent participate in his planning and scheduling of treatments RATIONALE: Because the adolescent is striving for independence, health care providers should promote self-reliance whenever possible, such as by letting him participate in planning and scheduling his treatments. He can help establish realistic goals and evaluation outcomes as well as help schedule procedures and chemotherapy doses to minimize lifestyle disruptions. Adolescents are oriented in the present and have relatively little concern for the long-term consequences of their behavior. Reprimanding him or threatening to discontinue care isn't likely to improve compliance and isn't in his best interest.

A 17-year-old adolescent with acute lymphocytic leukemia is discharged with written information about chemotherapy administration and his outpatient appointment schedule. He now is in the maintenance phase of chemotherapy but has missed clinic appointments for blood work and admits to omitting some chemotherapy doses. To improve his compliance, the nurse should include which intervention in the care plan? 1. Emphasizing the long-term consequences of noncompliance 2. Reprimanding the adolescent for failing to comply with his treatment 3. Letting the adolescent participate in his planning and scheduling of treatments 4. Threatening to discontinue care if he doesn't comply

4. Pyloric stenosis RATIONALE: Abdominal distention, forceful vomiting, dehydration, a palpable mass, and visible peristatic waves are classic symptoms of pyloric stenosis caused by hypertrophy of the circular pylorus muscle. Abdominal masses and abnormal peristalsis aren't necessarily related to colic or failure to thrive. Intussusception is usually characterized by acute onset and severe abdominal pain.

A 2-month-old infant is brought to the clinic by his mother. His abdomen is distended, and he has been vomiting forcefully and with increasing frequency over the past 2 weeks. On examination, the nurse notes signs of dehydration and a palpable mass to the right of the umbilicus. Peristaltic waves are visible, moving from left to right. The nurse should suspect which condition? 1. Colic 2. Failure to thrive 3. Intussusception 4. Pyloric stenosis

2. Welts or bruises in various stages of healing on the trunk 5. Circular, symmetrical burns on the lower legs 6. A parent who's hypercritical of the child and pushes the frightened child away RATIONALE: Injuries at various stages of healing in protected or padded areas can be signs of inflicted trauma, leading the nurse to suspect abuse. Burns that are bilateral as well as symmetrical and regular are typical of child abuse. The shape of the burn may resemble the item used to create it, such as a cigarette. When a child is burned accidentally, the burns form an erratic pattern and are usually irregular or asymmetrical. Pushing the child away and being hypercritical are typical behaviors of abusive parents. Superficial scrapes and bruises on the lower extremities are normal in a healthy, active child. A deep blue-black macular patch on the buttocks is more consistent with a Mongolian spot than a traumatic injury that would suggest abuse.

A 2-year-old boy is brought into the clinic with an upper respiratory tract infection. During the assessment, the nurse notes some bruising on the arms, legs, and trunk. Which findings should prompt the nurse to evaluate for suspected child abuse? Select all that apply. 1. A few superficial scrapes on the lower legs 2. Welts or bruises in various stages of healing on the trunk 3. A deep blue-black patch on the buttocks 4. One large bruise on the child's thigh 5. Circular, symmetrical burns on the lower legs 6. A parent who's hypercritical of the child and pushes the frightened child away

4. Immersing the child's hands in cold water RATIONALE: Vagal maneuvers, such as immersing the child's hands in cold water, are commonly tried first as a mechanism to decrease heart rate. Other vagal maneuvers include breath-holding, gagging, and placing the child's head lower than the rest of the body. Digoxin may be given after vagal maneuvers to help decrease heart rate; verapamil isn't recommended. Synchronized cardioversion may be necessary if vagal maneuvers fail and drugs are ineffective.

A 2-year-old child in the cardiac step-down unit is experiencing supraventricular tachycardia. Which intervention should be attempted first? 1. Administering digoxin (Lanoxin) I.V. 2. Administering verapamil (Calan) I.V. 3. Administering synchronized cardioversion 4. Immersing the child's hands in cold water

3. evaluate the child's neurologic status. RATIONALE: Petechiae across the child's chest, abdomen, and back are signs of meningitis. The priority is to evaluate neurologic status. Petechiae aren't allergic reactions, so the nurse shouldn't ask about allergies. Standard precautions should be used when there is risk of contacting body fluids. Contact precautions should be instituted for the client diagnosed with meningitis. Throat and ear examinations wouldn't be helpful in confirming a diagnosis of meningitis.

A 2-year-old child is admitted to the pediatric unit with fever, seizures, and vomiting. He's awake and alert. As the nurse is putting a gown on the child, the nurse notices petechiae across the child's chest, abdomen, and back. The nurse should: 1. question the mother about the child's allergies. 2. initiate standard precautions. 3. evaluate the child's neurologic status. 4. examine the child's throat and ears.

1. a barking cough. RATIONALE: Croup is an acute viral respiratory illness characterized by a barking cough. Fever is usually low grade. Croup has a gradual onset, and dysphagia isn't a symptom.

A 2-year-old child is brought to the emergency department with a history of upper airway infection that has worsened over the last 2 days. The nurse suspects the child has croup. Signs of croup include a hoarse voice, inspiratory stridor, and: 1. a barking cough. 2. a high fever. 3. sudden onset. 4. dysphagia.

4. allow the child to handle the stethoscope before listening to his lungs. RATIONALE: The best way to approach the 2-year-old is to allow the child to handle the stethoscope because toddlers are naturally curious about their environment. Letting them handle minor equipment is distracting and helps them gain trust with the nurse. The nurse should only expose one area at a time during assessment and should approach the child slowly and unhurriedly. The caregiver should be encouraged to hold and console her child and to comfort the child with objects with which he's familiar, and the child should be given limited choices to allow autonomy such as, "Do you want me to listen first to the front of your chest or your back?"

A 2-year-old child is brought to the emergency department with suspected croup. The child appears frightened and cries as the nurse approaches him. The nurse needs to assess the child's breath sounds. The best way to approach the 2-year-old child is to: 1. expose the child's chest quickly and auscultate breath sounds as quickly and efficiently as possible. 2. ask the mother to wait briefly outside until the assessment is over. 3. tell the child the nurse is going to listen to his chest with the stethoscope. 4. allow the child to handle the stethoscope before listening to his lungs.

1. Intercostal retractions RATIONALE: Clinical manifestations of respiratory distress include tachypnea, tachycardia, restlessness, dyspnea, intercostal retractions, and cyanosis. Bradycardia, LOC, and flushed skin aren't signs of increasing respiratory distress.

A 2-year-old child is brought to the emergency department with suspected croup. Which assessment finding reflects increasing respiratory distress? 1. Intercostal retractions 2. Bradycardia 3. Decreased level of consciousness (LOC) 4. Flushed skin

3. Social worker RATIONALE: The nurse should collaborate with the social worker to provide care for the child involved in a motor vehicle accident. After such a traumatic life event, this child's care will involve dealing with his emotional health as well as his physical recovery. Home health care isn't usually needed for this type of injury, and nutrition isn't a top priority problem for this child. There's nothing to suggest that the infectious disease nurse is required to care for this child.

A 6-year-old child was admitted to the pediatric unit after sustaining a broken leg in a motor vehicle accident. Which specialist would be most important to involve in this child's care during hospitalization? 1. Home care nurse 2. Nutritionist 3. Social worker 4. Infectious disease nurse

2. "Offer the medication diluted with chocolate milk or orange juice to make it more palatable." RATIONALE: Because liquid cyclosporine has a very unpleasant taste, diluting it with chocolate milk or orange juice will lessen the strong taste and help the child take the medication as ordered. It isn't acceptable to miss a dose because the drug's effectiveness is based on therapeutic blood levels, and skipping a dose could lower the level. Cyclosporine shouldn't be given by NG tube because it adheres to the plastic tube and, thus, all of the drug may not be administered. Taking the medication over a period of time could negatively affect the blood level.

A 2-year-old child with a low blood level of the immunosuppressive drug cyclosporine comes to a liver transplant clinic for her appointment. The mother says the child hasn't been vomiting and hasn't had diarrhea, but she admits that her daughter doesn't like taking the liquid medication. Which statement by the nurse is most appropriate? 1. "Let your daughter take her medication only when she wants it; it's okay for her to miss some doses." 2. "Offer the medication diluted with chocolate milk or orange juice to make it more palatable." 3. "Insert a nasogastric (NG) tube and administer the medication using the tube as ordered by the physician." 4. "Give the ordered dose a little bit at a time over 2 hours to ensure administration of the medication."

1. Suction the tracheostomy. RATIONALE: Diaphoresis, increased heart rate, increased respiratory effort, and decreased oxygen saturation are signs that mucus is partially occluding the airway. Therefore, the nurse should suction the tracheostomy first to prevent full occlusion. Turning the child to a side-lying position won't remove mucus from the airway. The child may require pain medication after his airway has been cleared if his condition warrants it. Chest physiotherapy will help drain excess mucus from the lungs but not from a tracheostomy.

A 2-year-old child with a tracheostomy suddenly becomes diaphoretic and has an increased heart rate, an increased work of breath, and a decreased oxygen saturation level. What should the nurse do first? 1. Suction the tracheostomy. 2. Turn the child to a side-lying position. 3. Administer pain medication. 4. Perform chest physiotherapy.

1. the infant should respond to gentle tactile or verbal stimulation. RATIONALE: An infant under moderate sedation should respond to verbal or tactile stimuli. Infants under general anesthesia have decreased or absent reflexes. Infants who undergo general or moderate sedation rarely remember the procedure. PCA pumps aren't used during sedation.

A 22-month-old infant is to have moderate sedation for an outpatient procedure. The nurse knows that: 1. the infant should respond to gentle tactile or verbal stimulation. 2. the infant's reflexes will be decreased or absent. 3. the infant will remember the procedure. 4. the infant will need a patient-controlled analgesia (PCA) pump during sedation.

1. Give 1 to 3 teaspoons of fluid every 10 to 15 minutes to set up a baseline for the child's tolerance. RATIONALE: Giving small amounts of fluid at frequent intervals is the first action a nurse should take when a child is vomiting. Doing so allows the nurse to observe the child's tolerance level. Simple sugars aren't a good source of hydration because of their osmotic affects. The nurse shouldn't wait 72 hours before taking action if a child is vomiting or has diarrhea. Toddlers can become dehydrated in a short time. A physician should see a child whose vomiting or diarrhea persists for 24 to 36 hours. Wet diapers are a good source of determining hydration; however, three wet diapers each day isn't a normal finding for toddler-age children. A hydrated toddler should have six to eight wet diapers per day.

A 29-month-old child who is dehydrated as a result of vomiting requires oral rehydration. Which concept regarding oral rehydration therapy should the nurse consider? 1. Give 1 to 3 teaspoons of fluid every 10 to 15 minutes to set up a baseline for the child's tolerance. 2. Sugar is a good source of nutrition when rehydrating a child. 3. If symptoms persist for more than 72 hours, contact the physician. 4. A child who has three wet diapers each day isn't considered dehydrated.

4. Right lateral RATIONALE: The toddler should be positioned on his right side because gravity contributes to increased blood flow to the right lung, thereby allowing for better gas exchange. Positioning the child prone, supine, or in the left lateral position doesn't allow for better gas exchange in this child.

A 2½-year-old child is being treated for left lower lobe pneumonia. In what position should the nurse position the toddler to maximize oxygenation? 1. Prone 2. Left lateral 3. Supine 4. Right lateral

1. ensuring that the suspected child abuse is reported to local authorities. RATIONALE: Nurses must report suspicions of child abuse to local authorities. The contact procedure may vary among hospitals, but the nurse is responsible for making the report. Reporting suspected abuse to the hospital's chief of pediatric services isn't appropriate. Contacting the infant's next of kin to begin discharge planning is inappropriate because the infant may not be discharged to his next of kin. Providing an anonymous tip isn't appropriate behavior for a professional nurse. The hospital record is important to the legal process, and the nurse must handle it professionally.

A 3-month-old infant is admitted to the hospital to rule out nonaccidental trauma. X-ray findings indicate a fractured right humerus, fractured ribs, and a fractured left scapula. In this situation, a nurse is responsible for: 1. ensuring that the suspected child abuse is reported to local authorities. 2. contacting the infant's next of kin to begin discharge planning. 3. reporting her suspicions to the hospital's chief of pediatric services. 4. contacting the local children's protective service office with an anonymous tip.

3. place the infant's arms in soft elbow restraints. RATIONALE: Soft restraints from the upper arm to the wrist are appropriate because they prevent the infant from touching his lip but allow him to hold a favorite item such as a blanket. Because they could damage the operative site, such objects as pacifiers, suction catheters, and small spoons shouldn't be placed in an infant's mouth after cleft palette repair. An infant in a prone position may rub his face on the sheets and traumatize the operative site. The suture line should be cleaned gently to prevent infection, which could interfere with healing and damage the cosmetic appearance of the repair. Dried blood collecting on the suture line can widen the scar.

A 3-month-old infant just had a cleft lip and palette repair. To prevent trauma to the operative site, the nurse should: 1. give the infant a pacifier to help soothe him. 2. lie the infant in the prone position. 3. place the infant's arms in soft elbow restraints. 4. avoid touching the suture line, even to clean.

4. cultural practice. RATIONALE: The nurse should consider that the lesions may be caused by cultural practice. Many Vietnamese perform coining, a cultural practice in which a coin is repeatedly rubbed lengthwise on the oiled skin to rid the body of a disease. Coining can produce weltlike lesions on the child's back or chest, and children subjected to the practice are commonly thought to have been abused. Interviewing the family and assessing its cultural background help distinguish between abuse and culture practice. Shingles, a form of herpes zoster, is a communicable disease usually affecting immunocompromised individuals and older adults. The disease produces small crusty pustules on the lower back and trunk. The description of the lesions doesn't fit those produced by an allergic reaction.

A 3-year-old Vietnamese child with a fever, decreased urine output, wheezing, and coughing is brought to the emergency department. On examination, the nurse discovers red, round, weltlike lesions on the child's upper back and chest. The nurse should consider that these lesions may be caused by: 1. shingles. 2. child abuse. 3. allergic reaction. 4. cultural practice.

2. Immediate isolation is required. RATIONALE: Immediate isolation is required because the incubation period for chickenpox is 2 to 3 weeks, and a client is commonly isolated 1 week after exposure to avoid the risk of an outbreak. A person is infectious from 1 day before eruption of lesions to 6 days after the lesions have formed crusts. Isolation 10 or 12 days after exposure would be too late, putting others at risk for exposure.

A 3-year-old child is admitted to the hospital with an acute exacerbation of asthma. The child's history reveals that the child was exposed to chickenpox 1 week ago. When would this child require isolation? 1. Isolation isn't required. 2. Immediate isolation is required. 3. Isolation is required 10 days after exposure. 4. Isolation is required 12 days after exposure.

62.5 milliliters/hour RATIONALE: To calculate the rate per hour for the infusion, the nurse should divide 500 ml by 8 hours: 500 ml ÷ 8 hours = 62.5 ml/hour.

A 3-year-old child is to receive 500 ml of dextrose 5% in normal saline solution over 8 hours. At what rate (in milliliters/hour) should the nurse set the infusion pump? Record your answer using one decimal place. Answer: milliliters/hour

3. how the child's condition today differs from his normal condition. RATIONALE: The nurse should ask how the child's condition differs from his normal condition in order to identify the chief complaint. Asking how long the child has been like this may be interpreted poorly by the caregiver. The nurse shouldn't ask if the child can walk without holding onto furniture because focusing on what the child can do — not on what he can't do — preserves the family's self-esteem. Focusing on negative aspects of the child's behavior, such as constant drooling, is inappropriate.

A 3-year-old child with Down syndrome is admitted to the pediatric unit with asthma. The child doesn't enunciate words well and holds onto furniture when he walks. The nurse should ask the mother: 1. how long the child has been like this. 2. if the child is able to walk without holding onto furniture. 3. how the child's condition today differs from his normal condition. 4. if the child always drools.

RATIONALE: A VSD is a small hole between the right and left ventricles. It's a common congenital heart defect and accounts for 20% to 30% of all heart lesions.

A 30-month-old toddler is being evaluated for a ventricular septal defect (VSD). Identify the area where a VSD occurs.

1. Move the family to an area where an assessment can be completed and call for a physician. RATIONALE: A head injury in an infant can be extremely serious. The nurse's priority should be to move the infant and family to an area where assessment and treatment can occur. Triaging the infant and having the parents wait for evaluation by a physician is inappropriate because of the potential seriousness of the injury. Although increased intracranial pressure can result from head trauma, it's unlikely that inserting a VP shunt would be the first treatment. The fact that the child was left unattended in an unsafe location is a significant safety issue, but notifying child protective services isn't a priority at this time.

A 4-month-old infant has been carried into the emergency department after falling off his parents' bed and hitting his head on the floor. What should the nurse do next? 1. Move the family to an area where an assessment can be completed and call for a physician. 2. Notify the supervisor that an operating room is needed because the physician will want to insert a ventriculoperitoneal (VP) shunt. 3. Assess the infant's vital signs in the triage area and instruct the family to wait until their names are called. 4. Call child protective services because of suspected child endangerment.

4. a low-intensity, painless electrical current is applied to the skin. RATIONALE: Because cystic fibrosis clients have elevated levels of sodium and chloride in their sweat, a sweat test is performed to confirm this disorder. The nurse should explain to the parents that after pilocarpine (a cholinergic medication that induces sweating) is applied to a gauze pad and placed on the arm, a low-intensity, painless electrical current is applied for several minutes. The arm is then washed off, and a filter paper is placed over the site with forceps to collect the sweat. Elevated levels of sodium and chloride are diagnostic of cystic fibrosis. No fasting is necessary before this test and no blood sample is required. A low-sodium diet isn't required before the test.

A 4-month-old infant is taken to the pediatrician by his parents because they're concerned about his frequent respiratory infections, poor feeding habits, frequent vomiting, and colic. The physician notes that the infant has failed to gain expected weight and recommends that the infant have a sweat test performed to detect possible cystic fibrosis. To prepare the parents for the test, the nurse should explain that: 1. the baby will need to fast before the test. 2. a sample of blood will be necessary. 3. a low-sodium diet is necessary for 24 hours before the test. 4. a low-intensity, painless electrical current is applied to the skin.

2. epidural hematoma. RATIONALE: An epidural hematoma is characterized by an initial loss of consciousness followed by transient consciousness leading to unconsciousness. Subdural hematoma results in rapid deterioration in level of consciousness. Subarachnoid hemorrhage causes irritability rather than loss of consciousness. As for a concussion, it may result in a brief loss of consciousness.

A 4-year-old child arrives in the emergency department with a history of transient consciousness and unconsciousness. The nurse should suspect: 1. subdural hematoma. 2. epidural hematoma. 3. subarachnoid hemorrhage. 4. concussion.

2. ALL affects all blood-forming organs and systems throughout the body. 4. Adverse effects of treatment include sleepiness, alopecia, and stomatitis. 5. There's a 95% chance of obtaining remission with treatment. RATIONALE: In ALL, abnormal white blood cells (WBCs) proliferate, but they don't mature past the blast phase. These blast cells crowd out the healthy WBCs, red blood cells, and platelets in the bone marrow, leading to bone marrow depression. The blast cells also infiltrate the liver, spleen, kidneys, and lymph tissue. Common adverse effects of chemotherapy and radiation include nausea, vomiting, diarrhea, sleepiness, alopecia, anemia, stomatitis, mucositis, pain, reddened skin, and increased susceptibility to infection. There's a 95% chance of obtaining remission with treatment. Leukemia is the most common form of childhood cancer. The child schould continue to brush his teeth, but he should use a soft toothbrush to minimize trauma. The child still needs appropriate discipline and limits. A lack of consistent parenting may lead to negative behaviors and fear.

A 4-year-old child has recently been diagnosed with acute lymphocytic leukemia (ALL). What information about ALL should the nurse provide when educating the client's parents? Select all that apply. 1. Leukemia is a rare form of childhood cancer. 2. ALL affects all blood-forming organs and systems throughout the body. 3. Because of the increased risk of bleeding, the child shouldn't brush his teeth. 4. Adverse effects of treatment include sleepiness, alopecia, and stomatitis. 5. There's a 95% chance of obtaining remission with treatment. 6. The child shouldn't be disciplined during this difficult time.

RATIONALE: The carotid artery should be used to check for a pulse when performing CPR on children and adults. The brachial pulse should be used when performing CPR on an infant.

A 4-year-old child is brought to the emergency department in cardiac arrest. The staff performs cardiopulmonary resuscitation (CPR). Identify the area where the child's pulse should be checked.

25 gtt/minute RATIONALE: When using a pediatric microdrip chamber, the number of milliliters per hour equals the number of drops per minute. If 25 ml/hour is ordered, the I.V. should infuse at 25 gtt/minute.

A 4-year-old child is ordered to receive 25 ml/hour of I.V. solution. The nurse is using a pediatric microdrip chamber to administer the medication. For how many drops per minute should the microdrip chamber be set? Record your answer using a whole number. Answer: gtt/minute

1. worsening dyspnea. RATIONALE: Dyspnea and other signs of respiratory distress signify fluid volume overload, which can occur quickly in a child as fluid shifts rapidly between the intracellular and extracellular compartments. Gastric distention suggests excessive oral (not I.V.) fluid intake or infection. Nausea and vomiting or an elevated temperature may indicate a fluid volume deficit, not an excess.

A 4-year-old child is receiving dextrose 5% in water and half-normal saline solution at 100 ml/hour. The nurse should suspect that the child's I.V. fluid intake is excessive if assessment reveals: 1. worsening dyspnea. 2. gastric distention. 3. nausea and vomiting. 4. a temperature of 102° F (38.9° C).

4. "I shouldn't let my daughter take bubble baths." RATIONALE: Saying that the child shouldn't take bubble baths demonstrates effective teaching because oils in the bubble bath preparation may irritate the urethra, contributing to UTIs. Girls and women should wipe the perineum from front to back, not back to front, to avoid contaminating the urinary tract with fecal bacteria. Although antibiotics are used to treat UTIs, they aren't given prophylactically. No evidence suggests that warm baths help prevent UTIs.

A 4-year-old girl has a urinary tract infection (UTI). Which statement by the mother demonstrates understanding of preventing future UTIs? 1. "I should help my child learn to wipe her bottom from back to front." 2. "When she starts urinating frequently, I should call the physician to request antibiotics." 3. "I will let her take a warm bath for 15 minutes each day." 4. "I shouldn't let my daughter take bubble baths."

1. Acetaminophen 225 mg (10 to 15 mg/kg/dose) q4h with intermittent doses of ibuprofen 180 mg (10 mg/kg/dose) q6h for temperature higher than 102.5° F (39.2° C) RATIONALE: The correct dosage schedule for acetaminophen is 10 to 15 mg/kg/dose every 4 hours, and for ibuprofen it's 10 mg/kg/dose every 6 hours for a temperature higher than 102.5° F. Aspirin shouldn't be given to children because of the association between aspirin use in children with influenza virus or chickenpox and Reye's syndrome (a life-threatening condition characterized by vomiting and lethargy that may progress to delirium and coma). Ibuprofen 5 mg/kg/dose is the correct dosage for a child with a temperature lower than 102.5° F.

A 4-year-old, 40-lb (18.1-kg) child is brought to the pediatrician's office. He has upper respiratory symptoms and has had a fever for 2 days. He's diagnosed with a viral illness, and the mother is instructed to treat him with rest, fluids, and antipyretics. Which medication dosage schedule is the most appropriate? 1. Acetaminophen 225 mg (10 to 15 mg/kg/dose) q4h with intermittent doses of ibuprofen 180 mg (10 mg/kg/dose) q6h for temperature higher than 102.5° F (39.2° C) 2. Aspirin 290 mg (65 mg/kg/24 hours) q6h with intermittent doses of acetaminophen 225 mg q4h 3. Acetaminophen 140 mg (5 to 10 mg/kg/dose) q4h for a temperature lower than 102.5° F 4. Acetaminophen 225 mg (10 to 15 mg/kg/dose) q4h with intermittent doses of ibuprofen 90 mg (5 mg/kg/dose) q6h for a temperature higher than 102.5° F

4. Heart rate less than 60 beats/minute RATIONALE: Bradycardia, a slow but steady heartbeat at a rate less than 60 beats/minute, is an ominous sign in children. Older children experiencing anaphylaxis initially demonstrate tachycardia in response to hypoxemia. When tachycardia can no longer maintain tissue oxygenation, bradycardia follows. The development of bradycardia usually precedes cardiopulmonary arrest. The average systolic blood pressure of children ages 1 to 7 can be determined by this formula: age in years plus 90. Thus, an average blood pressure for a 5-year-old child is 95 mm Hg. Urticaria should be treated after airway control has been established. The normal respiratory rate for a 5-year-old is 20 to 25 breaths/minute

A 5-year-old child is brought to the emergency department after being stung multiple times on the face by yellow jackets. Which symptom of anaphylaxis requires priority medical intervention? 1. Blood pressure of 95/50 mm Hg 2. Diffuse facial urticaria 3. Respiratory rate of 20 breaths/minute 4. Heart rate less than 60 beats/minute

4. Discuss the procedure with his parents. 1. Act out the procedure using a doll and biopsy kit. 5. Explain the discomforts that he'll feel. 2. Assure the child that the pain will go away. 3. Check the biopsy site for hemorrhage and infection. RATIONALE: The nurse must first discuss the procedure with the parents and encourage them to get involved with the plan for preparing the child. Next, the nurse should use play to teach the child about the procedure to help gain the child's confidence and put the child at ease. After the child is comfortable, the nurse can explain the discomfort he'll feel and then assure him that the pain will go away. Lastly, after the procedure, the nurse needs to check for bleeding, inflammation, and signs and symptoms of pain and infection.

A 5-year-old preschooler suspected of having leukemia is admitted to the hospital for diagnosis and treatment. The physician orders a bone marrow aspiration. Place the interventions below in ascending chronological order according to their importance. Use all options. 1. Act out the procedure using a doll and biopsy kit. 2. Assure the child that the pain will go away. 3. Check the biopsy site for hemorrhage and infection. 4. Discuss the procedure with his parents. 5. Explain the discomforts that he'll feel.

4. Pulse, skin turgor, and number of wet diapers the infant had in the last 24 hours RATIONALE: A sunken fontanel indicates dehydration. The nurse should assess pulse, skin turgor, and the number of wet diapers the infant had in the past 24 hours. These findings help evaluate the extent of dehydration. Temperature and respiratory rate may also be assessed, but these assessments don't provide the same detail about dehydration as pulse, skin turgor, and number of wet diapers.

A 6-month-old infant is brought to the clinic. The mother reports the infant has been lethargic. The infant's anterior fontanel is sunken. What other assessment data are a priority for the nurse to collect? 1. Temperature, pulse, and respiratory rate 2. Pulse, respiratory rate and skin turgor 3. Respiratory rate, skin and turgor 4. Pulse, skin turgor, and number of wet diapers the infant had in the last 24 hours

3. Fever, muscle weakness, and change in mental status RATIONALE: Severe West Nile virus infection (also called West Nile encephalitis or West Nile meningitis) affects the central nervous system and may cause headache, neck stiffness, fever, muscle weakness or paralysis, changes in mental status, and seizures. Such signs and symptoms as fever, rash, malaise, anorexia, nausea and vomiting, and lymphadenopathy suggest the mild, not severe, form of West Nile virus infection.

A 6-year-old child has tested positive for West Nile virus infection. The nurse suspects the child has the severe form of the disease when she recognizes which signs and symptoms? 1. Fever, rash, and malaise 2. Anorexia, nausea, and vomiting 3. Fever, muscle weakness, and change in mental status 4. Fever, lymphadenopathy, and rash

2. Aspirin RATIONALE: The parents require additional teaching if they state they will give their child aspirin because using aspirin during a viral infection has been linked to Reye's syndrome, a serious illness that can lead to brain damage and death in children. If the child requires medication for fever or discomfort, the nurse should recommend acetaminophen (Tylenol) or ibuprofen (Motrin). Naproxen (Aleve) isn't indicated for the treatment of fever.

A 6-year-old child is being discharged from the emergency department after being diagnosed with varicella (chickenpox). The nurse knows the parents need more medication teaching when they state they will give the child which over-the-counter medication? 1. Ibuprofen (Motrin) 2. Aspirin 3. Acetaminophen (Tylenol) 4. Naproxen (Aleve)

1. Fever, decreased level of consciousness (LOC), and impaired liver function RATIONALE: Reye's syndrome occurs in children with a history of a viral infection, varicella, or influenza. It's commonly associated with the administration of aspirin. The child presents with fever and decreased LOC, which can lead to coma and death. As the disease progresses, the child also develops impaired liver function. A child with joint pain, a red macular rash with a clear center, and a low-grade fever probably has rheumatic fever. A child presenting with peripheral edema, fever for more than 5 days, and a "strawberry tongue" probably has Kawasaki disease. A child with a red, raised "bull's eye" rash, malaise, and joint pain should be tested for Lyme disease.

A 6-year-old child with a history of varicella and aspirin intake is brought to the emergency department. The nurse suspects Reye's syndrome. Which assessment findings are consistent with this syndrome? 1. Fever, decreased level of consciousness (LOC), and impaired liver function 2. Joint inflammation, red macular rash with a clear center, and low-grade fever 3. Peripheral edema, fever for 5 or more days, and "strawberry tongue" 4. Red, raised "bull's eye" rash, malaise, and joint pain

1. perform chest physiotherapy every 4 hours. RATIONALE: The nurse should perform chest physiotherapy because it aids in loosening secretions in the entire respiratory tract. Pancreatic enzymes aid in the absorption of necessary nutrients — not in managing secretions. Oxygen therapy doesn't aid in loosening secretions and can cause carbon dioxide retention and respiratory distress in children with cystic fibrosis. A high-calorie diet is appropriate but doesn't facilitate respiratory effort.

A 7-year-old boy is hospitalized with cystic fibrosis. To help him manage secretions and avoid respiratory distress, the nurse should: 1. perform chest physiotherapy every 4 hours. 2. give pancreatic enzymes as ordered. 3. place the child in an oxygen tent and have oxygen administered continuously. 4. serve a high-calorie diet.

2. Apply a topical anesthetic to the I.V. site before the procedure. 3. Ask the child which hand he uses for drawing. RATIONALE: Topical anesthetics reduce the pain of a venipuncture. The topical anesthetic cream should be applied about 1 hour before the procedure and requires a physician's order. Asking which hand the child draws with helps to identify the dominant hand. The I.V. should be inserted into the opposite extremity so that the child can continue to play and do homework with minimal disruption. Younger school-age children don't have the capability for abstract thinking. The procedure should be explained using simple words, and definitions of unfamiliar terms should be provided. The child should have the procedure explained to him well before it takes place so that he has time to ask questions. Although the topical anesthetic will relieve some pain, there's usually some pain or discomfort involved in venipuncture, so the child shouldn't be told otherwise.

A 7-year-old child is admitted to the hospital for a course of I.V. antibiotics. What should the nurse do before inserting the peripheral I.V. catheter? Select all that apply. 1. Explain the procedure to the child immediately before the procedure. 2. Apply a topical anesthetic to the I.V. site before the procedure. 3. Ask the child which hand he uses for drawing. 4. Explain the procedure to the child using abstract terms. 5. Don't let the child see the equipment to be used in the procedure. 6. Tell the child that the procedure won't hurt.

3. Thickened mass RATIONALE: X-ray assessment of the lateral neck helps diagnose common respiratory emergencies in children. The lateral neck X-ray of a child with epiglottiditis shows a thickened mass. The steeple sign is found in a child with viral croup syndrome. Subglottic narrowing with membranous tracheal exudate is found in bacterial tracheitis. Supraglottic narrowing isn't a diagnostic indicator.

A 7-year-old child is admitted with epiglottiditis. Which is the most likely finding on a lateral neck X-ray in a child with this condition? 1. Supraglottic narrowing 2. Steeple sign 3. Thickened mass 4. Subglottic narrowing

3. Contact the authorities immediately. RATIONALE: When a nurse suspects abuse, she must contact the authorities immediately. Although speaking with the school psychologist may be helpful, the nurse shouldn't delay contacting the authorities. A family meeting might provide additional information, but the nurse must allow the authorities to investigate suspected abuse before confronting the child's parents. Because the child isn't in imminent distress, there's no need for an ambulance.

A 9-year-old child presents to a school nurse with complaints of arm and leg pain. Upon assessment, the nurse identifies numerous purple to yellow ecchymotic areas. When asked, the child says that the bruises are the result of "being in trouble at home." Which action by the nurse is most appropriate? 1. Arrange for the child to speak with the school psychologist as soon as possible. 2. Arrange for a meeting with the nurse, psychologist, school administrators, and the child's parents. 3. Contact the authorities immediately. 4. Contact an ambulance to transport the child to the emergency department.

3. "I'm giving the baby iron-fortified formula and a fluoride supplement because our water isn't fluoridated." RATIONALE: Iron-fortified formula supplies all the nutrients an infant needs during the first 6 months; however, fluoride supplementation is necessary if the local water supply isn't fluoridated. Before age 6 months, solid foods such as cereals aren't recommended because the GI tract tolerates them poorly. Also, a strong extrusion reflex causes the infant to push food out of the mouth. Mixing solid foods in a bottle with liquids deprives the infant of experiencing new tastes and textures and may interfere with development of proper chewing. Skim milk doesn't provide sufficient fat for an infant's growth.

A bottle-fed infant, age 3 months, is brought to the pediatrician's office for a well-child visit. During the previous visit, the nurse taught the mother about infant nutritional needs. Which statement by the mother during the current visit indicates effective teaching? 1. "I started the baby on cereals and fruits because he wasn't sleeping through the night." 2. "I started putting cereal in the bottle with formula because the baby kept spitting it out." 3. "I'm giving the baby iron-fortified formula and a fluoride supplement because our water isn't fluoridated." 4. "I'm giving the baby skim milk because he was getting so chubby."

1. "Our newborn daughter may be a carrier of the trait." RATIONALE: The father stating that his newborn daughter may be a carrier of the trait demonstrates understanding of X-linked recessive disorders. X-linked recessive genes behave like other recessive genes. A normal dominant gene hides the effects of an abnormal recessive gene. However, the gene is expressed primarily in male offspring because it's located on the X chromosome. Male offspring of a carrier mother and an unaffected father have a 50% chance of expressing the trait whereas female offspring are more likely to carry the trait than express it. These parents may produce offspring who neither express nor carry the trait for hemophilia.

A boy, age 2, is diagnosed with hemophilia, an X-linked recessive disorder. His parents and newborn sister are healthy. The nurse explains how the gene for hemophilia is transmitted. Which statement by the father indicates an understanding of X-linked recessive disorders? 1. "Our newborn daughter may be a carrier of the trait." 2. "If we have more sons, all of them will have hemophilia." 3. "All of our offspring will carry the trait for hemophilia." 4. "Our daughter will develop hemophilia when she gets older."

4. "I'll call my neighbor who's 2 months pregnant and tell her not to have contact with my son." RATIONALE: By saying she'll call her pregnant neighbor, the mother demonstrates that she understands the implications of rubella. Fetal defects can occur during the first trimester of pregnancy if the pregnant woman contracts rubella. Aspirin shouldn't be given to young children because aspirin has been implicated in the development of Reye's syndrome. Tylenol should be used instead of aspirin. Rubella immunization isn't recommended for children until ages 12 to 15 months. Having the measles (rubeola) won't provide immunity for rubella.

A boy, age 3, develops a fever and rash and is diagnosed with rubella. His mother has just given birth to a baby girl. Which statement by the mother best indicates that she understands the implications of rubella? 1. "I told my husband to give my son aspirin for his fever." 2. "I'll ask the physician about giving the baby an immunization shot." 3. "I don't have to worry because I've had the measles." 4. "I'll call my neighbor who's 2 months pregnant and tell her not to have contact with my son."

1. inform the woman that the Health Insurance Portability and Accountability Act (HIPAA) prevents her from disclosing the information. RATIONALE: The nurse has a legal responsibility to follow HIPAA guidelines regarding client information. She must never disclose information, such as a room number, about a client or his condition without the consent of the client or family members. The nurse doesn't need to call security at this point.

A charge nurse is at the front desk when a woman demands information about a child who has been admitted on the unit. The nurse should: 1. inform the woman that the Health Insurance Portability and Accountability Act (HIPAA) prevents her from disclosing the information. 2. direct the woman to the child's room. 3. call security because of the woman's angry demeanor. 4. refer to the child's chart and give the woman basic information.

1. A stable 6-month-old infant with pneumonia RATIONALE: Of the clients listed, the most appropriate assignment for a licensed practical nurse is the stable 6-month-old infant admitted with pneumonia. Because they require close assessment, a newly admitted infant with bronchiolitis, a 15-year-old with diabetic ketoacidosis, and a 12-year-old who requires chemotherapy should be cared for by a registered nurse.

A charge nurse is making client care assignments. Which client is most appropriate for a licensed practical nurse? 1. A stable 6-month-old infant with pneumonia 2. A newly admitted 1-month-old infant with bronchiolitis 3. A newly admitted 15-year-old child with diabetic ketoacidosis 4. A 12-year-old child admitted for chemotherapy

1. talk with the unit nurse about the assignment and why she doesn't want to take care of the child tonight. RATIONALE: It's the charge nurse's responsibility to make clinical assignments based on safety and client needs. Talking about her reasons for not wanting to care for the child may enable the unit nurse to recognize her duty to the child and to the unit. Continuity of care is in the child's best interest. A nurse should never promise to perform a duty or action; negative feelings will result if she can't keep her promise. Unless there's a valid reason to assign the child's care to another nurse, the charge nurse should talk with the unit nurse before making the assignment.

A charge nurse is making evening-shift assignments. A unit nurse has requested that she not be assigned to care for a particular child because she has cared for him for the past four shifts and hasn't been able to leave on time. The charge nurse knows that the child and his family have bonded with the unit nurse. The charge nurse's best action would be to: 1. talk with the unit nurse about the assignment and why she doesn't want to take care of the child tonight. 2. promise the unit nurse that she will help her so she can leave on time. 3. assign the child's care to the unit nurse anyway. 4. acknowledge the unit nurse's request and assign the child's care to another nurse.

2. talk with the nurses about proper technique and the risk of infection resulting from improper technique. RATIONALE: A nurse has the responsibility to do no harm. If a nurse observes other health care professionals implementing inappropriate practices, she should address the problem. The charge nurse's first action should be to counsel the nurses on correct I.V. techniques. She should contact the nurse-manager if the behaviors continue. She should never ignore the situation or talk with the child's parents regarding the incident unless a situation develops that requires the parents to be informed.

A charge nurse observes two nurses using inappropriate technique when starting an I.V. on a child. The charge nurse should first: 1. ignore the situation. 2. talk with the nurses about proper technique and the risk of infection resulting from improper technique. 3. talk with the nurse-manager about her observations. 4. talk with the child's parents about infection control.

2. A 10-year-old child with asthma whose oxygen saturation levels are dropping RATIONALE: Decreasing oxygen saturation levels indicate difficulty breathing and increased work of breathing. Airway, breathing, and circulation always take priority. The children complaining of pain and waiting for discharge instructions don't take priority because administration of pain medication and reviewing discharge instructions can be delegated to another registered nurse. Moving a client from the bed to the chair can be delegated to a nursing assistant.

A charge nurse on the pediatric unit informs the staff nurse that four children require attention. Which child should the nurse see first? 1. An 8-year-old child admitted from the postanesthesia care unit who's complaining of pain 2. A 10-year-old child with asthma whose oxygen saturation levels are dropping 3. A 7-year-old child whose mother is waiting for discharge instructions 4. A 9-year-old child with a broken leg who wants help moving from the bed to the chair

4. administer an antiemetic 30 to 60 minutes before the next chemotherapy session. RATIONALE: The nurse should administer an antiemetic 30 to 60 minutes before the chemotherapy session because antiemetics counteract nausea most effectively when given before administration of an agent that causes nausea. Antiemetics also work better when given continuously rather than as needed. A high-fiber diet or allopurinol wouldn't prevent or reduce nausea and vomiting. Increasing fluid intake before the next chemotherapy session would only worsen the nausea and could cause more vomiting.

A child experiences nausea and vomiting after receiving cancer chemotherapy drugs. To help prevent these problems from recurring, the nurse should: 1. provide a high-fiber diet before the next chemotherapy session. 2. administer allopurinol (Zyloprim) 2 hours before the next chemotherapy session. 3. encourage increased fluid intake before the next chemotherapy session. 4. administer an antiemetic 30 to 60 minutes before the next chemotherapy session.

2. Decreased pulmonary wheezing RATIONALE: Methylxanthines such as theophylline are highly potent bronchodilators used to relieve asthma symptoms. The bronchodilation will result in decreased wheezing. None of the other options are seen after administration of theophylline.

A child has just received a dose of theophylline I.V. for asthma. What assessment finding should the nurse expect? 1. Increased coughing because of postnasal drip 2. Decreased pulmonary wheezing 3. Stridor 4. White blood cell count of 12,000/μl

1. skin traction applied to a lower extremity, with the extremity suspended above the bed. RATIONALE: Russell traction is skin traction applied to a lower extremity, with the extremity suspended above the bed and a sling placed under the knee. Skeletal traction applied to a lower extremity is called 90-90 traction. Skin traction applied to an extended lower extremity is called Buck's extension traction. Skin traction applied bilaterally to the lower extremities is called Bryant's traction.

A child is admitted to the pediatric unit with a fracture of the hip. The physician orders Russell traction. This type of traction is: 1. skin traction applied to a lower extremity, with the extremity suspended above the bed. 2. skeletal traction applied to a lower extremity. 3. skin traction applied to an extended lower extremity. 4. skin traction applied bilaterally to the lower extremities.

2. Instituting seizure precautions RATIONALE: A serum sodium level of 118 mEq/L indicates severe hyponatremia, which places the client at risk for seizures. Therefore, instituting seizure precautions takes highest priority. Fluid and sodium replacement should be done rapidly. Diuretic therapy isn't indicated because it may cause additional sodium loss. In a child with hyperkalemia, administering sodium bicarbonate would be appropriate because it promotes movement of potassium into the intracellular spaces.

A child is admitted to the pediatric unit with a serum sodium level of 118 mEq/L. Which nursing action takes highest priority at this time? 1. Replacing fluids slowly as ordered 2. Instituting seizure precautions 3. Administering diuretic therapy as ordered 4. Administering sodium bicarbonate as ordered

1. tachycardia. RATIONALE: Proventil is a beta-adrenergic blocker bronchodilator used to relieve bronchospasms associated with acute or chronic asthma or other obstructive airway diseases. Signs and symptoms of proventil toxicity that the nurse should instruct the parents to watch for include tachycardia, restlessness, nausea, vomiting, and dizziness. Unusually slow respirations, urine retention, and constipation aren't associated with proventil toxicity.

A child is being discharged with proventil (Albuterol) nebulizer treatments. The nurse should instruct the parents to watch for: 1. tachycardia. 2. bradypnea. 3. urine retention. 4. constipation.

2. reduce the excretion of urinary protein. RATIONALE: The primary goal of treatment for a child with nephrotic syndrome is to reduce excretion of urinary protein and maintain protein-free urine. Nephrotic syndrome isn't associated with hematuria, cardiac failure, or hypertension. Fluid restriction isn't warranted.

A child is diagnosed with nephrotic syndrome. When planning the child's care, the nurse understands that the primary goal of treatment is to: 1. manage urinary changes by monitoring fluid intake and output and observing for hematuria. 2. reduce the excretion of urinary protein. 3. help prevent cardiac or renal failure by carefully monitoring fluid and electrolyte balance. 4. decrease edema and hypertension through bed rest and fluid restriction.

2. Small white spots that adhere to the hair shaft, close to the scalp RATIONALE: The small white spots that adhere to the hair shafts are the eggs, or nits, of lice. These are easy to see and can't be brushed off like dandruff. Flaking of the scalp may indicate dandruff or a dry scalp. Scaly pustules, resulting from the scratching, may accompany a lice infestation, but nits would also be found on the hair shafts.

A child is sent to the school nurse because, according to his teacher, he's constantly scratching his head. When the nurse assesses his hair and scalp, she finds evidence of lice. What did the nurse see? 1. Flaking of the scalp with pink, irritated skin exposed 2. Small white spots that adhere to the hair shaft, close to the scalp 3. Scaly, circumscribed patches on the scalp, with mild alopecia in these areas 4. Multiple tiny pustules on the scalp with no abnormal findings on the hair shafts

1. Snellen's test RATIONALE: To help diagnose amblyopia, the child will undergo the Snellen's test. Snellen's test assesses visual acuity and a child with amblyopia will have decreased visual acuity in the affected eye. The near vision test evaluates near vision. Weber's test is used to determine hearing loss. The peripheral vision test evaluates peripheral vision.

A child is suspected of having amblyopia ("lazy eye"). To help diagnose this disorder, the child will undergo which test? 1. Snellen's test 2. Near vision test 3. Weber's test 4. Peripheral vision test

1. Conduct brief teaching sessions, provide written materials during each visit, and repeat information as appropriate. RATIONALE: The nurse should provide simple instructions in short sessions, provide written materials, repeat information, and allow time for questions because these are the most effective teaching methods. Asking the parents to spend the day at the facility, calling the parents at home, and sending the parents the drug's package insert are ineffective teaching strategies because they may be overwhelming for the parents and frustrating for the nurse.

A child is to receive valproic acid (Depakote) 10 mg/kg by mouth each day. When teaching the parents about the medication regimen, the nurse should use which approach? 1. Conduct brief teaching sessions, provide written materials during each visit, and repeat information as appropriate. 2. Ask the parents to spend an entire day at the facility so they can learn every detail about their child's care. 3. Call the parents at home and explain everything, allowing time for them to ask questions. 4. Send the parents the drug's package insert so they can become familiar with the medication.

3. Total protein RATIONALE: The nurse anticipates the physician will order a total protein test because negative nitrogen balance may result from inadequate protein intake. Measuring total iron-binding capacity and Hb levels would help detect iron deficiency anemia, not a negative nitrogen balance. The sweat test helps diagnose cystic fibrosis, not a negative nitrogen balance.

A child with a poor nutritional status and weight loss is at risk for a negative nitrogen balance. To help diagnose this problem, the nurse anticipates that the physician will order which laboratory test? 1. Total iron-binding capacity 2. Hemoglobin (Hb) 3. Total protein 4. Sweat test

1. Decreased urine output RATIONALE: The primary action of DDAVP is to stimulate water reabsorption by the kidneys, thereby decreasing the urine output. DDAVP has no effect on glucose levels, blood pressure, or nausea.

A child with diabetes insipidus receives desmopressin acetate (DDAVP). When evaluating for therapeutic effectiveness, the nurse should interpret which finding as a positive response to this drug? 1. Decreased urine output 2. Increased urine glucose level 3. Decreased blood pressure 4. Relief of nausea

2. Within 2 weeks RATIONALE: Bone marrow depression is most likely to occur 10 days after methotrexate is administered.

A child with leukemia has just completed a course of methotrexate therapy. How soon should the nurse expect to see signs of bone marrow depression in this client? 1. Within hours 2. Within 2 weeks 3. Within 1 month 4. After induction therapy is completed

3. 250 mg every 6 hours RATIONALE: First, the nurse determines the minimum dose: 50 mg × 10 kg = 500 mg/day 500 mg/4 doses (for administration every 6 hours) = 125 mg/dose. Next, the nurse determines the maximum dose: 100 mg × 10 kg = 1,000 mg/day 1,000 mg/4 doses = 250 mg/dose. Thus, the acceptable dosage range for this client is 125 to 250 mg every 6 hours.

A child with osteomyelitis is to receive nafcillin I.V. every 6 hours. Before administering the drug, the nurse calculates the appropriate dosage. The recommended dosage is 50 to 100 mg/kg daily; the child weighs 22 lb (10 kg). Which dosage is acceptable? 1. 50 mg every 6 hours 2. 100 mg every 6 hours 3. 250 mg every 6 hours 4. 500 mg every 6 hours

2. Drink plenty of fluids. 4. Report a sore throat to an adult immediately. 6. Wash hands before meals and after playing. RATIONALE: Fluids should be encouraged to prevent stasis in the bloodstream, which can lead to sickling. Sore throats and other cold symptoms should be promptly reported because they may indicate the presence of an infection, which can precipitate a crisis (red blood cells sickle and obstruct blood flow to tissues). Children with sickle cell anemia should learn appropriate measures to prevent infection, such as proper hand-washing techniques and good nutrition practices. Folic acid intake should be encouraged to help support new cell growth; new cells replace fragile, sickled cells. Warm packs should be applied to provide comfort and relieve pain; cold packs cause vasoconstriction. The child should maintain an active, normal life. When the child experiences a pain crisis, he limits his own activity according to his pain level.

A child with sickle cell anemia is being discharged after treatment for a crisis. Which instructions for avoiding future crises should the nurse provide to the client and his family? Select all that apply. 1. Avoid foods high in folic acid. 2. Drink plenty of fluids. 3. Use cold packs to relieve joint pain. 4. Report a sore throat to an adult immediately. 5. Restrict activity to quiet board games. 6. Wash hands before meals and after playing.

3. A recent episode of pharyngitis RATIONALE: A recent episode of pharyngitis is the most important factor in establishing the diagnosis of rheumatic fever. Although the child may have a history of fever or vomiting or lack interest in food, these findings aren't specific to rheumatic fever.

A child with suspected rheumatic fever is admitted to the pediatric unit. When obtaining the child's history, the nurse considers which information to be most important? 1. A fever that started 3 days ago 2. Lack of interest in food 3. A recent episode of pharyngitis 4. Vomiting for 2 days

2. Toy chest in front of a second-story, locked window RATIONALE: A toy chest in front of a second-story locked window displays misunderstanding because toddlers are able to climb on low furniture and open windows that may not always be locked, especially in the summer. In such situations, the child could fall out of the window. Keeping child safety latches on kitchen cabinets, turning pot handles toward the back of the stove, and setting the hot water heater at a nonscalding temperature are all safeguards against toddler injury. These safeguards demonstrate full understanding of a toddler's developmental abilities.

A child's parents state that they childproofed their home for their 2-year-old. During a home visit, the nurse discovers some situations that show the parents don't fully understand the developmental abilities of their toddler. Which situation displays misunderstanding by the parents? 1. Safety latches on kitchen cabinets 2. Toy chest in front of a second-story, locked window 3. Pot handles turned toward the back of the stove 4. Hot water heater temperature set at 120° F (48.9° C) or below

1. "The child will change position with minimal discomfort." RATIONALE: To prevent pressure ulcers, the child must turn and change positions periodically. However, during the acute phase of osteomyelitis, moving the affected leg may cause extreme pain and discomfort. Therefore, the nurse must support and handle the leg gently during turning and repositioning. Weight bearing is contraindicated because it may cause pathologic fractures. Ambulating with crutches is an inappropriate outcome because the child is restricted to bed rest and the affected leg is immobilized to limit the spread of infection. Participation in age-appropriate activities isn't a realistic outcome because an acutely ill child isn't likely to be interested in activities; this outcome would be suitable after the acute disease phase ends.

A child, age 10, is hospitalized for treatment of acute osteomyelitis. After assessing swelling and tenderness of the left tibia, the nurse initiates antibiotic therapy as ordered. The child's left leg is immobilized in a splint. What is an appropriate expected outcome for this child? 1. "The child will change position with minimal discomfort." 2. "The child will bear weight on the affected limb." 3. "The child will ambulate with crutches." 4. "The child will participate in age-appropriate activities."

1. Heart rate, respiratory rate, and blood pressure RATIONALE: The most important data to obtain on a child's arrival in the emergency department are vital sign measurements. The nurse should gather data about disease exposure, immunizations, and height and weight later.

A child, age 2, is brought to the emergency department after ingesting an unknown number of aspirin tablets about 30 minutes earlier. On entering the examination room, the child is crying and clinging to the mother. Which data should the nurse obtain first? 1. Heart rate, respiratory rate, and blood pressure 2. Recent exposure to communicable diseases 3. Number of immunizations received 4. Height and weight

3. "Does your child tug at either ear?" RATIONALE: Although all of the options are appropriate questions to ask when assessing a young child's ear problems, questions about the child's behavior, such as "Does your child tug at either ear?" are most useful because a young child usually can't describe symptoms accurately.

A child, age 2, with a history of recurrent ear infections is brought to the clinic with a fever and irritability. To elicit the most pertinent information about the child's ear problems, the nurse should ask the parent: 1. "Does your child's ear hurt?" 2. "Does your child have any hearing problems?" 3. "Does your child tug at either ear?" 4. "Does anyone in your family have hearing problems?"

2. "Tell me why you think this is your fault." RATIONALE: Having the mother explain why she feels the illness is her fault is appropriate because many parents feel responsible for their child's illness and may need instruction about the actual cause of the illness. Pointing out that the mother could have brought the child in sooner could increase the mother's feelings of guilt. Telling the mother not to cry or be upset ignores her feelings.

A child, age 3, is admitted to the pediatric unit with dehydration after 2 days of nausea and vomiting. The mother tells the nurse that her child's illness "is all my fault." How should the nurse respond? 1. "Maybe next time you'll bring the child in sooner." 2. "Tell me why you think this is your fault." 3. "Try not to cry in front of the child. It'll only upset her." 4. "Don't be so upset. Your child will be fine."

1. Severe sore throat, drooling, and inspiratory stridor RATIONALE: A child with acute epiglottiditis appears acutely ill and clinical manifestations may include drooling (because of difficulty swallowing), severe sore throat, hoarseness, a high temperature, and severe inspiratory stridor. A low-grade fever, stridor, and barking cough that worsens at night are suggestive of croup. Pulmonary congestion, productive cough, and fever along with nasal flaring, retractions, chest pain, dyspnea, decreased breath sounds, and crackles indicate pneumococcal pneumonia. A sore throat, fever, and general malaise point to viral pharyngitis.

A child, age 3, is brought to the emergency department in respiratory distress caused by acute epiglottiditis. Which clinical manifestations should the nurse expect to assess? 1. Severe sore throat, drooling, and inspiratory stridor 2. Low-grade fever, stridor, and a barking cough 3. Pulmonary congestion, a productive cough, and a fever 4. Sore throat, a fever, and general malaise

3. Risk for injury RATIONALE: Kawasaki disease, which affects young children, is characterized by acute systemic vasculitis. Risk for injury should receive priority because this inflammation of blood vessels leads to platelet accumulation and the formation of thrombi or obstruction in the heart and blood vessels. Approximately 10 days after the onset of the disease process, the platelet count rises and thrombi may form in the coronary arteries, leading to a myocardial infarction. The nurse must monitor the child closely for chest pain, cyanosis or pallor, and changes in the blood pressure. Diarrhea isn't a symptom of Kawasaki disease. Although Self-care deficit and Caregiver role strain may be appropriate diagnoses for this child, they don't take priority over Risk for injury.

A child, age 3, is hospitalized for treatment of Kawasaki disease. Which of these nursing diagnoses should receive priority in the child's care plan: 1. Self-care deficit 2. Diarrhea 3. Risk for injury 4. Caregiver role strain

3. Seizures RATIONALE: Chelation therapy removes lead by combining it with another substance to form a soluble compound that the kidneys can excrete. The nurse should stay alert for seizures because as lead is mobilized from bone and other tissues, the serum lead level rises rapidly, increasing the client's risk of seizures. Chelation therapy doesn't cause anaphylaxis, fever, chills, or heart failure.

A child, age 3, with lead poisoning is admitted to the facility for chelation therapy. The nurse must stay alert for which adverse effect of chelation therapy? 1. Anaphylaxis 2. Fever and chills 3. Seizures 4. Heart failure

2. coarctation of the aorta. RATIONALE: The nurse should suspect coarctation of the aorta because it causes signs of peripheral hypoperfusion, such as a weak femoral pulse and a bounding radial pulse. These signs are rare in patent ductus arteriosus, ventricular septal defect, and truncus arteriosus.

A child, age 4, is admitted with a tentative diagnosis of congenital heart disease. When assessment reveals a bounding radial pulse coupled with a weak femoral pulse, the nurse suspects that the child has: 1. patent ductus arteriosus. 2. coarctation of the aorta. 3. a ventricular septal defect. 4. truncus arteriosus.

3. Pinkish gray RATIONALE: The tympanic membrane normally appears pinkish gray, shiny, and translucent. A light pink, deep red, or yellowish white tympanic membrane is abnormal.

A child, age 4, is brought to the clinic for a routine examination. When observing the tympanic membrane, the nurse identifies which color as normal? 1. Light pink 2. Deep red 3. Pinkish gray 4. Yellowish white

3. Providing play situations that allow disclosure RATIONALE: The best nursing intervention is to provide play situations because through certain play situations, a sexually abused child can disclose information without actually talking about himself or herself. Avoiding touch would be inappropriate because an abused child needs to be touched and cared for like any other hospitalized child. The nurse can't restrict visitation unless the threat of repeated abuse exists while the child is hospitalized. The nurse shouldn't discourage discussion of the abuse if the child feels able to talk about it.

A child, age 4, is hospitalized because of alleged sexual abuse. What is the best nursing intervention for this child? 1. Avoiding touching the child 2. Preventing the suspected abuser from visiting the child 3. Providing play situations that allow disclosure 4. Discouraging the child from talking about what happened

1. Deficient fluid volume related to dehydration. RATIONALE: Tenting, which indicates decreased skin turgor, is normal only in elderly clients and results from decreased elastin content. However, in other adults and in children, tenting more commonly results from dehydration. This finding supports a nursing diagnosis of Deficient fluid volume related to dehydration. The other diagnoses are inappropriate because capillary fragility, altered tissue perfusion, and hypoxia rarely are associated with gastroenteritis.

A child, age 4, with a recent history of nausea, vomiting, and diarrhea is admitted to the pediatric unit with a diagnosis of gastroenteritis. During the physical examination, the nurse detects tenting. This finding supports a nursing diagnosis of: 1. Deficient fluid volume related to dehydration. 2. Risk for injury related to capillary fragility. 3. Ineffective peripheral tissue perfusion related to peripheral cyanosis. 4. Activity intolerance related to hypoxia

3. Up to 20 RATIONALE: A child may have up to 20 deciduous teeth by age 5. The first tooth usually erupts by age 6 months; the last, by age 30 months. Deciduous teeth usually are shed between ages 6 and 13.

A child, age 5, is brought to the pediatrician's office for a routine visit. When inspecting the child's mouth, the nurse expects to find how many teeth? 1. Up to 10 2. Up to 15 3. Up to 20 4. Up to 32

2. Instituting cardiac monitoring RATIONALE: Kawasaki disease sometimes causes cardiac complications, including arrhythmias. Therefore, instituting cardiac monitoring is the best action for detecting such complications. Auscultating for breath sounds, monitoring blood pressure, and assessing the skin daily are also important but not as important as cardiac monitoring.

A child, age 5, is hospitalized for treatment of Kawasaki disease. Which nursing action best identifies potential complications of this disease? 1. Auscultating breath sounds 2. Instituting cardiac monitoring 3. Monitoring blood pressure 4. Assessing the skin daily

3. monitor fluid intake and output. RATIONALE: The nurse should first monitor fluid intake and output because potassium shouldn't be added to the I.V. fluid until the child's kidney function is shown to be adequate, as indicated by balanced fluid intake and output and certain diagnostic test results. Assessing the child's apical pulse rate, measuring blood pressure, and assessing respiratory rate and depth aren't related to potassium administration.

A child, age 5, is to have potassium added to his I.V. fluid. Before initiating this therapy, the nurse first should: 1. assess the child's apical pulse rate. 2. measure the blood pressure. 3. monitor fluid intake and output. 4. assess respiratory rate and depth.

4. the child's body surface area. RATIONALE: Using a child's body surface area may be the most accurate method for calculating safe drug dosages because body surface area is thought to parallel the child's organ growth and maturation and metabolic rate. Using the child's weight in kilograms, Young's rule based on the child's age, or Clark's rule based on the child's weight in pounds is likely to yield less accurate dosages.

A child, age 5, takes amoxicillin (Amoxil) orally three times per day to treat otitis media. For the most accurate calculation of a safe dosage, the nurse should use: 1. the child's weight in kilograms. 2. Young's rule based on the child's age. 3. Clark's rule based on the child's weight in pounds. 4. the child's body surface area.

2. would have a diagnosis of mild mental retardation. RATIONALE: The nurse should keep in mind that this child would have a diagnosis of mild mental retardation. According to the American Association on Mental Deficiency, a person with an IQ between 50 and 70 is classified as mildly mentally retarded. An IQ above 70 is considered normal. A person with an IQ between 36 and 50 is classified as moderately retarded. One with an IQ below 36 is severely impaired.

A child, age 5, with an intelligence quotient (IQ) of 65 is admitted to the facility for evaluation. When planning care, the nurse should keep in mind that this child: 1. is within the lower range of normal intelligence. 2. would have a diagnosis of mild mental retardation. 3. would have a diagnosis of moderate mental retardation. 4. would have a diagnosis of severe mental retardation.

2. "We will give our child penicillin every day for 5 years." RATIONALE: Parents stating they will give penicillin indicates effective teaching because a child recovering from acute rheumatic fever must receive prophylactic penicillin for at least 5 years. Bed rest isn't indicated once the acute disease phase ends. Rheumatic fever doesn't call for blood pressure monitoring or corticosteroid therapy.

A child, age 6, is about to be discharged after treatment for acute rheumatic fever. Which statement by the parents indicates effective discharge teaching? 1. "We will keep our child in bed for at least a week." 2. "We will give our child penicillin every day for 5 years." 3. "We will measure our child's blood pressure every day." 4. "We will keep giving our child corticosteroids."

1. describing what the child will hear, see, smell, and feel will help the child cope with the procedure. RATIONALE: Children cope with situations better when they can anticipate sensations rather than just trying to comprehend technical explanations. Therefore, describing what the child will hear, see, smell, and feel will help the child cope. Commonly, a child's anxiety increases rather than decreases with each successive procedure. A school-age child can't assimilate every detail. A 6-year-old child can't understand an explanation of bone marrow function; also, such an explanation would be irrelevant.

A child, age 6, is anxious and upset before a scheduled bone marrow aspiration. During client preparation, the nurse should keep in mind that: 1. describing what the child will hear, see, smell, and feel will help the child cope with the procedure. 2. the child's anxiety will decrease with each successive procedure. 3. no small detail about the procedure should go unexplained. 4. explaining bone marrow function will help the child understand the reason for the procedure.

4. "How are you doing in school?" RATIONALE: The nurse should ask about school because a child's poor progress in school may indicate a visual disturbance. Asking whether a person has problems with seeing colors, seeing at night, or glare is more appropriate when assessing vision in an elderly client.

A child, age 6, is brought to the health clinic for a routine checkup. To assess the child's vision, the nurse should ask: 1. "Do you have any problems seeing different colors?" 2. "Do you have trouble seeing at night?" 3. "Do you have problems with glare?" 4. "How are you doing in school?"

2. Administer pain medication as ordered. RATIONALE: A pain rating of 7 out of 10 indicates significant pain. Therefore, the most appropriate action would be to administer pain medication as ordered. The nurse can ask the child what makes the pain better after medication has been given. Providing diversional activities is appropriate only after administration of pain medication. It isn't appropriate to not treat the child's pain.

A child, age 8, complains of leg pain shortly after being admitted with a fractured tibia sustained in a fall. When the nurse assesses his pain, the child states, "My pain is a 7 out of 10." What action by the nurse would be most appropriate? 1. Ask the child what makes the pain better. 2. Administer pain medication as ordered. 3. Provide diversional activities to distract him. 4. The nurse doesn't need to do anything for this pain level.

1. Let the child visit the playroom daily. RATIONALE: School-age children need peer interaction and thrive on peer approval and acceptance. Allowing the child to visit the playroom daily provides a nonthreatening atmosphere for peer interaction and helps the child feel less isolated. Sitting with the child for an hour wouldn't foster the necessary peer interaction. Placing a telephone in the child's room would allow the child to communicate with family and friends, but could reinforce feelings of isolation. Having another child visit would be appropriate only if the child is of the same age-group.

A child, age 8, is immobilized with a hip spica cast. The nurse enters the room and notices the child is withdrawn and avoiding eye contact. The child's mother states, "He's just bored. He's tired of watching television." The nurse should perform which action? 1. Let the child visit the playroom daily. 2. Sit with the child for an hour in the room. 3. Place a telephone in the child's room. 4. Arrange a visit by a cooperative child from the same unit.

4. Lower right abdominal quadrant RATIONALE: The child's symptoms indicate appendicitis. Therefore, the nurse should assess the lower right abdominal quadrant. The nurse would assess the left lower abdominal quadrant to detect descending and sigmoid colon problems; right upper quadrant to detect gallbladder disease; and the left upper quadrant to detect pancreatitis.

A child, age 9, is admitted to the emergency department with abdominal pain. The child's mother states the pain began about 12 hours ago. The nurse notes the child has a temperature of 100.8° F (38.2° C) and nausea. The child vomited once. Which abdominal area would be most appropriate for the nurse to assess? 1. Left lower abdominal quadrant 2. Right upper abdominal quadrant 3. Left upper abdominal quadrant 4. Lower right abdominal quadrant

2. A restless infant with a high-pitched cry who was transferred from intensive care unit (ICU) the previous evening RATIONALE: An infant's restlessness and high-pitched cry can indicate increased intracranial pressure (ICP). Because the infant was transferred from ICU the previous night, assessing him for increased ICP should be a nursing priority. The infant with a myelomeningocele who is scheduled for surgical placement of a ventriculoperitoneal shunt is stable, so assessing him isn't the most urgent nursing priority. Although the nurse must assess a low-grade fever on the third postoperative day, this stable infant isn't the priority at this time. Pumping a ventriculoperitoneal shunt is less urgent than evaluating increased ICP.

A day-shift nurse on the pediatric neurologic unit has just received a report from the previous shift. Which infant should the nurse assess first? 1. An infant with a myelomeningocele who is scheduled for surgical placement of a ventriculoperitoneal shunt at 10 a.m. 2. A restless infant with a high-pitched cry who was transferred from intensive care unit (ICU) the previous evening 3. An infant with an axillary temperature of 100.4 ° F (38° C) on the third postoperative day 4. An infant whose ventriculoperitoneal shunt must be pumped every 2 hours following shunt revision the previous day. The shunt was last pumped at 6 a.m.

2. "I'll spread a thin layer of lotion over pressure points." RATIONALE: Using a lotion on the pressure points will soften the skin and promote its breakdown and therefore, should be avoided. Gently massaging the skin with a lubricating substance is recommended because it will stimulate circulation and help prevent breakdown. Changing the toddler's position frequently will help minimize pressure, prevent edema, and stimulate circulation. Keeping the skin clean will lessen the chances of irritation and breakdown.

A day-shift nurse tells a night-shift nurse that she's been attempting to reduce the risk for Impaired skin integrity related to immobility in a toddler. Which statement by the night-shift nurse should the day-shift nurse question? 1. "I'll gently massage the skin with a lubricating substance." 2. "I'll spread a thin layer of lotion over pressure points." 3. "I'll change the toddler's position frequently." 4. "I'll clean the skin as often as necessary."

3. Show the mother how to hold the infant properly. RATIONALE: Infants with I.V. lines should be held with care. The nurse should encourage and show the mother how to hold the infant properly and teach her about I.V. care measures to enhance her confidence and skill. The nurse should encourage the mother to participate in the child's care whenever possible, not just during I.V. therapy. There's no need for the infant to have to lie quietly in bed.

A dehydrated infant is receiving I.V. therapy. The mother tells the nurse she wants to hold her infant but is afraid this might cause the I.V. line to become dislodged. What should the nurse do? 1. Tell the mother it's best not to move the infant now. 2. Inform the mother that only a nurse should hold the infant during I.V. therapy. 3. Show the mother how to hold the infant properly. 4. Advise the mother to let the infant lie quietly in bed.

1. Dependency, fearfulness, and lack of outside interests RATIONALE: Disabled children whose parents are overprotective tend to have marked dependency, fearfulness, inactivity, and lack of outside interests. Children who are raised by oversolicitous and guilt-ridden parents are often overly independent, defiant, and high-risk takers. Children who are reared by parents who emphasize the child's deficits and tend to isolate the child may appear shy and lonely. Children who are reared by parents who establish reasonable limits have pride and confidence in their ability to cope successfully.

A disabled school-age child whose parents are overprotective may display which characteristics? 1. Dependency, fearfulness, and lack of outside interests 2. Extreme independence, defiance, and a high level of risk taking 3. Shyness and loneliness 4. Pride and confidence in one's ability to cope

3. In 48 to 72 hours RATIONALE: Tuberculin skin tests are tests of delayed hypersensitivity. If the test results are positive, a reaction should appear in 48 to 72 hours. Immediately afterward and within 24 hours of administration are too soon to observe a reaction. Waiting more than 5 days to evaluate the test is too long because any reaction that occurred may no longer be visible.

A high-risk adolescent is given a tuberculin intradermal skin test to detect tuberculosis infection. How long after the test is administered should the results be evaluated? 1. Immediately afterward 2. Within 24 hours 3. In 48 to 72 hours 4. After 5 days

4. this is an appropriate request and arrangements will be made as soon as possible. RATIONALE: The school's request is appropriate because screening for scoliosis should begin at age 8 and be performed yearly thereafter. Also, because screening for scoliosis involves inspection of the spine and use of a scoliometer, both can be done in a school setting.

A local elementary school has requested scoliosis screening for its students from the hospital's community outreach program. The school should be informed that: 1. these students are too young to screen; instead, older students should be screened. 2. these students are too old to screen and will no longer benefit from screening for scoliosis. 3. scoliosis screening requires sophisticated equipment and can't be done in school. 4. this is an appropriate request and arrangements will be made as soon as possible.

1. decreasing respiratory tract edema. RATIONALE: The cool humidity of the mist tent helps the infant breathe by decreasing respiratory tract edema. The confinement of the mist tent can increase anxiety, not avoid it. Also, the tent liquefies secretions, rather than drying them, and it doesn't increase the infant's fluid intake.

A mist tent contains a nebulizer that creates a cool, moist environment for an infant with an upper respiratory tract infection. The cool humidity helps the infant breathe by: 1. decreasing respiratory tract edema. 2. avoiding anxiety. 3. drying secretions. 4. increasing fluid intake.

3. "We've found that babies can't digest solid food properly until they're 3 or 4 months old." RATIONALE: Stating that babies can't digest solid food properly is correct because infants younger than 3 or 4 months lack the enzymes needed to digest complex carbohydrates. Saying that there's no need for solid food doesn't address the grandmother's question directly. Saying that things have changed is a cliché that may block further communication with the grandmother. Stating that introducing solid food early leads to eating disorders is incorrect because no evidence suggests that this occurs.

A mother and grandmother bring a 2-month-old infant to the clinic for a routine checkup. As the nurse weighs the infant, the grandmother asks, "Shouldn't the baby start eating solid food? My kids started on cereal when they were 2 weeks old." Which response by the nurse would be appropriate? 1. "The baby is gaining weight and doing well. There is no need for solid food yet." 2. "Things have changed a lot since your children were born." 3. "We've found that babies can't digest solid food properly until they're 3 or 4 months old." 4. "We've learned that introducing solid food early leads to eating disorders later in life."

3. ask the infant's father to sign consent for emergency treatment of the infant. RATIONALE: The father may give consent for treatment of the infant, but he may not give consent to treat the mother (his former wife). The mother's next of kin should be contacted for consent. Because the father may give consent for the infant to be treated, it isn't necessary to contact the court at this time.

A mother and infant are admitted to the emergency department following a motor vehicle crash. The infant is unresponsive to verbal and tactile stimuli, his pupils are dilated, and a nurse observes lacerations on his head, neck, and upper torso. The infant's mother is experiencing respiratory distress and is being treated in another room in the emergency department. The nurse learns that the parents are divorced and have joint custody of the infant. The father arrives in the emergency department. The nurse should: 1. contact social services to establish contact with the next of kin and obtain consent to treat the mother and infant. 2. ask the infant's father to sign consents for emergency treatment of the mother and infant. 3. ask the infant's father to sign consent for emergency treatment of the infant. 4. contact social services to establish contact with the court to obtain consent to treat the infant.

3. "My child is most likely regressing back to a behavior that increases his sense of security." RATIONALE: The statement about regression indicates understanding because the stress of starting nursery school may trigger a return to a level of successful behavior from earlier stages of development. A child's skills remain intact, although increased stress may prevent the child from using these skills. The child's behavior isn't an indication that he hates school or wants to punish the mother. Regression isn't a trait that can be inherited.

A mother asks the nurse how to handle her 4-year-old child, who recently started wetting the pants after being completely toilet-trained. The child just started attending nursery school 2 days per week. Which statement by the mother indicates understanding of the situation? 1. "My child hates school." 2. "My child is punishing me for sending him away for a few hours." 3. "My child is most likely regressing back to a behavior that increases his sense of security." 4. "He must have inherited this from my husband. My husband did the same thing when he started nursery school."

1. "The baby's eustachian tubes are shorter and lie more horizontally." RATIONALE: Infants and young children are more prone to otitis media because their eustachian tubes are shorter and lie more horizontally. Pathogens from the nasopharynx can more readily enter the eustachian tube of the middle ear. The inability to clear nasal passages by blowing the nose, lying down on the floor, and putting dirty toys in the mouth don't increase the tendency toward otitis media.

A mother asks the nurse why her 12-month-old baby gets otitis media more frequently than her 10-year-old son. What should the nurse tell her? 1. "The baby's eustachian tubes are shorter and lie more horizontally." 2. "The baby is too young to blow his nose when he has a cold." 3. "The baby spends more time lying down than his older brother; therefore, more dirt gets in the baby's ear." 4. "The baby puts dirty toys in his mouth."

3. as the mother feeds the infant. RATIONALE: The nurse can best assess mother-infant interaction during feeding, such as by observing how closely the mother holds the infant and how she looks at the infant's face. These behaviors help reveal the mother's anxiety level and overall feelings for the infant. The infant's posture and response during feeding provide clues to the infant's comfort level and feelings. Sleeping doesn't provide an opportunity for mother-infant interaction. Although playing and rocking may provide clues about mother-infant interaction, they aren't the best activities to assess. During playing, for example, the mother may interact with the infant at a distance whereas rocking promotes closeness but not interaction; the mother can rock the infant while talking to someone else or staring off into the distance.

A mother brings her 2-month-old infant to the clinic for a well-baby checkup. To best assess the interaction between the mother and infant, the nurse should observe them: 1. as the infant plays. 2. as the infant sleeps. 3. as the mother feeds the infant. 4. as the mother rocks the infant.

3. DTaP, IPV, Hib, hepatitis B, and pneumococcal conjugate vaccine (PCV) RATIONALE: DTaP, IPV, Hib, hepatitis B, and PCV are administered at ages 2 and 4 months. The MMR vaccine is typically administered at age 12 to 15 months. Rotavirus vaccine is no longer recommended because of the associated risk of intussusception. The varicella vaccine is commonly administered between ages 12 and 18 months.

A mother brings her 4-month-old infant to the clinic for a wellness checkup. Which immunizations should the infant receive? 1. Diphtheria, tetanus toxoids, and acellular pertussis (DTaP), inactivated polio virus (IPV), rotavirus, and measles-mumps-rubella (MMR) 2. Haemophilus influenzae type B (Hib), rotavirus, DTaP, and IPV 3. DTaP, IPV, Hib, hepatitis B, and pneumococcal conjugate vaccine (PCV) 4. DTaP, hepatitis B, Hib, and varicella

1. "Your baby's behavior indicates stranger anxiety, which is common at his age." RATIONALE: Stranger anxiety, common in infants ages 6 to 8 months, may cause the child to cry, cling to the caregiver, and turn away from strangers. Typically, it occurs when the child starts to differentiate familiar and unfamiliar people. The child's behavior doesn't necessarily indicate shyness. According to Piaget, fear of pain characterizes the operational stage of development in school-age children, not infants. Temper tantrums are typical in toddlers who are trying to assert their independence. During a temper tantrum, children may kick, scream, hold their breath, or throw themselves onto the floor rather than cling to a parent.

A mother brings her 8-month-old son to the pediatrician's office. When the nurse approaches to measure the child's vital signs, he clings to his mother tightly and starts to cry. The mother says, "He used to smile at everyone. I don't know why he's acting this way." How should the nurse respond to the mother's statement? 1. "Your baby's behavior indicates stranger anxiety, which is common at his age." 2. "Children who behave that way are developing shy personalities." 3. "Children at his age begin to fear pain." 4. "Your baby's having a temper tantrum, which is common at his age."

3. 5th to 95th percentile RATIONALE: Height and weight measurements that fall between the 5th and 95th percentiles represent normal growth for most children. Children whose measurements fall outside this range require further evaluation.

A mother brings her child, age 3, to the clinic for an annual checkup. After plotting the child's height and weight on a pediatric growth chart, the nurse identifies which percentile range as normal? 1. 25th to 75th percentile 2. 50th to 100th percentile 3. 5th to 95th percentile 4. 10th to 100th percentile

4. Gastric lavage and administration of activated charcoal RATIONALE: The physician will probably order gastric lavage or activated charcoal administration. Ipecac syrup is no longer recommended and an antacid isn't an effective treatment for poisoning. Infusing normal saline solution I.V. may be helpful in treating dehydration caused by vomiting, but in itself isn't effective in eliminating the poisonous substance.

A mother brings her preschool child to the emergency department after the child ingested an unknown quantity of acetaminophen. Which treatment will the physician probably order? 1. Administration of a dose of ipecac syrup 2. Insertion of a nasogastric tube and administration of an antacid 3. I.V. infusion of normal saline solution 4. Gastric lavage and administration of activated charcoal

4. Diphtheria, tetanus, and acellular pertussis (DTaP), MMR, and inactivated polio virus (IPV) RATIONALE: Between ages 4 and 6, the child should receive DTaP, MMR, and IPV. Hepatitis A isn't a required immunization. MMR alone is incomplete and H. influenzae, type B immunization is completed by age 15 months.

A nurse is assessing whether a child has received all recommended immunizations for his age. Which immunizations should he have received between ages 4 and 6? 1. Hepatitis A 2. Measles, mumps, and rubella (MMR) 3. Haemophilus influenzae, type B 4. Diphtheria, tetanus, and acellular pertussis (DTaP), MMR, and inactivated polio virus (IPV)

3. "Continue your infant's normal feedings." RATIONALE: If an infant has mild diarrhea, his mother should be advised to continue his normal diet and to call back if the diarrhea doesn't stop or if he shows signs of dehydration. There's no need to give the infant clear liquids only. Notifying the day care about the infant's illness is important but doesn't take priority.

A mother calls the clinic to report that her 9-month-old infant has diarrhea. Upon further questioning, the nurse determines that the child has mild diarrhea and no signs of dehydration. Which advice is most appropriate to give this mother? 1. "Call back if your infant has 10 stools in 1 day." 2. "Feed your infant clear liquids only." 3. "Continue your infant's normal feedings." 4. Notify your infant's day care of his illness.

2. Treat the child's symptoms and use diphenhydramine (Benadryl) for itching. RATIONALE: The most likely explanation for the child's illness is chickenpox. The nurse should review the treatment for chickenpox, which includes acetaminophen for fever and fussiness, and oatmeal baths and diphenhydramine for itching. Unless the child is severely ill or has complications, the child doesn't need to be seen in the clinic for diagnosis confirmation. Limiting a preschooler's television viewing is appropriate but isn't the most important advice. Typically, children will limit their own activities as needed. The child will need to stay out of day care until the lesions of the rash are crusted over.

A mother calls the clinic to report that her preschool-age child has had a fever, has been fussy, and now has a rash that started on the neck and has spread to the rest of the child's body. The child was exposed to chickenpox about 3 weeks ago. Which advice is the most important to give the mother? 1. Bring the child in immediately so the diagnosis can be confirmed. 2. Treat the child's symptoms and use diphenhydramine (Benadryl) for itching. 3. Be sure the child stays quiet, and limit the amount of television viewing. 4. After the fever is gone, the child can return to day care.

4. "Acknowledge him by saying, 'That's a pretend story.'" RATIONALE: It's important to acknowledge the child's imagination, while also letting him know in a nice way that what he has said isn't real. Punishment isn't appropriate for a 4-year-old child using his imagination, and accusing him of lying is a negative reinforcement. The child isn't truly lying in the adult sense. Although imagination and creativity need to be acknowledged by the mother, the nurse must respond to the mother's concern with appropriate interventions, not opinion on why the behavior is occurring.

A mother complains to the nurse that her 4-year-old son often "lies." What is the nurse's best response? 1. "Let the child know that he'll be punished for lying." 2. "Ask him why he isn't telling the truth." 3. "It's probably due to his vivid imagination and creativity." 4. "Acknowledge him by saying, 'That's a pretend story.'"

2. "Continue to pick her up when she cries because young infants need cuddling and holding to meet their needs." RATIONALE: The nurse should advise the mother to continue to pick the infant up when she cries because a young infant needs to be cuddled and held when crying. Because the infant's cognitive development isn't advanced enough for her to associate crying with getting attention, it would be difficult to spoil her at this age. Even if the infant's diaper is dry, a gentle touch may be necessary until she falls asleep. Crying for 10 minutes wears an infant out; ignoring crying can make the infant mistrust caregivers and the environment. Infants cry for many reasons, not just when hungry, so the mother shouldn't assume the infant is crying because she's hungry.

A mother is concerned that she might be spoiling her 2-month-old daughter by picking her up each time she cries. Which suggestion should the nurse offer? 1. "If the baby's diaper is dry when she's crying, leave her alone and she'll fall asleep." 2. "Continue to pick her up when she cries because young infants need cuddling and holding to meet their needs." 3. "Leave your baby alone for 10 minutes. If she hasn't stopped crying by then, pick her up." 4. "Crying at this age indicates hunger. Try feeding her when she cries."

4. 10 months RATIONALE: The nurse would estimate that the infant is 10 months old because an infant this age can sit alone and understands object permanence, so he would look for the hidden toy. Between ages 4 and 6 months, children can't sit securely alone. At age 8 months, children can sit securely alone but don't understand the permanence of objects.

A mother is playing with her infant, who's sitting securely alone on the floor of the clinic. The mother hides a toy behind her back and the infant looks for it. What age should the nurse estimate the infant to be? 1. 4 months 2. 6 months 3. 8 months 4. 10 months

3. This behavior is normal in a 2-year-old child. RATIONALE: Toddlers are confronted with the conflict of achieving autonomy yet relinquishing their much-enjoyed dependence on — and affection of — others. Therefore, their negativism is a necessary assertion of self-control and should be considered a normal behavior. Nothing about this behavior indicates that the child is under stress, isn't receiving sufficient affection, or requires counseling.

A mother tells the nurse that her 22-month-old child says no to everything. When scolded, the toddler gets angry and starts crying loudly but then immediately wants to be held. What is the best interpretation of this behavior? 1. The toddler isn't coping with stress effectively. 2. The toddler's need for affection isn't being met. 3. This behavior is normal in a 2-year-old child. 4. This behavior suggests the need for counseling.

4. Notify hospital security or the local authorities. RATIONALE: The Protection from Abuse order legally prohibits the father from seeing the child. In this situation, the nurse should notify hospital security or the local authorities of this attempt to breach the order, and allow them to escort the father out of the building. The father could be jailed or fined if he violates the order. The nurse shouldn't argue or continue explaining to the father that he must leave because it could place her and the child at risk if the father becomes angry or agitated. The nursing coordinator and nurse-manager should be notified of the incident; the nurse's first priority, however, should be contacting security or the authorities.

A mother of a 5-year-old child who was admitted to the hospital has a Protection from Abuse order for the child against his father. A copy of the order is kept on the pediatric medical surgical unit where the child is being treated. The order prohibits the father from having any contact with the child. One night, the father approaches the nurse at the nurses' station, politely but insistently demanding to see his child, and refusing to leave until he does so. What should the nurse do first? 1. Firmly tell the father he must leave. 2. Notify the nursing coordinator on duty. 3. Notify the nurse-manager. 4. Notify hospital security or the local authorities.

2. "Tell me more about how you feel." RATIONALE: Many parents feel guilty when their child is sick. Therefore, it's most appropriate to encourage parents to talk more about their feelings because doing so provides support and helps to develop a therapeutic relationship. Giving a stereotyped answer, such as "Don't worry," shows a lack of interest in what the parent is feeling. Commenting on the course of the disease doesn't address the parent's feelings. Being judgmental or offering an opinion can also block therapeutic communication by inhibiting the parent from discussing her feelings and developing solutions.

A mother of a child with sickle cell anemia confides in the nurse that she feels guilty about letting the child run and play with the neighborhood children and that if she had been a better mother, the child wouldn't have suffered a sickle cell crisis. Which response would be most appropriate? 1. "She's just fine now. Don't worry." 2. "Tell me more about how you feel." 3. "But you know that children with sickle cell anemia often have crises." 4. "You shouldn't be so protective of her."

1. "It's common for a child to exhibit regressive behavior when anxious or stressed." RATIONALE: The nurse should tell the mother that young children commonly demonstrate regressive behavior when anxious, under stress, or in a strange environment. Although the child could be deliberately wetting the bed out of anger, her behavior most likely isn't under voluntary control. It's appropriate to expect a 3-year-old child to be toilet-trained, but it isn't appropriate to expect the child to be able to use a call button to summon the nurse.

A mother of a hospitalized 3-year-old girl expresses concern because her daughter is wetting the bed. What should the nurse tell her? 1. "It's common for a child to exhibit regressive behavior when anxious or stressed." 2. "Your child is probably angry about being hospitalized. This is her way of acting out." 3. "Don't worry. It's common for a 3-year-old child to not be fully toilet-trained." 4. "The nurses probably haven't been answering the call button soon enough. They will try to respond more quickly."

3. Measure the child's blood glucose level. RATIONALE: In a child with type 1 diabetes, behavioral changes may signal either hypoglycemia or hyperglycemia. Measuring the blood glucose level is the only way to determine which condition is present and, therefore, should be the mother's first action. Urine glucose measurement doesn't accurately reflect the current blood glucose level. Forcing a lethargic child to drink fluids could cause aspiration. After measuring the child's blood glucose level, the mother may need to take additional emergency measures such as administering insulin or a simple glucose source. If the child doesn't respond to these measures, she may need to call for emergency help.

A mother of a preschooler recently diagnosed with type 1 diabetes makes an urgent call to the pediatrician's office. She says her child had an uncontrollable temper tantrum while playing and now is lethargic and hard to rouse. The nurse should instruct the mother to take which action first? 1. Obtain a urine sample and measure the glucose level. 2. Force the child to drink orange juice. 3. Measure the child's blood glucose level. 4. Call 911 because this situation is an emergency.

2. "I should have my child soak in oatmeal baths twice daily." RATIONALE: Chickenpox is characterized by pruritic lesions; colloidal oatmeal baths may soothe the skin and relieve itching. Therefore, the mother demonstrates effective teaching by saying she'll soak her child in oatmeal baths. Although a fever is common during the first 24 hours the communicable period extends beyond the febrile stage and a normal temperature shouldn't be used as the basis for letting the child leave home. Chickenpox is communicable from 1 day before the lesions erupt until they dry — approximately 1 week. The child should stay home during this time to prevent disease transmission. Aspirin isn't recommended because it's associated with Reye's syndrome; acetaminophen is a suitable substitute. The incubation period for chickenpox is 2 to 3 weeks; the mother should begin to check the other children for lesions 2 weeks after exposure to the infected child.

A mother of several young children calls the nurse when her school-age child comes down with chickenpox. The nurse provides instruction on communicability and home management of this disease. Which response by the mother indicates effective teaching? 1. "I should keep my child at home until the fever is gone." 2. "I should have my child soak in oatmeal baths twice daily." 3. "I should give my child aspirin every 4 hours until the fever is gone." 4. "I should start checking my other children for lesions in about 4 weeks."

1. Obtain more information from the mother and the child. RATIONALE: In this situation, the nurse needs more information before proceeding and should question the mother and child about the problems. Referring the child to the school psychologist and talking to the child's health care provider and teacher are all important components of a treatment plan, but obtaining more information comes first.

A mother reports that her school-age child is having some problems in school. Which action would be the priority? 1. Obtain more information from the mother and the child. 2. Refer the child to the school psychologist for testing. 3. Talk to the child's health care provider to understand the child better. 4. Talk to the child's teacher to gain a perspective on the situation.

1. Allow the child to feed herself. RATIONALE: The best recommendation is to allow the child to feed herself because the child's stage of development is the preschool period of initiative. Special dishes would enhance the primary recommendation but wouldn't be an effective approach on their own. It's important to offer new foods and choices, not just serve her favorite foods. Using a small table and chair would also enhance the primary recommendation of allowing the child to feed herself.

A mother tells the nurse that her 4-year-old child is a very poor eater. What is the nurse's best recommendation for helping the mother increase her child's nutritional intake? 1. Allow the child to feed herself. 2. Use specially designed dishes for children — for example, a plate with the child's favorite cartoon character. 3. Only serve the child's favorite foods. 4. Allow the child to eat at a small table and chair by herself.

1. developmental readiness of the child. RATIONALE: The most important factor is developmental readiness because if the child isn't developmentally ready, both the child and parent will become frustrated. Consistency is important when toilet training is started; the mother's positive attitude is important when the child is determined to be ready. Developmental levels of children are individualized and comparison to peers isn't useful.

A mother tells the nurse that she wants to begin toilet training her 22-month-old child. The most important factor regarding toilet training that the nurse should stress to her is: 1. developmental readiness of the child. 2. consistency in approach. 3. the mother's positive attitude. 4. developmental level of the child's peers.

2. take the infant and mother back to a treatment room. RATIONALE: Taking the infant and mother into a treatment room for assessment is appropriate because this action provides privacy and a controlled environment. Taking the infant away from the mother is inappropriate because the mother should be allowed to remain with her child if she wishes. If she doesn't want to be present, the nurse should find a private area for her. The nurse must assess the child before calling the resuscitation team. Security isn't warranted in this situation.

A mother, who is visibly upset, carries her 2-month-old infant into the crowded emergency department. The child appears limp and lifeless. The mother screams to the nurse for help. The nurse's first action should be: 1. take the infant from the mother and offer to help. 2. take the infant and mother back to a treatment room. 3. call the resuscitation team and the supervisor. 4. call security and the hospital administration.

RATIONALE: Moving a client into a wheelchair is within the scope of practice of the nursing assistant. Only licensed personnel are authorized to administer medications. A registered nurse should personally assess the client's surgical wound so she can monitor for adverse changes. Also, the registered nurse should provide adequate client education about a newly diagnosed disease to ensure complete compliance; the nursing assistant may not have the knowledge to do so.

A nurse and a nursing assistant are caring for a group of adolescents. Which task could the nurse safely delegate to the nursing assistant? 1. Helping a girl into a wheelchair 2. Administering acetaminophen (Tylenol) for a fever 3. Assisting a physician during the first postoperative dressing change 4. Reviewing discharge instructions for an adolescent recently diagnosed with diabetes

1. "Heart rate regular, grade I murmur auscultated." RATIONALE: A heart rate of 80 beats/minute is considered normal for an 8-year-old child. In this age-group, bradycardia is typically associated with a heart rate of less than 70 beats/minute. A grade I murmur is barely audible in a quiet room; a grade II murmur is faint but clearly audible.

A nurse assessing the heart rate and rhythm of an 8-year-old child hears a murmur that's barely audible even in a quiet room. The child's heart rate is 80 beats/minute. The nurse should document her assessment findings as: 1. "Heart rate regular, grade I murmur auscultated." 2. "Heart rate bradycardic, grade I murmur auscultated." 3. "Heart rate regular, grade II murmur auscultated" 4. "Heart rate bradycardic, grade II murmur auscultated."

2. Call the physician and ask for a verbal order to clarify the dosage. RATIONALE: Clarification of written orders must come from the physician or health care provider who wrote the order. A verbal order should be obtained and then entered into the medical chart on a separate line. Assuming or guessing what the writer intended could lead to a medication error. Medical charts are legal documents; information should never be altered or erased. The nurse shouldn't ask the mother because the mother may not be reliable and the physician may have ordered a different dose during hospitalization.

A nurse caring for an 8-month-old infant diagnosed with respiratory syncytial virus is unable to read a medication dosage written in the infant's medical record. What is the only ethical and responsible solution for the nurse? 1. Erase the original order and rewrite it more clearly. 2. Call the physician and ask for a verbal order to clarify the dosage. 3. Ask another nurse what she thinks the dosage should be. 4. Ask the mother what dosage the infant takes at home.

1. assess pin sites every shift and as needed. RATIONALE: Nursing care for a client in traction includes assessing pin sites every shift and as needed and ensuring that the knots in the rope don't catch on the pulley. The nurse should add and remove weights at the physician's order, not at the adolescent's request. All joints, except those immediately proximal and distal to the fracture, should be put through range of motion every shift.

A nurse caring for an adolescent in traction should: 1. assess pin sites every shift and as needed. 2. ensure that the rope knots catch on the pulley. 3. add and remove weights at the adolescent's request. 4. put all his joints through range of motion every shift.

1. call for help, open the airway, provide two rescue breaths, and begin compressions at a rate of 100 per minute. RATIONALE: The nurse should call for help, open the airway, provide two rescue breaths, begin compressions at a rate of 100 per minute, give two breaths for every 30 compressions, continue for approximately 2 minutes, and reassess. This is the accepted sequence defined by the American Heart Association for one-rescuer child cardiopulmonary resuscitation (CPR). Calling for help should be the first action to ensure that assistance arrives quickly. The accepted sequence of events for one-rescuer adult CPR is to call for help, open the airway, provide two rescue breaths, begin compressions at a rate of 100 per minute, give two breaths for every 30 compressions, continue for approximately 2 minutes, and reassess. As soon as unresponsiveness, breathlessness, or lack of pulse has been established, CPR should begin immediately.

A nurse discovers a 5-year-old child who's unresponsive, apneic, and pulseless. The correct sequence of events that should follow is: 1. call for help, open the airway, provide two rescue breaths, and begin compressions at a rate of 100 per minute. 2. open the airway, provide two rescue breaths, and begin compressions at a rate of 100 per minute. 3. call for help, open the airway, provide two rescue breaths, and begin compressions at a rate of 80 per minute. 4. call for help, continue to attempt to arouse, and assess for breathlessness and lack of pulse until a second rescuer arrives.

1. accidents are the leading cause of death among toddlers. RATIONALE: The leading cause of death in toddlers is accidents, so it's important for parents, family members, and childcare providers to understand the importance of accident prevention. Toddlers don't have the highest risk for homicide. Toddlers are just beginning to understand right from wrong, but don't understand the consequences of their actions. Although many children will chase balls or toys into the street, not all children will do so.

A nurse discussing injury prevention with a group of workers at a day-care center is focusing on toddlers. When discussing this age-group, the nurse should stress that: 1. accidents are the leading cause of death among toddlers. 2. the risk for homicide is highest among toddlers. 3. toddlers can distinguish right from wrong. 4. toddlers will always chase a ball that rolls into the street.

3. 8 months RATIONALE: Most infants can sit up without support by age 8 months. At age 4 months, the infant can lift the head off the mattress up to a 90-degree angle. Between ages 6 and 7 months, the infant can sit while leaning forward on the hands. At age 10 months, the infant typically can move from a prone to a sitting position and pull himself up to a standing position.

A nurse expects an infant to sit up without support at which age? 1. 4 months 2. 6 months 3. 8 months 4. 10 months

1. gather supporting evidence and contact the physician with her concerns. RATIONALE: If a nurse suspects a diagnosis, she must evaluate the situation further and collect more data. Then she should present her findings to the physician. It isn't appropriate for the nurse to wait 24 hours before addressing the possible diabetes. It would be premature for the nurse to contact the dietitian about a diabetic diet, and a nurse doesn't have authority to order a random glucose test.

A nurse feels that a 5-year-old boy in her care is showing signs and symptoms of diabetes mellitus. The nurse should: 1. gather supporting evidence and contact the physician with her concerns. 2. ask the dietitian to talk with the child and his parents about a diabetic diet. 3. ask the laboratory to perform a random glucose test. 4. monitor the child's activity for 24 hours.

1. Muscular hypotonicity RATIONALE: Several conditions make the child with Down syndrome highly vulnerable to respiratory infections. For example, the hypotonicity of chest muscles in children with Down syndrome leads to diminished respiratory expansion and pooling of secretions, and an underdeveloped nasal bone impairs mucus drainage. Down syndrome isn't associated with muscle spasticity or increased mucus viscosity. Although hypothyroidism is common in children with Down syndrome, it doesn't increase the risk of infection.

A nurse formulates a nursing diagnosis of Risk for infection for a child with Down syndrome. Which condition typically seen in children with this syndrome supports this nursing diagnosis? 1. Muscular hypotonicity 2. Muscle spasticity 3. Increased mucus viscosity 4. Hypothyroidism

4. A 6-year-old child with acute heart failure on 2 L of oxygen RATIONALE: Following the ABCs (airway, breathing, and circulation), the nurse should assess the child on oxygen first to make sure the child has the oxygen in place and the pulse oximeter reading is above 94%. The other children should be assessed as soon as possible, but the child on oxygen takes priority.

A nurse has just received a report from the nurse who worked the previous shift. Which child should she assess first? 1. A 5-year-old child who needs factor VIII before a tonsillectomy 2. A 4-year-old child admitted with reactive airway disease receiving proventil (Albuterol) every 4 hours 3. A 3-year-old child who had an appendectomy and is complaining of pain 4. A 6-year-old child with acute heart failure on 2 L of oxygen

2. Trust versus mistrust. 5. Autonomy versus shame and doubt. 1. Initiative versus guilt. 3. Industry versus inferiority. 4. Identity versus role confusion. RATIONALE: During the first stage of Erikson's five stages of psychosocial development, trust versus mistrust (birth to age 1), the child develops trust as the primary caregiver meets his needs. In the second stage, autonomy versus shame and doubt (ages 1 to 3), the child gains control of body functions and becomes increasingly independent. In the third stage, initiative versus guilt (ages 3 to 6), the child develops a conscience and learns about the world through play. In the fourth stage, industry versus inferiority (ages 6 to 12), the child enjoys working on projects with others, follows rules, and forms social relationships. As body changes begin to take place, the child enters the fifth stage, identity versus role confusion (ages 12 to 19), and becomes preoccupied with looks, how others view him, meeting peer expectation, and establishing his own identity.

A nurse has received report on her clients and notices that they're of varying ages. To prepare for the shift, the nurse reviews Erik Erikson's five stages of psychosocial development. Place the stages in chronological order from infancy to adolescence. Use all options. 1. Initiative versus guilt. 2. Trust versus mistrust. 3. Industry versus inferiority. 4. Identity versus role confusion. 5. Autonomy versus shame and doubt.

4. "Saying 'no' is part of toddler development and is normal at this age." RATIONALE: Telling the mother that saying "no" is normal for a 15-month-old child is an appropriate response. The child's behavior doesn't mean that the child doesn't love the mother. It means the child is attempting to exert independence. Punishing the child isn't appropriate because this is a normal stage of development. Saying that it's best to ignore the behavior is also inappropriate because the child needs to learn about limits. Explaining to the child that his behavior is inappropriate isn't an age-appropriate response for this child.

A nurse in a clinic finds the mother of a 15-month-old child in tears. The mother states that her child doesn't love her because the child says "no" to everything. Which response is appropriate? 1. "Have you punished your child for saying 'no' to you?" 2. "This is normal at this age; it's best to ignore the behavior." 3. "Explain to your child that saying 'no' all of the time is inappropriate behavior." 4. "Saying 'no' is part of toddler development and is normal at this age."

4. "You seem upset. Tell me about it." RATIONALE: Asking the client to talk about her feelings is appropriate because by verbalizing the nurse acknowledges the client's feelings. By listening, the nurse acknowledges the client's feelings and can help the client understand them and begin to deal with them. Telling the client that there are babies with worse defects doesn't acknowledge — and may even belittle — her feelings. Providing a stock answer, such as "Don't worry," shows a lack of interest in the client's feelings. Offering to ask the physician also doesn't address the client's feelings.

A nurse is approached by the mother of a child with hypospadias. She says to the nurse, "Why did this have to happen to my baby? Why couldn't he be perfect? How could this have happened?" What should the nurse say in response? 1. "This is only a minor problem. Many other babies are born with worse defects." 2. "Don't worry. After surgical repair you'll hardly remember there was anything wrong with your baby." 3. "I'll ask the physician to explain to you how this defect occurs." 4. "You seem upset. Tell me about it."

1. "The changes in your daughter's breasts are the first signs of puberty." RATIONALE: Stating that such changes are the first signs of puberty is correct because breast bud development — elevation of the nipple and areola to form a breast bud — is the first sign of sexual maturity in girls. It's a normal finding in a girl this age and doesn't require physician assessment. Telling the mother that nothing is wrong doesn't give the mother concrete information to help alleviate her concern. The change isn't a result of weight gain. Sexual maturation continues with the appearance of pubic hair, axillary hair, and menarche, consecutively.

A nurse is assessing a 10-year-old girl. The girl's mother informs the nurse that she's concerned about her daughter's breasts. The nurse assesses the breasts and notes the areola and nipple protrude slightly. Which statement by the nurse is an appropriate response? 1. "The changes in your daughter's breasts are the first signs of puberty." 2. "This is abnormal and should be assessed by her physician." 3. "I see nothing wrong with her breasts." 4. "The change is a result of increased adipose tissue. Has your daughter gained weight recently?"

3. Aminoglycosides RATIONALE: Aminoglycosides have a high risk of ototoxicity, which is indicated by hearing loss and tinnitus. Cephalosporins, penicillins, and carbapenems aren't associated with ototoxicity.

A nurse is assessing a child who recently received an antibiotic for an ear infection. The mother states that her child seems to have a harder time hearing than before and that the child told her that he hears ringing in his ears. The nurse suspects the child is taking an antibiotic from which class? 1. Cephalosporins 2. Penicillins 3. Aminoglycosides 4. Carbapenems

4. an altered level of consciousness. RATIONALE: One sign of neurologic deficit in an 8-month-old child includes a decreased or altered level of consciousness. The fontanel would bulge — not depress — if he had increased ICP. Slurred speech isn't a sign of increased ICP in an infant because the child isn't able to speak at this age. However, a change in cry may be noted. Bradycardia — not tachycardia — is a sign of increased ICP.

A nurse is assessing an 8-month-old child for signs of neurologic deficit and increased intracranial pressure (ICP). These signs include: 1. a depressed fontanel. 2. slurred speech. 3. tachycardia. 4. an altered level of consciousness.

3. Fear related to altered body image RATIONALE: Fear related to altered body image is the most appropriate nursing diagnosis for a hospitalized adolescent because of the adolescent's developmental level and concern for physical appearance. An adolescent may fear disfigurement resulting from procedures and treatments. Separation is rarely a major stressor for the adolescent, eliminating a diagnosis of Anxiety related to separation from parents. Adolescents may have Fear related to the unknown, but they typically ask questions if they want information. A diagnosis of Ineffective coping related to activity restrictions may be appropriate for a toddler who has difficulty tolerating activity restrictions but is an unlikely nursing diagnosis for an adolescent.

A nurse is assigned to an adolescent. Which nursing diagnosis is most appropriate for a hospitalized adolescent? 1. Anxiety related to separation from parents 2. Fear related to the unknown 3. Fear related to altered body image 4. Ineffective coping related to activity restrictions

2. Softer than the heart sounds RATIONALE: A grade 1 heart murmur is commonly difficult to hear and softer than heart sounds. A grade 2 murmur is usually equal in sound to the heart sounds. A grade 4 murmur is associated with a precordial thrill (a palpable manifestation associated with a loud murmur). A grade 6 murmur can be heard without a stethoscope.

A nurse is auscultating for heart sounds in a 2-year-old child. She notes a grade 1 heart murmur. Which characteristic best describes a grade 1 heart murmur? 1. Equal in loudness to the heart sounds 2. Softer than the heart sounds 3. Can be heard without a stethoscope 4. Associated with a precordial thrill

4. Encourage the parents to allow their child to follow as normal a childhood as possible. RATIONALE: The nurse should encourage the parents to treat their child as much like a normal child as possible. The nurse should avoid being critical of how parents handle their child's condition. Children with cystic fibrosis can live productive lives well into adulthood, so telling the parents they're lucky their child has lived this long not only is rude, it's inappropriate. Although each child the couple has has a 25% chance of having cystic fibrosis, it isn't appropriate for the nurse to counsel the parents. If they express uncertainty about having more children, the nurse should refer them to their physician or a genetic counselor.

A nurse is caring for a 10-year-old child with cystic fibrosis. The child's parents tell the nurse that they're having difficulty coping with their child's disease. Which action should the nurse take? 1. Tell the parents they should be glad their child has lived this long. 2. Point out to the parents ways in which they might have done things differently. 3. Counsel the parents on not having any more children because they could also have cystic fibrosis. 4. Encourage the parents to allow their child to follow as normal a childhood as possible.

1. Obtain consent from the foster parents. RATIONALE: Foster parents have the right to consent to medical care of minors in their care. The parents of a minor in foster care don't have authority to make decisions regarding his care. The nurse should call Child Protective Services only if she has concerns about a foster parent's authenticity. The nurse needn't notify the director of nursing unless complications occur.

A nurse is caring for a 14-month-old infant being treated for an upper respiratory infection. The physician would like to order a series of X-rays for the infant, who has been in a foster home for 4 months. How should the nurse obtain consent? 1. Obtain consent from the foster parents. 2. Call Child Protective Services. 3. Contact the child's biological mother. 4. Contact the unit's director of nursing.

1. Breast development is delayed. 2. The client is at risk for developing diabetes. 4. Normal sexual relationships can be expected. RATIONALE: Cystic fibrosis delays growth and the onset of puberty. Children with cystic fibrosis tend to be smaller than average size and develop secondary sex characteristics later in life. In addition, clients with cystic fibrosis are at risk for developing diabetes mellitus because the pancreatic duct becomes obstructed as pancreatic tissues are destroyed. Clients with cystic fibrosis can expect to have normal sexual relationships, but fertility becomes difficult because thick secretions obstruct the cervix and block sperm entry. Males and females carry the gene for cystic fibrosis. Pulmonary disease commonly progresses as the client ages, requiring additional respiratory treatment — not less.

A nurse is caring for a 17-year-old girl with cystic fibrosis who has been admitted to the hospital to receive antibiotics and respiratory treatment for exacerbation of a lung infection. The girl has a number of questions about her future and the consequences of the disease. Which statements about the course of cystic fibrosis are true? Select all that apply. 1. Breast development is delayed. 2. The client is at risk for developing diabetes. 3. Pregnancy and child-bearing aren't affected. 4. Normal sexual relationships can be expected. 5. Only males carry the gene for the disease. 6. By age 20, the client should be able to decrease the frequency of respiratory treatment.

3. request that a dietitian talk with the parent about infants and nutrition. RATIONALE: The infant's mother needs assistance in maintaining her child's diet. Requesting that a dietitian speak with the mother about the child's diet is within the nurse's scope of practice. The nurse shouldn't call the local police or the social worker on duty because there is no evidence of child abuse or neglect. Many infants are picky eaters and choose not to eat or drink. The nurse doesn't need to call the physician to have the infant put in isolation. Isolation isn't indicated for dehydration.

A nurse is caring for a 19-month-old infant with dehydration and weight loss. The infant's mother states that her son doesn't like to eat and that she hates to make him eat. The nurse should: 1. contact the social worker on duty and give her information about the situation. 2. contact the physician to have the child put in isolation. 3. request that a dietitian talk with the parent about infants and nutrition. 4. contact the local police department to report suspected child abuse.

2. Physical therapist RATIONALE: After the final cast has been removed, foot and ankle exercises may be necessary to improve range of motion. A physical therapist should work with the child. A physical therapist is trained to help clients restore function and mobility, which will prevent further disability. An occupational therapist, who helps the chronically ill or disabled to perform activities of daily living and adapt to limitations, isn't necessary at this time. A recreational therapist, who uses games and group activities to redirect maladaptive energy into appropriate behavior, also isn't required. The nurse hasn't been trained to design an exercise regimen for a child with congenital clubfoot.

A nurse is caring for an infant with congenital clubfoot. After the final cast has been removed, which member of the health care team will most likely help the infant with leg and ankle exercises and provide his parents with a home exercise regimen? 1. Occupational therapist 2. Physical therapist 3. Recreational therapist 4. Nurse

2. Consult with a play therapist about activities in which the child can participate. RATIONALE: Play is an important part of a child's growth and development. A nurse should facilitate play even when a child has a chronic illness. Consulting a play therapist is one way of facilitating such play. Although it's important for children to get adequate sleep, it isn't necessary for a toddler to get 12 hours' sleep per night. A child with a chronic illness may need to be temporarily isolated, but he should still have interaction with family members. A diet high in carbohydrates and low in fat isn't indicated for every toddler with a chronic illness.

A nurse is caring for a 2-year-old child admitted for long-term treatment of a chronic illness. Which action should the nurse take to promote normal childhood growth and development? 1. Allow the child to sleep for at least 12 hours per night. 2. Consult with a play therapist about activities in which the child can participate. 3. Make sure the child is continuously isolated because of his chronic illness and risk of infection. 4. Maintain a diet high in carbohydrates and low in fats.

2. Pulmonic stenosis, intraventricular septal defect, overriding aorta, and right ventricular hypertrophy RATIONALE: TOF consists of four congenital anomalies: pulmonic stenosis, intraventricular septal defect, overriding aorta, and right ventricular hypertrophy. The other combinations of defects aren't characteristic of TOF.

A nurse is caring for a 2½-year-old child with tetralogy of Fallot (TOF). Which abnormalities are associated with TOF? 1. Aortic stenosis, atrial septal defect, overriding aorta, and left ventricular hypertrophy 2. Pulmonic stenosis, intraventricular septal defect, overriding aorta, and right ventricular hypertrophy 3. Pulmonic stenosis, patent ductus arteriosus, overriding aorta, and right ventricular hypertrophy 4. Transposition of the great vessels, intraventricular septal defect, right ventricular hypertrophy, and patent ductus arteriosus

2. Fever 3. Nuchal rigidity 5. Irritability 6. Photophobia RATIONALE: Common signs and symptoms of viral meningitis include fever, nuchal rigidity, irritability, and photophobia. A bulging anterior fontanel is a sign of hydrocephalus, which isn't likely to occur in a toddler because the anterior fontanel typically closes by age 24 months. A petechial, purpuric rash may be seen with bacterial meningitis.

A nurse is caring for a 3-year-old child with viral meningitis. Which signs and symptoms does the nurse expect to find during the initial assessment? Select all that apply. 1. Bulging anterior fontanel 2. Fever 3. Nuchal rigidity 4. Petechiae 5. Irritability 6. Photophobia

1. inform the physician that he is legally responsible for obtaining informed consent. RATIONALE: Obtaining informed consent is the physician's responsibility. A nurse should never perform a procedure without informed consent. If a procedure is performed without this signed document, the nurse, physician, and facility could face legal consequences.

A nurse is caring for a 4-year-old boy who needs a blood transfusion. The physician tells the nurse that the boy's parents must give informed consent. The nurse should: 1. inform the physician that he is legally responsible for obtaining informed consent. 2. recognize that the physician is busy and obtain the consent. 3. perform the procedure without a signed consent form. 4. simply explain the procedure to the child and his parents before performing it.

1. Talk with the parents about the dying process and make them aware of what their child has confided. RATIONALE: Chronically ill children commonly recognize their fate, whereas their parents continue to believe they'll become well again. The nurse should talk with the parents about the child's concerns. It's possible that the parents don't know what their child is feeling. Chronically ill children tend to have a good understanding of death, and should have input into decisions about their care. The nurse shouldn't tell the child that she can change the parents' minds; she might not be able to keep that promise. It would be unethical for the nurse to call the physician and misrepresent the parents' wishes.

A nurse is caring for a 5-year-old boy with end-stage acquired immunodeficiency syndrome (AIDS). The child confides that he is ready to go to heaven and see his grandpa. The nurse knows that the child's parents aren't comfortable with the idea of discontinuing treatment. What should she do? 1. Talk with the parents about the dying process and make them aware of what their child has confided. 2. Listen to the child but recognize that he's too young to make his own decisions. 3. Tell the child that she will talk with his parents and change their minds. 4. Tell the physician that the family would like to discontinue treatment.

1. The parents may be at different stages in dealing with the child's death. 3. The dying child may become clingy and act like a toddler. 5. The death of a child may have long-term disruptive effects on the family. 6. The child doesn't fully understand the concept of death. RATIONALE: When dealing with a dying child, parents may be at different stages of grief at different times. The child may regress in his behaviors. The stress of a child's death commonly results in parents' divorce and behavioral problems in siblings. Preschoolers see death as temporary — a type of sleep or separation. They recognize the word "dead" but don't fully understand its meaning. Thinking about the future is typical of an adolescent facing death, not a preschooler. Whispering in front of the child would likely increase his fear of death.

A nurse is caring for a 5-year-old child who's in the terminal stages of cancer. Which statements are true? Select all that apply. 1. The parents may be at different stages in dealing with the child's death. 2. The child is thinking about the future and knows he may not be able to participate. 3. The dying child may become clingy and act like a toddler. 4. Whispering in the child's room will help the child to cope. 5. The death of a child may have long-term disruptive effects on the family. 6. The child doesn't fully understand the concept of death.

2. Transplant coordinator RATIONALE: The transplant coordinator is the best health team member to approach the family about organ donation. The transplant coordinator is typically available to hospitals that routinely perform organ transplants. When the coordinator isn't available, the attending physician or another physician not directly involved in determining brain death should approach the family. Although the emergency department nurse may have admitted the child, she and the nurse-manager aren't directly involved with the child's care or with the family. Pastoral care staff members provide emotional and religious support and aren't involved with approaching the family about organ donation; they may, however, be present in a supportive capacity if the family wishes.

A nurse is caring for a 9-year-old child who has a grave prognosis after receiving a closed injury from being struck by a car. Which health team member should approach the family about organ donation? 1. Nurse-manager 2. Transplant coordinator 3. Emergency department nurse 4. Pastoral care staff member

1. it's normal for their child to want to sleep with them at night. RATIONALE: It's normal for children involved in traumatic events to experience regression in growth and development or ability to perform physical tasks. For example, a child who has been in an accident may wish to sleep with his parents. Children recovering from traumatic events should have a routine for school, play, meals, and sleep. The parents shouldn't let the child watch television or other media programs about the accident. Children are very resilient; there's no reason to assume this child needs immediate psychiatric counseling.

A nurse is caring for a child who was involved in a bus accident on his way home from preschool. Several people were killed in the accident. When talking with the child's parents about normal reactions to a traumatic event, the nurse should tell them that: 1. it's normal for their child to want to sleep with them at night. 2. they should allow their child to eat and sleep when he wants. 3. they should allow their child to watch television programs about the accident. 4. they should immediately seek psychiatric care for their child.

4. The child is responding to stress. RATIONALE: This behavior indicates the child is responding to stress. Regression (reverting back to previously outgrown behaviors) is a common response to stressful situations. The nurse should reassure the parents that thumb sucking and other regressive behaviors should disappear after the stressful situation is resolved. Thumb sucking isn't a sign of depression or pain or an attention-seeking behavior.

A nurse is caring for a child with tetralogy of Fallot. The child's mother becomes concerned when she visits her son and notices him sucking his thumb, a behavior that he had previously given up. What does this behavior indicate? 1. The child is depressed. 2. The child is in pain. 3. The child wants attention. 4. The child is responding to stress.

1. help the family prepare for the infant's imminent death. RATIONALE: Anencephaly is incompatible with life. The nurse should support family members as they prepare for the infant's imminent death. Facilitating the attachment process, helping the family to adjust to the infant's problems, and preparing the family for extensive surgical procedures are inappropriate because the infant can't survive.

A nurse is caring for a family whose infant has anencephaly. The most appropriate nursing intervention is to: 1. help the family prepare for the infant's imminent death. 2. implement measures to facilitate the attachment process. 3. provide emotional support so the family can adjust to the birth of an infant with health problems. 4. prepare the family for the extensive surgical procedures the infant will require.

1. include the child in the teaching process. RATIONALE: The nurse should include preschoolers in any discharge teaching she performs. Preschoolers have developed reasoning skills and are beginning to understand the concepts of right and wrong and cause and effect. It isn't necessary for both parents to be present during teaching, although it's desirable.

A nurse is caring for a preschooler who sustained deep partial-thickness burns on his hands as a result of touching a hot pot on the stove. When performing discharge teaching, the nurse should: 1. include the child in the teaching process. 2. go into the hallway with the parent to do the teaching. 3. be sure that the child has learned a lesson and won't repeat the action. 4. delay the teaching until both parents are present.

3. Occupational therapist RATIONALE: An occupational therapist helps physically disabled clients adapt to physical limitations and is most qualified to help a child with cerebral palsy eat and perform other activities of daily living. A registered dietitian manages and plans for the nutritional needs of children with cerebral palsy but isn't trained in modifying or fitting utensils with assistive devices. A physical therapist is trained to help a child with cerebral palsy gain function and prevent further disability but not to assist the child in performing activities of daily living. A nursing assistant can help a child eat; however, the nursing assistant isn't trained in modifying utensils.

A nurse is caring for a school-age child with cerebral palsy. The child has difficulty eating using regular utensils and requires a lot of assistance. Which referral is most appropriate? 1. Registered dietitian 2. Physical therapist 3. Occupational therapist 4. Nursing assistant

2. generalized urticaria. RATIONALE: The nurse should instruct parents to immediately report generalized urticaria because it can herald the onset of a life-threatening episode. A child may experience some pain, redness at the sight, localized swelling, or mild temperature elevation; however, these reactions can be treated symptomatically and aren't life-threatening.

A nurse is caring for a toddler who has just been immunized. When teaching the child's parents about potential adverse effects, the nurse should instruct the parents to immediately report: 1. pain at the injection site. 2. generalized urticaria. 3. mild temperature elevation. 4. local swelling at the injection site.

2. The nurse can act as a liaison between the child, the child's parents, and the health care team. RATIONALE: It is important to involve the nurse because she can act as a liaison between all parties. The nurse has the most direct contact with the child and his parents, and she can listen to and communicate their wishes for treatment. She can also aid in interpreting information about the child's condition and course of treatment, helping the parents to make an informed decision. The nurse isn't viewed as the authority on ethical issues at the hospital. In fact, hospitals commonly employ ethicists to help with ethical dilemmas. Time shouldn't be a factor when it comes to helping parents make decisions about their child's care.

A nurse is caring for a toddler who was diagnosed with an inoperable brain tumor. The parents are having difficulty deciding on a course of action for their child. Why is it important to have the nurse involved in an ethical discussion about a planned course of treatment? 1. The nurse is viewed as the authority on ethical issues at the hospital. 2. The nurse can act as a liaison between the child, the child's parents, and the health care team. 3. The nurse can easily make time to discuss issues with the parents. 4. It isn't important to involve the nurse in this type of discussion.

2. Squatting RATIONALE: Placing the child in a squatting position sequesters a large amount of blood to the legs, reducing venous return. Sitting with the head of the bed at a 45-degree angle, lying flat, and lying on the right side don't reduce venous return; therefore, they won't relieve the child's dyspnea and cyanosis. A child with TOF may also assume a knee-chest position to reduce venous return to the heart.

A nurse is caring for a young child with tetralogy of Fallot (TOF). The child is upset and crying. The nurse observes that he's dyspneic and cyanotic. Which position would help relieve the child's dyspnea and cyanosis? 1. Sitting in bed with the head of the bed at a 45-degree angle 2. Squatting 3. Lying flat in bed 4. Lying on his right side

1. Reposition the infant as often as needed. 2. Let the infant play with his favorite toy. 3. Allow the infant's family to participate in his care as much possible. 4. Explain to the infant what she's going to do before she does it. RATIONALE: Frequent repositioning helps decrease discomfort and gives the nurse an opportunity to assess for changes in status. Infants and children derive comfort and security from playing with a favorite toy or animal. Such play should be encouraged as long as it's permitted. Familiarity is a positive force with children, and parents should be encouraged to participate in their child's care. The nurse should explain her actions to the infant. Although the infant may not understand each event, it's better for the nurse to provide an explanation rather than leave the infant fearful of what might happen. It isn't necessary for an infant who has undergone surgery to get at least 14 hours of sleep per night. Pain, comfort level, and general anxiety may prevent him from receiving much sleep in the acute-care setting. Giving the infant favorite foods in the first 24 to 48 postoperative hours may not be an option; physicians order postoperative diet regimens.

A nurse is caring for an 18-month-old infant 24 hours after surgery to repair a fractured tibia. Which comfort interventions are appropriate? Select all that apply. 1. Reposition the infant as often as needed. 2. Let the infant play with his favorite toy. 3. Allow the infant's family to participate in his care as much possible. 4. Explain to the infant what she's going to do before she does it. 5. Be sure the infant gets at least 14 hours of sleep each night. 6. Give the infant his favorite foods.

3. Before a respiratory therapy treatment, wheezing isn't heard on auscultation. RATIONALE: Typically, before a respiratory therapy treatment, wheezing has increased and the child has increased respiratory distress. No wheezing on auscultation is an indication that the child isn't moving air in and out and is in respiratory distress. A respiratory rate of 24 breaths/minute in an 8-year-old child is normal. An oxygen saturation of 95% is somewhat of a concern, possibly indicating that the child needs oxygen or needs to clear the airways. However, this finding is a lower priority than no wheezing on auscultation. The fact that the mother makes the 8-year-old child responsible for taking medications is of concern and needs to be investigated, but this isn't as important at this time as the lack of wheezing.

A nurse is caring for an 8-year-old child with acute asthma exacerbation. Which situation would be of greatest concern to the nurse? 1. The child's respiratory rate is now 24 breaths/minute. 2. Recent blood gas analysis indicates an oxygen saturation of 95%. 3. Before a respiratory therapy treatment, wheezing isn't heard on auscultation. 4. The child's mother reports that the child sometimes forgets to take the inhalers.

2. Ask the girl's parents if they keep firearms in their home. 3. Ask the girl if she's currently having suicidal thoughts. 4. Assist the girl with bathing and grooming as needed. 5. Inspect the girl's mouth after giving oral medications. RATIONALE: Safety is the primary consideration when caring for suicidal clients. Because firearms are the most common method used in suicides, the girl's parents should be encouraged to remove firearms from the home, if applicable. Safety also includes assessing for current suicidal ideation. In many cases, clients who are suicidal are depressed and don't have the energy to care for themselves, so the client may need assistance with bathing and grooming. Because depressed and suicidal clients may hide pills in their cheeks, the nurse should inspect the girl's mouth after giving oral medications. Rather than limit care, the nurse should try to establish a trusting relationship through nursing interventions and therapeutic communication. The girl can't be assured of confidentiality when self-destructive behavior is an issue.

A nurse is caring for an adolescent girl who was admitted to the hospital's medical unit after attempting suicide by ingesting acetaminophen (Tylenol). The nurse should incorporate which interventions into the care plan for this girl? Select all that apply. 1. Limit care until the girl initiates a conversation. 2. Ask the girl's parents if they keep firearms in their home. 3. Ask the girl if she's currently having suicidal thoughts. 4. Assist the girl with bathing and grooming as needed. 5. Inspect the girl's mouth after giving oral medications. 6. Assure the girl that anything she says will be held in strict confidence.

3. cover the opening with petroleum gauze. RATIONALE: If a chest tube is accidentally removed, the nurse should cover the insertion site with sterile petroleum gauze. The nurse should then observe the client for respiratory distress because tension pneumothorax may develop. If tension pneumothorax does develop, the nurse should remove the gauze to allow air to escape. The nurse shouldn't reintroduce the tube. Rather, the nurse should have another staff member call a physician so another tube can be introduced by the physician under sterile conditions.

A nurse is caring for an adolescent involved in a motor vehicle crash. The adolescent has a chest tube in place. If the chest tube is accidentally removed, the nurse should immediately: 1. reintroduce the tube and attach it to water seal drainage. 2. call a physician and obtain a chest tray. 3. cover the opening with petroleum gauze. 4. clean the wound with povidone-iodine and apply a gauze dressing.

3. Deficient fluid volume (hemorrhage) RATIONALE: Deficient fluid volume (hemorrhage) is of highest priority because the spleen is a vascular organ. Laceration may lead to hemorrhage. Risk for injury related to unsteady gait isn't indicated in this situation. Disturbed body image isn't a concern because the adolescent doesn't have a visible injury. Although the adolescent may be placed on bed rest for 5 to 7 days, Impaired physical mobility isn't the priority nursing diagnosis.

A nurse is caring for an adolescent who has been diagnosed with a spleen laceration resulting from a skateboard accident. Which nursing diagnosis should be the highest priority? 1. Risk for injury related to unsteady gait 2. Disturbed body image 3. Deficient fluid volume (hemorrhage) 4. Impaired physical mobility

2. develop an identity and independence. RATIONALE: An adolescent strives for a sense of independence and identity. During this time, conflicts are heightened, not resolved. Trust begins to develop during infancy and matures during the course of development. Adolescents rarely finalize plans for the future; this normally happens later in adulthood.

A nurse is caring for an adolescent who underwent surgery for a perforated appendix. When caring for this adolescent, the nurse should keep in mind that the main life-stage task for an adolescent is to: 1. resolve conflict with parents. 2. develop an identity and independence. 3. develop trust. 4. plan for the future.

2. The nurse attempts to influence the family's decisions by presenting her own thoughts and opinions. RATIONALE: When a nurse attempts to influence a family's decision with her own opinions and values, the situation becomes one of overinvolvement on the nurse's part, creating a nontherapeutic relationship. When a nurse keeps communication channels open, works with family members to decrease their dependence on health care providers, and instructs family members so they can accomplish tasks independently, she has developed an appropriate therapeutic relationship.

A nurse is concerned about another nurse's relationship with the members of a family and their ill preschooler. Which behavior should be brought to the attention of the nurse-manager? 1. The nurse keeps communication channels open among herself, the family, physicians, and other health care providers. 2. The nurse attempts to influence the family's decisions by presenting her own thoughts and opinions. 3. The nurse works with the family members to find ways to decrease their dependence on health care providers. 4. The nurse has developed teaching skills to instruct the family members so they can accomplish tasks independently.

RATIONALE: Chest circumference is most accurately measured by placing the measuring tape around the infant's chest with the tape covering the nipples. If measured above or below the nipples, a false measurement is obtained.

A nurse is conducting a physical examination on an infant. Identify the anatomical landmark she should use to measure chest circumference.

4. Incompatibility between the child's history and the injury RATIONALE: Incompatibility between the history and the injury is the most important criterion on which to base the decision to report suspected child abuse. For example, the child may have a skull fracture but the parents state that the child fell off of the sofa. The other criteria also may suggest child abuse but are less reliable indicators.

A nurse is deciding whether to report a suspected case of child abuse. Which criterion is the most important for the nurse to consider? 1. Inappropriate parental concern for the degree of injury 2. Absence of parents to question about the injury 3. Inappropriate response of the child to the injury 4. Incompatibility between the child's history and the injury

4. Blood pressure monitoring RATIONALE: Because poststreptococcal glomerulonephritis may cause severe, life-threatening hypertension, it is most important for the nurse to teach the parents how to monitor the child's blood pressure. Infection control, nutritional planning, and prevention of streptococcal pharyngitis are important but are secondary to blood pressure monitoring.

A nurse is developing a teaching plan for a child with acute poststreptococcal glomerulonephritis. What is the most important point to address in this plan? 1. Infection control 2. Nutritional planning 3. Prevention of streptococcal pharyngitis 4. Blood pressure monitoring

1. Decreased hematuria RATIONALE: Decreased hematuria, a sign of improving kidney function, typically is the first sign that a child with APSGN is improving. Increased appetite, an increased energy level, and decreased diarrhea aren't specific to APSGN.

A nurse is evaluating a child with acute poststreptococcal glomerulonephritis (APSGN) for signs of improvement. Which finding typically is the earliest sign of improvement? 1. Decreased hematuria 2. Increased appetite 3. Increased energy level 4. Decreased diarrhea

RATIONALE: The heart's apex for a toddler is located at the fourth intercostal space immediately to the left of the midclavicular line. It's one or two intercostal spaces above what's considered normal for the adult because the heart's position in a child of this age is more horizontal and larger in diameter than that of an adult.

A nurse is feeling the apical impulse of a 28-month-old child. Identify the area where the nurse should assess the apical impulse.

3. Documenting the care provided during her shift RATIONALE: Documentation should take top priority because it's the only way the nurse can legally claim that interventions were performed. Changing linens, restocking supplies, and emptying trash cans would be appreciated by the nurses on the oncoming shift but aren't mandatory and don't take priority over documentation.

A nurse is finishing her shift on the pediatric unit. Because her shift is ending, which intervention takes top priority? 1. Changing the linens on the clients' beds 2. Restocking the bedside supplies needed for a dressing change on the upcoming shift 3. Documenting the care provided during her shift 4. Emptying the trash cans in the assigned client rooms

2. "My infant may ride in a front-facing car seat as soon as he weighs 21 pounds." RATIONALE: An infant must be at least 1 year old and weigh at least 20 lb (9.1 kg) to move from a rear-facing car seat to a front-facing car seat. The make or model of the vehicle is irrelevant.

A nurse is giving discharge instructions to a parent of a 13-month-old infant who weighs 18 lb (8.2 kg). The nurse knows the parent understands car-seat safety when the parent states: 1. "My infant may ride in a front-facing car seat because he's 1 year old." 2. "My infant may ride in a front-facing car seat as soon as he weighs 21 pounds." 3. "If I have a sports utility vehicle, my infant may ride in a rear-facing or front-facing car seat." 4. "My child will need to ride in a rear-facing care seat until he's 3 years old."

3. avoiding areas of low oxygen concentration such as high altitudes. RATIONALE: The child should avoid areas of low oxygen, such as high altitudes, because they may precipitate sickle cell crisis. Applying warm compresses will reduce discomfort to the affected area; cold compresses, however, may add to discomfort by increasing sickling and impairing circulation. The child should be encouraged to drink fluids to rehydrate cells. Strenuous exercise may induce, not reduce, sickle cell crisis.

A nurse is giving instructions to parents of a school-age child diagnosed with sickle cell anemia. The instructions should include: 1. applying cold to affected areas to reduce the child's discomfort. 2. restricting the child's fluids during crisis situations. 3. avoiding areas of low oxygen concentration such as high altitudes. 4. encouraging the child to exercise to reduce the likelihood of crisis.

4. "I'll tell my child that the new baby can be a playmate when he arrives." RATIONALE: Telling a toddler that he will have a new playmate when the baby arrives sets up unrealistic expectations and, therefore, indicates the client needs more instruction. The parents should stress activities that will take place, such as feeding, changing, and crying. Giving the toddler a doll is a good strategy because having the doll allows the toddler to take part in the new routines. For example, the toddler can pretend to meet the needs of the doll just like the mother tends to the baby. Participation in a sibling preparation class may also decrease sibling rivalry behaviors. Discussing changes in family routines will help the toddler know what to expect.

A nurse is helping a pregnant client devise a plan to help her 2-year-old child adjust to the birth of her second child. Which statement by the client indicates more instruction is needed? 1. "I'll give my child a doll so he can imitate us when we care for the new baby." 2. "I'll enroll my child in a sibling class. 3. "I'll discuss with my child what routines will be the same and what will be different after the baby arrives." 4. "I'll tell my child that the new baby can be a playmate when he arrives."

2. "After advancing both crutches the length of one step, move your 'bad' leg forward." RATIONALE: When walking with crutches, a child should be instructed to advance both crutches, then advance the affected leg. The unaffected leg then supports much of the weight associated with ambulation. It wouldn't be effective to move the unaffected leg forward first. It wouldn't be safe for the child to advance only one crutch.

A nurse is instructing a school-age child with a fracture on proper use of crutches. Which statement made by the nurse is most accurate? 1. "After advancing both crutches the length of one step, move your 'good' leg forward." 2. "After advancing both crutches the length of one step, move your 'bad' leg forward." 3. "Move one crutch forward, then advance your 'good' leg." 4. "Move one crutch forward, then advance your 'bad' leg."

1. Recent bed-wetting RATIONALE: Polyuria, recognized by parents as bed-wetting in a child recently toilet-trained, is a hallmark of type 1 diabetes mellitus. Polyphagia is also a hallmark of type 1 diabetes mellitus. A parent is also likely to report weight loss despite excessive eating, not weight gain or a poor appetite. The child with type 1 diabetes mellitus may complain of fatigue rather than boundless energy.

A nurse is interviewing the mother of a 7-year-old child. Which symptom reported by the mother leads the nurse to suspect that the child has type 1 diabetes? 1. Recent bed-wetting 2. Poor appetite 3. Weight gain 4. Boundless energy

1. most toddler deaths are accidental. RATIONALE: Most toddler deaths are accidental. Many injuries or deaths in this age-group result from fire, drowning, motor vehicle accidents, and firearms. Toddlers don't generally overdose on medications, although this situation could happen if a toddler were given too much medication in the home or hospital setting. A child must be older than age 12 months and weigh more than 20 lb (9.1 kg) to ride in a front-facing car seat. Toddlers are at higher risk for injury than adults because of their developmental level and their limited ability to distinguish right from wrong and to recognize danger signs.

A nurse is leading a group of parents of toddlers in a discussion on home safety. The nurse should emphasize the fact that: 1. most toddler deaths are accidental. 2. medication overdose is the leading cause of death in toddlers. 3. any infant older than age 12 months can safely ride in a front-facing car seat. 4. a toddler's risk of injury is the same as that of an adult.

3. An infant requiring abdominal dressing changes for a wound infection RATIONALE: The infant requiring dressing changes is within an LPN's scope of practice. This care has a predictable outcome. Client and family teaching — such as how to care for a gastrostomy tube — is an RN's responsibility. A client care assistant can be assigned to obtain vital signs and report the findings to the supervising RN. Because the outcome of the infant with agonal respirations is unpredictable, the RN shouldn't delegate his care to the LPN.

A nurse is making assignments for the infant unit. The shift's team members include a licensed practical nurse (LPN) with 10 years of experience, a registered nurse (RN) with 3 months of experience, and a client care assistant. Which assignment is most appropriate for the LPN? 1. An infant being discharged to home following placement of a gastrostomy tube 2. An infant just returned from the postanesthesia care unit who requires hourly assessment of vital signs 3. An infant requiring abdominal dressing changes for a wound infection 4. An infant with agonal respirations who is receiving palliative care

3. 1 hour after administering insulin RATIONALE: Short-acting insulins peak in 30 minutes to 2 hours after administration. Therefore, the nurse should check the child's blood glucose level during this period, such as 1 hour after administration. Measuring the glucose level immediately before or 15 minutes after administering insulin would be too soon. Waiting until 4 hours after administering insulin would be too late to obtain an accurate reading.

A nurse is preparing to administer short-acting insulin to a child with type 1 diabetes. When should the nurse measure the child's blood glucose level? 1. Immediately before administering insulin 2. 15 minutes after administering insulin 3. 1 hour after administering insulin 4. 4 hours after administering insulin

1. "Can you ride a tricycle?" RATIONALE: Asking the child if he can ride a tricycle best helps evaluate the child's developmental status because a 4-year-old child should be able to perform such an action. A child may draw stick-like figures, but wouldn't be able to draw complicated pictures such as a school. A 4-year-old child may not be aware of his feelings, so asking whether he likes his brother wouldn't be appropriate. A 4-year-old child may not know his mother's first name, so asking it wouldn't evaluate developmental status.

A nurse is obtaining the history of a child, age 4. Which question best evaluates the child's developmental status? 1. "Can you ride a tricycle?" 2. "Can you draw your school?" 3. "Do you like your brother?" 4. "What's your mommy's first name?"

3. Moodiness RATIONALE: Moodiness may occur often during early adolescence. Frequent anger and combativeness are more typical of middle adolescence. Cooperativeness typically occurs during late adolescence.

A nurse is performing a psychosocial assessment on a 14-year-old adolescent. Which emotional response is typical during early adolescence? 1. Frequent anger 2. Cooperativeness 3. Moodiness 4. Combativeness

3. Moderate intercostal retractions RATIONALE: Normally, children and men use the abdominal muscles to breathe, whereas women use the thoracic muscles. Use of the accessory or intercostal muscles would indicate a respiratory problem and should be immediately reported to the physician. Mouth breathing and a foul odor from the mouth aren't cause for concern. Irregular respirations while awake aren't an unusual finding in a young child.

A nurse is performing a respiratory assessment on a 5-year-old child diagnosed with pneumonia. Which assessment finding should be reported to the physician immediately? 1. Mouth breathing 2. Foul odor from the mouth 3. Moderate intercostal retractions 4. Irregular respirations while awake

RATIONALE: The brachial pulse should be assessed when performing infant CPR. The carotid pulse, which is used in children and adults, is extremely difficult to locate in an infant because of his short neck.

A nurse is performing cardiopulmonary resuscitation (CPR) on an infant. Identify the area where the nurse should assess for a pulse.

3. strong food preferences. RATIONALE: A toddler can't be expected to use consistent table manners and, generally, the appetite decreases during the toddler stage because of a slowed growth rate. A toddler typically enjoys socializing during meals and commonly imitates others.

A nurse is planning a health teaching session for a group of parents with toddlers. When describing a toddler's typical eating pattern, the nurse should mention that many children of this age exhibit: 1. consistent table manners. 2. an increased appetite. 3. strong food preferences. 4. a preference for eating alone.

5 milliliters RATIONALE: To calculate the child's weight in kilograms, the nurse should use the following formula: 2.2 lb/1 kg = 33 lb/X kg X = 33 ÷ 2.2 X = 15 kg. Next, the nurse should calculate the daily dosage for the child: 50 mg/kg/day × 15 kg = 750 mg/day. To determine divided daily dosage, the nurse should know that "every 8 hours" means 3 times per day. So, she should perform that calculation in this way: Total daily dosage ÷ 3 times per day = Divided daily dosage 750 mg/day ÷ 3 = 250 mg The drug's concentration is 250 mg/5 ml, so nurse should administer 5 ml.

A nurse is preparing a dose of amoxicillin for a 3-year-old child with acute otitis media. The child weighs 33 lb. The dosage ordered is 50 mg/kg/day in divided doses every 8 hours. The concentration of the drug is 250 mg/5 ml. How many milliliters should the nurse administer? Record your answer using a whole number. Answer: milliliters

0.6 milligrams RATIONALE: To perform this dosage calculation, the nurse should first convert the child's weight to kilograms: 44 lb ÷ 2.2 kg/lb = 20 kg Then she should use this formula to determine the dose: 20 kg × 0.03 mg/kg = X mg X = 0.6 mg

A nurse is preparing to administer I.V. methylprednisolone sodium succinate (Solu-Medrol) to a child who weighs 44 lb. The order is for 0.03 mg/kg I.V. daily. How many milligrams should the nurse prepare? Record your answer using one decimal place. Answer: milligrams

1. I.V. tubing with a volume-control chamber RATIONALE: Because infants have a small circulating blood volume, inadvertent administration of extra I.V. fluid can cause fluid volume excess. To prevent this from occurring, I.V. tubing with a volume-control chamber (such as a Buretrol or Solu-set) should always be used for infants and children to closely regulate the amount of fluid infused. The volume-control chamber should be filled only with enough I.V. fluid for the next two 2 hours. A microdrip chamber that allows for 60 drops/ml (as opposed to a macrodrip chamber, which allows for 10 to 20 drops/ml, depending on the manufacturer) should be used to infuse the smaller amounts of I.V. fluids an infant needs. A filter is typically used only for the administration of total parenteral nutrition and certain blood products. Standard I.V. tubing for adults should be avoided for infants because of the inability to closely regulate the amount of fluid infused.

A nurse is preparing to administer an I.V. containing dextrose 10% in ¼ normal saline solution to a 6-month-old infant. The nurse should select which tubing to safely administer the solution? 1. I.V. tubing with a volume-control chamber 2. I.V. tubing with a macrodrip chamber 3. I.V. tubing with a special filter 4. Standard I.V. tubing used for adults

43.2 milligrams RATIONALE: To perform this dosage calculation, the nurse should first convert the client's weight to kilograms using this formula: 1 kg/2.2 lb = X kg/95 lb 2.2X = 95 X = 43.2 kg Then, she should calculate the client's daily dose using this formula: 43.2 kg × 3 mg/kg = 129.6 mg Lastly, the nurse should calculate the divided dose: 129.6 mg ÷3 doses = 43.2 mg/dose

A nurse is preparing to administer the first dose of tobramycin (Nebcin) to an adolescent with cystic fibrosis. The order is for 3 mg/kg I.V. daily in three divided doses. The client weighs 95 lb. How many milligrams should the nurse administer per dose? Record your answer using one decimal place. Answer: milligrams

RATIONALE: The vastus lateralis muscle, located in the thigh, is the muscle into which the nurse should administer an I.M. injection in the leg of a toddler. To give an injection into the vastus lateralis muscle, the nurse should divide the distance between the greater trochanter and the knee joints into quadrants. The injection should be given in the center of the upper quadrant.

A nurse is preparing to give an I.M. injection in the left leg of a 2-year-old child. Identify the area where the nurse should give the injection.

2. Adolescents are worried about appearing different from their peers. RATIONALE: Adolescents have a strong need to belong, and they seek social approval from their peers. Knowing this information will help the nurse construct an effective teaching plan. Adolescents are capable of following detailed instructions. According to Piaget, adolescents are at the formal operations stage and are capable of deductive, reflective, and hypothetical reasoning. Fine motor coordination is well developed by adolescence. According to Erikson's stages of psychosocial development, adolescence is the stage of identity versus role confusion. During this stage, the adolescent strives to establish a sense of identity; identity isn't already well-developed.

A nurse is preparing to teach a 13-year-old adolescent with asthma to administer his own breathing treatments. Which principle should the nurse keep in mind when planning the teaching session? 1. Adolescents are unable to follow detailed instructions. 2. Adolescents are worried about appearing different from their peers. 3. Adolescents' fine motor coordination isn't sufficiently developed to administer treatments. 4. Adolescents have a well-developed sense of self-identity.

4. use the heel of one hand for sternal compressions. RATIONALE: The nurse should use the heel of one hand and compress one-third to one-half the depth of the chest. The nurse should use the heels of both hands clasped together and compress the sternum 1½″ to 2″ for an adult. For a small child, two-person rescue may be inappropriate. For a child, the nurse should deliver 20 breaths/minute instead of 12 breaths/minute.

A nurse is providing cardiopulmonary resuscitation (CPR) to a child, age 4. The nurse should: 1. compress the sternum with both hands at a depth of 1½″ to 2″ (4 to 5 cm). 2. deliver 12 breaths/minute. 3. perform only two-person CPR. 4. use the heel of one hand for sternal compressions.

3. "Our child should avoid eating prepared puddings." RATIONALE: Teaching is effective if the parents identify prepared puddings as a food their child should avoid. A child with celiac disease mustn't consume foods containing gluten and therefore should avoid prepared puddings, commercially prepared ice cream, malted milk, and all food and beverages containing wheat, rye, oats, or barley. The other options don't contain gluten and are permitted on a gluten-free diet.

A nurse is providing dietary teaching for the parents of a child with celiac disease. Which statement by the parents indicates effective teaching? 1. "Our child should avoid eating vegetables." 2. "Our child should avoid eating fruits." 3. "Our child should avoid eating prepared puddings." 4. "Our child should avoid eating rice."

4. "The gun should be stored in a locked cabinet." RATIONALE: The nurse should instruct the parents to keep the gun in a locked cabinet. Keeping the gun out of the child's sight isn't sufficient; the child might be able to locate the gun. It's inappropriate to refer a school-age child to a gun-safety program. The parents shouldn't keep the gun on hand with the understanding that the child won't touch it.

A nurse is providing injury-prevention education to the parents of a school-age child. The parents admit that they keep a gun in their home. Which of the nurse's statements is most appropriate? 1. "The gun should be kept in a discreet location out of your child's sight." 2. "Your child should attend a community gun-safety program." 3. "Remind your child that only a parent may touch the gun." 4. "The gun should be stored in a locked cabinet."

2. Teach blood glucose monitoring. RATIONALE: Teaching blood glucose monitoring and the use of equipment is necessary in diabetic teaching within a care plan that focuses on demonstrating testing blood glucose levels, not a care plan that focuses on proper hygiene. Encouraging regular dental care, teaching proper care of cuts and scratches, which minimizes the risk of infection, and teaching proper foot care are all appropriate for a teaching plan focusing on proper hygiene for a child with type 1 diabetes.

A nurse is reviewing a care plan for a 10-year-old child who has recently been diagnosed with type 1 diabetes. Which instruction should the nurse remove from a teaching plan focusing on proper hygiene? 1. Encourage regular dental care. 2. Teach blood glucose monitoring. 3. Teach care of cuts and scratches. 4. Teach proper foot care.

4. "We'll get the baby a push toy." RATIONALE: Effective teaching is demonstrated if the parents say they'll get the baby a push toy because at age 10 months, a push toy promotes development of an infant's gross and fine motor skills and aids cognitive development. A mobile provides appropriate visual stimulation for an infant up to age 4 months; after this age, a mobile may pose a danger to an infant. Rattles and brightly colored blocks promote gross and fine motor abilities in infants ages 4 to 8 months.

A nurse is teaching parents how to select appropriate toys for their 10-month-old infant. Which statement by the parents indicates effective teaching? 1. "We'll get a mobile to place over the baby's crib." 2. "We'll get a rattle for the baby to play with." 3. "We'll get the baby some brightly colored blocks." 4. "We'll get the baby a push toy."

4. Performing frequent visual assessments of jaundice RATIONALE: Visual assessment of jaundice isn't a valid method for assessing jaundice. Serum bilirubin levels must be checked every 4 to 12 hours. Repositioning the infant and shielding the infant's eyes are appropriate interventions for an infant undergoing phototherapy and should be included in the care plan.

A nurse is reviewing a care plan for an infant undergoing phototherapy for hyperbilirubinemia. Which intervention should the nurse remove from the care plan? 1. Repositioning the infant frequently to expose all body surfaces 2. Obtaining frequent serum bilirubin levels 3. Shielding the infant's eyes with an opaque mask to prevent exposure to the light 4. Performing frequent visual assessments of jaundice

1. Chickenpox RATIONALE: Reye's syndrome commonly occurs about 1 week after a child has had a viral infection, such as chickenpox (varicella) or influenza. Children with flulike symptoms or chickenpox who receive aspirin are at increased risk for Reye's syndrome. Bacterial meningitis and strep throat are caused by bacteria and don't lead to Reye's syndrome. Lyme disease is caused by a spirochete and isn't implicated in Reye's syndrome.

A nurse is taking a history from the parents of a 11-year-old girl admitted with Reye's syndrome. Which illness should the nurse expect the parents to report their child having the previous week? 1. Chickenpox 2. Bacterial meningitis 3. Strep throat 4. Lyme disease

1. Eustachian tubes RATIONALE: The nurse should mention the importance of the eustachian tubes because they're short in a child and lie in a horizontal plane, promoting entry of nasopharyngeal secretions into the tubes and thus setting the stage for otitis media. The nasopharynx, tympanic membrane, and external ear canal have no unusual features that would predispose a child to otitis media.

A nurse is teaching a group of parents about otitis media. When discussing why children are predisposed to this disorder, the nurse should mention the significance of which anatomical feature? 1. Eustachian tubes 2. Nasopharynx 3. Tympanic membrane 4. External ear canal

1. Automobile accidents 2. Drowning 3. Pedestrian accidents 4. Fire RATIONALE: Preschoolers are most susceptible to accident-related injuries. Preschoolers are naturally curious and can't anticipate the results of their actions, which can result in accidents. Sexually transmitted diseases and homicide aren't special risks for preschoolers.

A nurse is teaching a safety class for parents of preschoolers. Which injuries should the nurse include as common among preschoolers? Select all that apply. 1. Automobile accidents 2. Drowning 3. Pedestrian accidents 4. Fire 5. Sexually transmitted diseases 6. Homicide

2. observe for behavioral changes. RATIONALE: A well-fitting helmet is the most important safety feature to stress to children and parents because, according to the American Academy of Pediatrics, wearing a helmet correctly can prevent or lessen the severity of brain injuries resulting from bicycle crashes. Knee pads, elbow pads, and reflectors are also important safety devices but they aren't as important as a helmet.`

A nurse is teaching bicycle safety to a child and his parents. What protective device should the nurse tell the parents is most important in preventing or lessening the severity of injury related to bicycle crashes? 1. Helmet 2. Knee pads 3. Elbow pads 4. Reflectors

3. have the adolescents crawl around on the floor to look for potential hazards. RATIONALE: Crawling on the floor is a participative activity that can help promote understanding of infant safety in relation to the infant's perspective. The nurse doesn't need to instruct the adolescents to discuss infant safety with their pediatricians because she can provide such information in the class environment. Presenting a lecture or video doesn't directly focus on the infant's perspective on safety.

A nurse is teaching childcare classes for adolescent mothers. To enhance the adolescents' understanding of infant safety in relation to the infant's perspective, the nurse should: 1. instruct the adolescents to discuss infant safety with their pediatricians. 2. present a video about pregnancy prevention. 3. have the adolescents crawl around on the floor to look for potential hazards. 4. lecture the adolescents about poison control.

1. "We won't start any new foods now." RATIONALE: The parents show understanding of their infant's dietary needs by stating they won't start any new foods. Breast milk provides all the nutrients a full-term infant needs for the first 6 months. They shouldn't provide skim milk because it doesn't have sufficient fat for infant growth. The parents also shouldn't provide solid foods, such as cereal and fruit, before age 6 months because an infant's GI tract doesn't tolerate them well.

A nurse is teaching parents about the nutritional needs of their full-term infant, age 2 months, who's breast-feeding. Which response shows that the parents understand their infant's dietary needs? 1. "We won't start any new foods now." 2. "We'll start the baby on skim milk." 3. "We'll introduce cereal into the diet now." 4. "We should add new fruits to the diet one at a time."

3. teach children the importance of proper hand washing. RATIONALE: The spread of childhood infections, including impetigo, can be reduced when children are taught proper hand-washing technique. Because impetigo is spread through direct contact, covering the mouth and nose when sneezing won't prevent its spread. Currently, there is no vaccine to prevent a child from contracting impetigo. Isolating the child with impetigo is unnecessary.

A nurse is teaching parents how to reduce the spread of impetigo. The nurse should encourage parents to: 1. teach children to cover mouths and noses when they sneeze. 2. have their children immunized against impetigo. 3. teach children the importance of proper hand washing. 4. isolate the child with impetigo from other members of the family.

2. "I know that this disease is serious and can lead to asthma." RATIONALE: By saying bronchiolitis places the child at risk for developing asthma, the mother demonstrates understanding of her infant's condition. If diagnosed and treated promptly, most infants recover from the illness and return home. Infants typically don't have recurrences of bronchiolitis. Infants diagnosed with bronchiolitis rarely require mechanical ventilation.

A nurse is teaching the mother of a 5-month-old infant diagnosed with bronchiolitis. Which statement by the mother indicates that teaching has been effective? 1. "I hope my baby will come home from the hospital." 2. "I know that this disease is serious and can lead to asthma." 3. "My baby needs to be cured this time so it won't happen again." 4. "My baby has been sick. A machine will help him breathe."

3. call the poison control center. RATIONALE: Before intervening in any way, the parents should first call the poison control center for specific instructions. Ipecac syrup is no longer recommended for the ingestion of poisons. The parents may have to call an ambulance after calling the poison control center. Punishment for being bad isn't appropriate because the parents are responsible for making the environment safe.

A nurse is teaching the parents of a young child how to handle suspected poisoning. If the child ingests poison, the parents should first: 1. administer ipecac syrup. 2. call an ambulance. 3. call the poison control center. 4. punish the child for being bad.

4. "Immunizations will have to be delayed until the casts come off." RATIONALE: The father's statement about delaying immunizations indicates the need for further teaching. Immunizations can be administered in the thighs because the casts cover only the lower legs and feet. The other responses are correct statements, indicating effective teaching.

A nurse is teaching the parents of an infant with clubfeet about cast care. Which statement by the father indicates the need for further teaching? 1. "I hope this cast will cure his feet in the next several weeks." 2. "I know I will have to be careful when changing his diapers." 3. "We will have to be careful how we hold our baby." 4. "Immunizations will have to be delayed until the casts come off."

4. Use care pathways to specify care and identify daily outcomes. RATIONALE: Using care pathways to specify care and identify daily outcomes ensures that clients progress toward a timely discharge and that resources are used appropriately. A longer hospital stay requires more resources, which, in turn, leads to a more costly health care bill. Generic brands are less expensive than brand name products; therefore, their use should be encouraged. Filling a personal medicine cabinet with supplies from work constitutes stealing and offering the unit phones to parents generates higher phone bills.

A nurse manager of the pediatric unit discovers that she's overbudget on supplies. How could each nurse assigned to the unit help with cost containment? 1. Order only brand-name supplies instead of the generic equivalent. 2. Use the supply closet at work to stock personal medicine cabinets because the supplies are free. 3. Offer clients' parents the use of unit phones. 4. Use care pathways to specify care and identify daily outcomes.

4. Logging off a computer containing client information RATIONALE: All members of the health care team are required to maintain strict client confidentiality, including securing electronic client information. Therefore, the clerical support staff should be instructed about the importance of logging off a computer containing client information immediately after use. Taking a verbal order, administering medications, and client education aren't within the scope of practice of the clerical support staff.

A nurse manager of the pediatric unit is responsible for making sure that each staff member reviews the unit policies annually. What policy should the nurse manager emphasize with the clerical support staff? 1. Proper documentation of a verbal order from a physician 2. Policy changes in the administration of opioids 3. New education materials for the management of diabetes 4. Logging off a computer containing client information

4. conducting root cause analysis. RATIONALE: Root cause analysis is used to gather information about factors that contribute to a problem (root causes) so that the nurse can identify ways to correct the problem. Random observation doesn't necessarily produce data to explain a specific sentinel event. Evaluation of a single incident rarely identifies underlying causes and contributing factors to sentinel events. An expert consultation doesn't necessarily reveal site-specific underlying causes and contributing factors in an individual health care facility.

A nurse may use the performance improvement process to determine underlying causes and contributing factors related to sentinel events by: 1. randomly observing client care without advance warning. 2. evaluating a single incident that resulted in an unanticipated outcome. 3. requesting that a documented expert in the field perform a review. 4. conducting root cause analysis.

3. "Here is your medicine. Would you like apple juice or grape drink after?" RATIONALE: Asking the child if he would like apple juice or grape drink is the best approach because involving the child promotes cooperation, and permitting the child to make choices provides a sense of control. Telling a child to take the medicine "right now" could provoke a negative response. Promising that the child will go home sooner could decrease the child's trust in nurses and physicians. Telling the child to "see how Jimmy took his medicine" is inappropriate because it compares one child with another and doesn't encourage cooperation.

A nurse must administer an oral medication to a 3-year-old child. The best way for the nurse to proceed is by saying: 1. "It's time for you to take your medicine right now." 2. "If you take your medicine now, you'll go home sooner." 3. "Here is your medicine. Would you like apple juice or grape drink after?" 4. "See how Jimmy took his medicine? He's a good boy. Now it's your turn."

3. distract the infant with a more appropriate toy. RATIONALE: Distraction with an appropriate chewing toy provides safety and is developmentally supportive. Removing the security device isn't appropriate; it must remain attached to the infant. Telling an infant not to chew on the security device isn't appropriate because chewing is typical behavior at the age of 10 months. Instructing the infant's parents about the safety hazard isn't the best response; doing so won't eliminate the immediate hazard and doesn't refocus the infant's attention.

A nurse observes a 10-month-old infant chewing on the security alarm attached to his identification bracelet. The nurse should: 1. remove the security device because it's a choking hazard. 2. instruct the infant to stop chewing on the device. 3. distract the infant with a more appropriate toy. 4. instruct the infant's parent regarding the safety hazard.

3. two children side by side in the sandbox building sand castles. RATIONALE: Two-year-olds exhibit parallel play; that is, they engage in similar activity, side by side. Playing dodgeball and tag are examples of interactive play, common to school-age children. Playing with clay and using flash cards are behaviors seen in preschool children.

A nurse observes a play group of 2-year-old children. The nurse expects to see: 1. four children playing dodgeball. 2. three children playing tag. 3. two children side by side in the sandbox building sand castles. 4. one child playing with clay and another child using flash cards.

1. express negativism. RATIONALE: A toddler's increasing autonomy is commonly expressed by negativism. They're unreliable in expressing pain — they respond just as strongly to painless procedures as they do to painful ones. Toddlers have little concept of danger and have common fears.

A nurse on the pediatric floor is caring for a toddler. The nurse should keep in mind that toddlers: 1. express negativism. 2. have reliable verbal responses to pain. 3. have a good concept of danger. 4. have little fear.

4. Taking a lunch break RATIONALE: Taking a lunch break takes lowest priority over child care. If the nurse is unable to delegate child care responsibilities to another nurse or nursing assistant, the nurse's lunch break needs to be rescheduled. A fever indicates an adverse reaction to the blood transfusion, and requires immediate intervention. The postsurgical child is losing blood through the surgical incision, which also requires attention. The telephone call is important for medication changes and to prevent a delay in treatment.

A nurse on the pediatric unit is caring for a group of preschool children. Which situation takes lowest priority? 1. A child who develops a fever during a blood transfusion 2. A child admitted from the postanesthesia care unit who has a blood-saturated surgical dressing 3. A physician waiting on the telephone to give the nurse a verbal order 4. Taking a lunch break

4. call for assistance. RATIONALE: After 2 minutes of CPR, the nurse should call for assistance and then resume efforts. CPR shouldn't be stopped after it has been started unless the nurse is too exhausted to continue. A cycle usually ends with breaths, so the next beginning cycle after pulse check and summoning help would begin with chest compressions.

A nurse performs cardiopulmonary resuscitation (CPR) for 1 minute on an infant without calling for assistance. In reassessing the infant after 2 minutes of CPR, the nurse finds he still isn't breathing and has no pulse. The nurse should then: 1. resume CPR beginning with breaths. 2. declare her efforts futile and stop CPR. 3. resume CPR beginning with chest compressions. 4. call for assistance.

4. registered dietitian, RN, physician, and infant's primary caregiver. RATIONALE : The registered dietitian, RN, physician, and infant's primary caregiver are crucial interdisciplinary team members who should participate in this care conference. The dietitian can address nutritional needs. The primary caregiver can provide input. The social worker and occupational therapist may become involved after the infant's condition improves, but they aren't crucial members of the team at this point.

A nurse plans a conference to discuss the care plan for an infant admitted to the hospital with a diagnosis of nonorganic failure to thrive. Appropriate participants in the care conference include the: 1. registered nurse (RN), physician, social worker, and infant's parents. 2. social worker, RN, occupational therapist, and dietitian. 3. infant's primary caregiver, RN, physician, and occupational therapist. 4. registered dietitian, RN, physician, and infant's primary caregiver.

3. Premature infants RATIONALE: Factors related to growth and maturation significantly alter an individual's capacity to metabolize and excrete drugs. Thus, the premature infant is at risk for problems because of immaturity. Deficiencies associated with immaturity become more important with decreasing age. Enzyme systems develop quickly, with most increasing to adult levels within 1 to 8 weeks after birth. Within the first year of life, all are probably as active as they will ever be.

A nurse should assess the maturity of enzyme systems (kidney and liver) in which pediatric population before administering medications? 1. Adolescents 2. Neonates 3. Premature infants 4. Toddlers

2. 12 months RATIONALE: The nurse should start screening a child for lead poisoning at age 12 months and perform repeat screenings at 24 months. High-risk infants, such as premature infants and formula-fed infants not receiving iron supplementation, should be screened for iron deficiency anemia at age 6 months. Regular dental visits should begin at age 24 months.

A nurse should begin screening for lead poisoning when a child reaches which age? 1. 6 months 2. 12 months 3. 18 months 4. 24 months

1. Ride a tricycle RATIONALE: The nurse should expect the child to ride a tricycle because, at age 3, gross motor development and refinement in eye-hand coordination enable a child to perform such an action. The fine motor skills required to tie shoelaces and the gross motor skills required for roller-skating and jumping rope develop around age 5.

A nurse should expect a 3-year-old child to be able to perform which action? 1. Ride a tricycle 2. Tie his shoelaces 3. Roller-skate 4. Jump rope

4. Acetaminophen (Tylenol) RATIONALE: The nurse should question an order to administer acetaminophen by intraosseous infusion because the drug can only be administered orally or rectally. Any medication that can be administered via I.V. can be administered by intraosseous infusion. Therefore, sodium bicarbonate, dopamine, and calcium chloride can all be administered by way of intraosseous infusion.

A nurse should question an order for intraosseous infusion of which agent? 1. Sodium bicarbonate 2. Dopamine (Intropin) 3. Calcium chloride 4. Acetaminophen (Tylenol)

4. Report the case to local authorities. RATIONALE: The nurse is required to report the case to local authorities because every state in the United States has laws for mandatory reporting of suspected child abuse and neglect. These cases are then referred to local agencies, such as Child Protective Services, for investigation. Social workers should be consulted before approaching a child and discussing child abuse. Confronting the mother could increase the risk of harm to the child and to the nurse. Discussing the case with another nurse breaches the client's confidentiality.

A nurse suspects that a toddler, who is admitted to the pediatric unit, has been physically abused by his mother. What is the nurse required to do? 1. Talk with the child about she suspects. 2. Confront the mother with her suspicions. 3. Discuss the case with another nurse during lunch break. 4. Report the case to local authorities.

2. Talk with the other nurse and try to work out differences so they don't affect client care. RATIONALE: When personal conflicts arise, it's always best to have the individuals involved try to work them out. If the differences are irreconcilable, other trained professionals may be needed to mediate the situation. Gossiping to other nurses, complaining to the nurse-manager, and avoiding the situation by working different shifts don't help resolve the problem.

A nurse working on the adolescent unit has a strained working relationship with a coworker and finds it difficult to work well with her. What is the best way for her to go about defusing this situation? 1. Ask other nurses assigned to the unit to see what they think might improve the situation. 2. Talk with the other nurse and try to work out differences so they don't affect client care. 3. Complain to the nurse-manager about the coworker's attitude. 4. Avoid the other nurse by working different shifts.

4. collaborating in development and implementation of the plan. RATIONALE: Collaboration is crucial in developing a disaster plan. Nurses must take an active role in disaster planning, but nurses aren't solely responsible for planning disaster response and conducting practice drills. Although the nurse should try to make sure that the neonates are safe during a disaster, she can't ensure on her own that all of them will be safe.

A nurse works in the neonatal intensive care unit. Her responsibility for disaster planning includes: 1. developing the plan for disaster response and conducting weekly practice drills. 2. following the disaster coordinator's instructions if a disaster occurs. 3. ensuring the safety of all neonates in the disaster area. 4. collaborating in development and implementation of the plan.

2. provide written instructions, education, and ongoing supervision. RATIONALE: When working with a nursing assistant, the nurse-manager must provide written instructions, education, and ongoing supervision. Although the nurse-manager should be concerned with the equitable division of work and proper payment for hours worked, these concerns aren't the highest priorities. The provision of health services to those in greatest need is an important overall goal, but isn't specific to working with a nursing assistant.

A nurse-manager for a community health organization is planning for the home health needs of an 8-year-old child who requires around-the-clock care by nursing assistants. The nurse-manager knows that when working with a nursing assistant, she must: 1. ensure that the work is divided equitably to prevent staff burnout and rapid turnover. 2. provide written instructions, education, and ongoing supervision. 3. ensure that the nursing assistant is paid fairly and for any additional time worked. 4. in the event of limited staff resources, provide health services to those in greatest need.

1. Gather data on possible reasons for this increase. RATIONALE: Gathering data about the reasons for infection or injury is within the scope of nursing practice. It wouldn't be appropriate for the nurse to contact infection control or the Centers for Disease Control and Prevention at this time. After gathering supporting data, the nurse should speak with the hospital administrator about her concerns and findings.

A nurse-manager in a pediatric intensive care unit notices an increase in nosocomial infections. What should the nurse do next? 1. Gather data on possible reasons for this increase. 2. Report the issue to the Centers for Disease Control and Prevention. 3. Notify infection control that staff members aren't wearing gloves. 4. Talk with the hospital administrator about her concerns.

3. obtain data about the types and frequency of infiltrations involved to conduct further study. RATIONALE: The nurse must obtain more information about the problem before implementing a change intended to improve performance on the unit. Developing an I.V. team, providing in-service education, and establishing a policy of restarting I.V. sites after 72 hours of infusion therapy aren't the best actions at this time.

A nurse-manager recognizes that infiltration commonly occurs during I.V. infusions for infants on the hospital's inpatient unit. The nurse-manager should: 1. develop an I.V. team with expertise in starting infant infusions. 2. provide nursing staff with in-service education about I.V. infusions. 3. obtain data about the types and frequency of infiltrations involved to conduct further study. 4. develop a policy for restarting all I.V. sites after 72 hours of infusion therapy.

1. toddlers. RATIONALE: The nurse knows that erratic eating is typical of toddlers because the physiologic need for food decreases at about age 18 months as growth declines from the rapid rate of infancy. The toddler also develops strong food and taste preferences, sometimes eating just one type of food for days or weeks and then switching to another.

A parent calls the pediatric clinic to express concern over her child's eating habits. She says the child eats very little and consumes only a single type of food for weeks on end. The nurse knows that this behavior is characteristic of: 1. toddlers. 2. preschool-age children. 3. school-age children. 4. adolescents.

4. performing a preoperative surgical scrub for at least 3 to 5 minutes. RATIONALE: The surgical team should perform a surgical scrub lasting at least 3 to 5 minutes before any operative procedure. Although surgical gowns may be considered sterile, surgical scrubs are considered clean rather than sterile. Jewelry harbors bacteria; team members should remove it rather than simply clean it. A surgical team member with an open wound shouldn't be involved in a procedure requiring asepsis.

A parent of a 9-year-old-child scheduled to have surgery expresses concern about the potential for postoperative infection. A nurse provides the parent with information about the measures taken to maintain surgical asepsis. Typical surgical asepsis involves: 1. using sterile surgical scrubs. 2. preoperative cleansing of jewelry worn by the surgical team. 3. applying bandages to cover any wounds surgical team members have. 4. performing a preoperative surgical scrub for at least 3 to 5 minutes.

2. Swelling of the genitals RATIONALE: The most likely finding for suspected sexual abuse would be difficulty walking or sitting; pain, swelling, or itching in the genitals; or bruises, bleeding, or lacerations of the genital area. Poor hygiene is a sign of physical neglect. Poor eye contact and fear of parents are common signs of physical, not sexual, abuse.

A pediatric nurse is caring for a child suspected of having been sexually abused. Which finding would best support the nurse's suspicions? 1. Poor hygiene 2. Swelling of the genitals 3. Fear of parents 4. Poor eye contact

2. ask the graduate what's bothering her. RATIONALE: Caring for acute or chronically ill children can be emotionally and physically stressful. A preceptor to a new nurse should be supportive and empathetic by asking about the new nurse's feelings. It isn't appropriate for the preceptor to make judgments by asking the new nurse if she thinks she can handle being a pediatric nurse, and it isn't acceptable for the preceptor to talk with the nurse-manager about the issue at this time. It isn't unusual for a nurse to need time to emotionally adjust to a new situation or new client population.

A pediatric nurse preceptor working on an oncology floor observes a new graduate crying in the nurses' lounge. The nurse's best action would be to: 1. let the graduate cry and get it out of her system. 2. ask the graduate what's bothering her. 3. ask the graduate if she thinks she can handle being a pediatric nurse. 4. let the nurse-manager know that the new graduate isn't ready for the emotions that working on this unit evokes.

2. Activity intolerance related to anemia RATIONALE: A nursing diagnosis of Activity intolerance related to anemia reflects the nurse's understanding of leukemia's physiologic effects because a child with leukemia may experience anemia from bone marrow depression, such as from chemotherapy or replacement of normal bone marrow elements by immature white blood cells. Anemia results in fatigue, lack of energy, and activity intolerance. The nurse's findings don't support the other diagnoses of Ineffective airway clearance related to fatigue, Imbalanced nutrition: More than body requirements related to lack of activity, and Ineffective cerebral tissue perfusion related to central nervous system infiltration by leukemic cells.

A physician diagnoses leukemia in a child, age 4, who complains of being tired and sleeps most of the day. Which nursing diagnosis reflects the nurse's understanding of the physiologic effects of leukemia? 1. Ineffective airway clearance related to fatigue 2. Activity intolerance related to anemia 3. Imbalanced nutrition: More than body requirements related to lack of activity 4. Ineffective cerebral tissue perfusion related to central nervous system infiltration by leukemic cells

2. The adolescent is estranged from his parents and lives independently. RATIONALE: An emancipated minor is a person younger than age 18 who is legally recognized as an adult under certain conditions. These conditions include becoming pregnant, getting married, graduating from high school, and living independently. Otherwise, an adolescent is considered a minor until his 18th birthday. Written consent must always be obtained, even if verbal consent is given. Major surgery, minor surgery, diagnostic tests such as biopsies, and treatments such as blood transfusions are all examples of procedures that require written informed consent.

A physician needs to obtain written informed consent for a surgical procedure on an adolescent. Which situation allows the physician to obtain written informed consent from the adolescent rather than his parents? 1. The adolescent's 18th birthday is the following week. 2. The adolescent is estranged from his parents and lives independently. 3. The adolescent gives his verbal consent to the procedure. 4. The physician doesn't need to obtain consent because the procedure is a minor one.

1. Transport personnel RATIONALE: Transport personnel are responsible for escorting clients throughout the hospital, including to various test locations. The physician isn't required to transport any client to the radiology department. The pharmacist is responsible for anything related to medications. The circulating nurse assists with surgical procedures in the operating room; she doesn't help transport clients to the X-ray department

A physician ordered an X-ray for an adolescent in the pediatric unit. With whom should the nurse collaborate to carry out this order? 1. Transport personnel 2. Physician 3. Pharmacist 4. Circulating nurse

1. None because this isn't a safe dose RATIONALE: For this client, the safe dose of this drug is 90.8 mg (9.08 kg × 10 mg/kg = 90.8 mg). This dose is equivalent to 2.8 ml. Therefore, the ordered dose isn't safe.

A physician orders acetaminophen (Tylenol) elixir, 160 mg every 4 hours, for a 14-month-old child who weighs 20 lb (9.08 kg). This drug, supplied in a bottle labeled 160 mg/tsp, has a safe dosage of 10 mg/kg/dose. The nurse should administer how many milliliters? 1. None because this isn't a safe dose 2. 2.5 ml 3. 5 ml 4. 7.5 ml

70 milliliters per hour RATIONALE: To perform this dosage calculation, the nurse should first convert the infant's weight to kilograms: 2.2 lb/kg = 22 lb/X kg X = 22 ÷ 2.2 X = 10 kg Next, she should multiply the infant's weight by the ordered rate: 10 kg × 7 ml/kg/hour = 70 ml/hour

A physician orders an I.V. infusion of dextrose 5% in quarter-normal saline solution to be infused at 7 ml/kg/hour for a 10-month-old infant. The infant weighs 22 lb. How many milliliters per hour should the nurse infuse of the ordered solution? Record your answer using a whole number. Answer: milliliters per hour

3. To decrease proteinuria RATIONALE: The primary purpose of administering corticosteroids to a child with nephrotic syndrome is to decrease proteinuria. Corticosteroids have no effect on blood pressure. Although they help reduce inflammation, this isn't the reason for their use in clients with nephrotic syndrome. Corticosteroids may predispose a client to, rather than prevent infection.

A physician orders corticosteroids for a child with nephrotic syndrome. What is the primary purpose of administering corticosteroids to this child? 1. To increase blood pressure 2. To reduce inflammation 3. To decrease proteinuria 4. To prevent infection

1. combat inflammation. RATIONALE: Corticosteroids are used to combat inflammation in a child with SLE. To prevent infection, the physician would order antibiotics. Aspirin is used to prevent platelet aggregation. Diuretics, not corticosteroids, promote diuresis.

A physician orders corticosteroids for a child with systemic lupus erythematosus (SLE). The nurse knows that the purpose of corticosteroid therapy for this child is to: 1. combat inflammation. 2. prevent infection. 3. prevent platelet aggregation. 4. promote diuresis.

4. apical pulse. RATIONALE: Because digoxin may reduce the heart rate and heart failure may cause a pulse deficit, the nurse should measure the toddler's apical pulse before administering the drug to prevent further slowing of the heart rate. The serum sodium level doesn't affect digoxin's action. For a child with heart failure, the nurse should check urine output and measure weight regularly, but not necessarily just before digoxin administration.

A physician orders digoxin (Lanoxin) elixir for a toddler with heart failure. Immediately before administering this drug, the nurse must check the toddler's: 1. serum sodium level. 2. urine output. 3. weight. 4. apical pulse.

2. "Using the monitor will help your physician determine the frequency of apneic events and how long monitoring is required." RATIONALE: Home apnea monitoring helps the physician determine the frequency of apneic events and how long monitoring is required. Use of home monitoring has been helpful in improving neonatal survival. Generally, most infants outgrow apnea of prematurity by the time they're 44 weeks postgestational age. The average length of monitoring is 6 weeks; only occasionally is it required beyond 1 year. The monitor can be removed for bathing and during times when parent or caregiver is physically present and actively engaged with the care of the infant.

A premature infant has been placed on a home apnea monitor. The nurse is giving discharge instructions to the parents. Which statement should the nurse include in the teaching? 1. "Your baby will probably need to be monitored until at least age 1." 2. "Using the monitor will help your physician determine the frequency of apneic events and how long monitoring is required." 3. "You can only give your baby sponge baths until monitoring is discontinued because it's dangerous to take the monitor off at any time." 4. "You can expect the monitoring to be discontinued by the time your baby is the equivalent of 34 postgestational weeks of age."

2. Turning and repositioning the child frequently RATIONALE: The child with nephrotic syndrome is at risk for skin breakdown from generalized edema. Because this syndrome typically impairs independent movement, the nurse's highest priority is to turn and reposition the child frequently to help prevent skin breakdown. Frequent turning also helps prevent respiratory infections, which may arise during the edematous phase of nephrotic syndrome. The syndrome typically causes hypotension, not hypertension, from significant loss of intravascular protein and a subsequent drop in oncotic pressure. Dietary sodium should be restricted because it worsens edema. Although the nurse should discuss the adverse effects of steroids with the parents, this action isn't a priority at this time.

A preschool-age child is admitted to the facility with nephrotic syndrome. Nursing assessment reveals a blood pressure of 100/60 mm Hg, lethargy, generalized edema, and dark, frothy urine. After prednisone (Deltasone) therapy is initiated, which nursing action takes highest priority? 1. Monitoring the child for hypertension 2. Turning and repositioning the child frequently 3. Providing a high-sodium diet 4. Discussing the adverse effects of steroids with the parents

4. Showing trust in the child's ability to cooperate even with an unpleasant procedure RATIONALE: To gain a preschooler's cooperation, the most appropriate strategy is for the nurse to show trust and express faith in the child's ability to cooperate even with an unpleasant procedure. Hiding the medication in milk may foster mistrust. The nurse should provide simple, not detailed, explanations and should use terms the child can understand. Shaming the child is inappropriate and may lead to feelings of guilt.

A preschool-age child refuses to take ordered medication. Which nursing strategy is most appropriate? 1. Mixing the medication in milk so the child isn't aware that it's there 2. Explaining the medication's effects in detail to ensure cooperation 3. Making the child feel ashamed for not cooperating 4. Showing trust in the child's ability to cooperate even with an unpleasant procedure

3. Having the child act out the surgical experience using dolls and medical equipment RATIONALE: Having the child act out the surgical experience using dolls and medical equipment would ease anxiety and give the nurse an opportunity to clarify the child's misconceptions. Preschoolers have a limited concept of time, so the nurse should provide preoperative teaching just before surgery rather than starting it as soon as possible; also, a delay between teaching and surgery may heighten anxiety by giving the child a chance to worry or fantasize. The nurse should avoid using such phrases as "put to sleep" because these may have a dual or negative meaning to a young child. Long explanations are inappropriate for the preschooler's developmental level and may increase anxiety.

A preschool-age child scheduled for surgery in the morning is admitted to the facility for the first time. Which nursing action would ease the child's anxiety? 1. Beginning preoperative teaching as soon as possible 2. Explaining that the child will be "put to sleep" during the operation and will feel nothing 3. Having the child act out the surgical experience using dolls and medical equipment 4. Explaining preoperative and postoperative procedures step by step

4. Frequent swallowing RATIONALE: Frequent swallowing — an attempt to clear the throat of trickling blood — suggests postoperative hemorrhage. Emesis may be brown or blood-tinged after a tonsillectomy; only bright red emesis signals hemorrhage. The child may refuse fluids because of painful swallowing, not bleeding. Hemorrhage is associated with an increased, not decreased, heart rate.

A preschool-age child underwent a tonsillectomy 4 hours ago. Which assessment finding should make the nurse suspect postoperative hemorrhage? 1. Vomiting of dark brown emesis 2. Refusal to drink clear fluids 3. Decreased heart rate 4. Frequent swallowing

1. Providing fluids RATIONALE: During a vaso-occlusive crisis, sickle-shaped red blood cells (RBCs) clump together and obstruct blood vessels, causing ischemia and tissue damage. Therefore, the highest priority is providing I.V. and oral fluids, which promotes hemodilution and aids the free flow of RBCs through blood vessels. The client must be kept away from known infection sources but doesn't require protective isolation. Warm compresses may be applied to painful joints to promote comfort; cool compresses would cause vasoconstriction, which exacerbates sickling. Antipyretics may be administered to reduce fever but don't play a crucial role in resolving the crisis.

A preschool-age child with sickle cell anemia is admitted to the health care facility in vaso-occlusive crisis after developing a fever and joint pain. What is the nurse's highest priority when caring for this child? 1. Providing fluids 2. Maintaining protective isolation 3. Applying cool compresses to affected joints 4. Administering antipyretics as ordered

2. meet physiologic needs. RATIONALE: A child with vomiting and diarrhea loses excessive fluids and electrolytes, which must be replaced. Fluid and electrolyte replacement can't eliminate the cause of diarrhea, which may result from various factors. Administration of I.V. fluids that contain glucose (such as dextrose 5% in water) may induce, not prevent, hyperglycemia. Fluid and electrolyte replacement has no effect on stool elimination.

A preschooler has vomiting, diarrhea, and a potassium level of 3 mEq/L. The physician orders an I.V. infusion of 500 ml of dextrose 5% in water and half-normal saline solution with 20 mEq of potassium chloride. The nurse knows that a child with vomiting and diarrhea needs fluids and potassium chloride to: 1. eliminate the cause of diarrhea. 2. meet physiologic needs. 3. avoid hyperglycemia. 4. promote normal stool elimination.

RATIONALE: A Wilms' tumor, also known as a nephroblastoma, is a tumor located on the kidney. It's most commonly found in children ages 2 to 4.

A preschooler is scheduled to have a Wilms' tumor removed. Identify the area of the urinary system where a Wilms' tumor is located.

1. The 2-year-old child who has started eating soft, solid foods following a tonsillectomy RATIONALE: The nurse can delegate care of the child who had the tonsillectomy to the LPN because he is stable and likely preparing for discharge to home. The infant with a WBC count of 34/μl and fever requires close monitoring for additional signs of infection. Infection could lead to sepsis or septic shock. Although the infant with contusions from the motor vehicle accident may be stable, children sometimes experience delayed reactions to injury. This infant requires close monitoring for signs or injury or shock. The RN should care for the infant with type 1 diabetes, who could become ill very quickly.

A registered nurse (RN) has been "care-paired" with a licensed practical nurse (LPN) during the evening shift. Whose care should the RN assign to the LPN? 1. The 2-year-old child who has started eating soft, solid foods following a tonsillectomy 2. A 12-month-old infant who has a white blood cell (WBC) count of 34/μl and a fever 3. A 17-month-old infant with a contusion as a result of a motor vehicle accident 4 hours earlier 4. A 22-month-old infant with type 1 diabetes who has a blood glucose level of 277 g/dl

1. A 2-year-old child who nearly drowned 2 days earlier RATIONALE: The nurse can delegate care of the near-drowning victim to an LPN. Children recover quite quickly from near-drowning experiences; acute care isn't necessary. The infant who has undergone surgery is still under the effects of anesthesia and requires close observation for dehydration, pain, and signs of adverse reactions. The infant with gastroenteritis also requires close monitoring for signs of dehydration. The infant who lost consciousness will need to be monitored most closely. His status could quickly become very critical.

A registered nurse (RN) has been paired with a licensed practical nurse (LPN) for the shift. Whose care should the RN delegate to the LPN? 1. A 2-year-old child who nearly drowned 2 days earlier 2. A 19-month-old infant who had surgery for a fractured tibia 12 hours ago 3. A 6-month-old infant who has gastroenteritis and vomits every 30 minutes 4. A 17-month-old infant who lost consciousness 2 hours earlier because of a head injury

4. Vesicular lesions that ooze, forming crusts on the face and extremities RATIONALE: Impetigo starts as papulovesicular lesions surrounded by redness. The lesions become purulent and begin to ooze, forming crusts. Impetigo occurs most commonly on the face and extremities. Small red lesions on the trunk and in the skin folds are characteristic of scarlet fever. A discrete pink-red maculopapular rash that starts on the face and progresses down to the trunk and extremities is characteristic of rubella (German measles). Red spots with a blue base found on the buccal membranes, known as Koplik's spots, are characteristic of measles (rubeola).

A school nurse is examining a student at an elementary school. Which findings would lead the nurse to suspect impetigo? 1. Small, red lesions on the trunk and in the skin folds 2. A discrete pink-red maculopapular rash that starts on the head and progresses down the body 3. Red spots with a blue base found on the buccal membranes 4. Vesicular lesions that ooze, forming crusts on the face and extremities

4. Examine skin once per month, looking for suspicious lesions or changes in moles. RATIONALE: To detect skin cancer in its early stages, the nurse should emphasize the importance of monthly skin self-examinations and yearly examinations by a physician. To reduce the risk of skin cancer, the nurse should teach clients to avoid the sun's ultraviolet rays between 10 a.m. and 3 p.m. Repeated exposure to artificial sources of ultraviolet radiation, such as tanning booths, increases the risk of skin cancer. Although protective clothing offers some protection, some of the sun's harmful rays can penetrate clothing.

A school nurse is planning a program about skin cancer prevention for a group of teenagers. Which instruction should the nurse emphasize in her talk? 1. Stay out of the sun between 1 p.m. and 3 p.m. 2. Tanning booths are a safe alternative sun exposure for those who wish to tan. 3. Sun exposure is safe, provided the client wears protective clothing. 4. Examine skin once per month, looking for suspicious lesions or changes in moles.

2. Ease the child to the floor and turn him on his side. RATIONALE: Because the child is standing, he should first be eased to the floor and turned to the side to prevent aspiration. Notifying the physician wouldn't be the first action the nurse would take because the child's safety is of primary importance. Diazepam would be administered only if it had been ordered. Notifying the parents, although important, isn't the priority. They can be informed after the seizure is over.

A school-age child begins to have a seizure while walking to the bathroom. What should the nurse do first? 1. Call the physician caring for the child. 2. Ease the child to the floor and turn him on his side. 3. Administer diazepam (Valium) through the I.V. tubing. 4. Notify the parents so they can be with their child.

1. ½ to 1 hour RATIONALE: Regular insulin, a rapid-acting insulin, begins to act in ½ to 1 hour, reaches peak concentration levels in 2 to 10 hours, and has a duration of action of 5 to 15 hours.

A school-age child experiences symptoms of excessive polyphagia, polyuria, and weight loss. The physician diagnoses type 1 diabetes and admits the child to the facility for insulin regulation. The physician orders an insulin regimen of insulin (Humulin R) and isophane insulin (Humulin N) administered subcutaneously. How soon after administration can the nurse expect the regular insulin to begin to act? 1. ½ to 1 hour 2. 1 to 2 hours 3. 4 to 8 hours 4. 8 to 10 hours

2. "Has your child had strep throat recently?" RATIONALE: Asking if the child had strep throat recently is appropriate because group A streptococcal infection typically precedes rheumatic fever — an inflammatory disease that affects the heart, joints, and central nervous system. Rheumatic fever isn't infectious and can't be transmitted from one person to another. Congenital heart defects don't play a role in the development of rheumatic fever. H. influenzae vaccine doesn't prevent streptococcal infection or rheumatic fever.

A school-age child has a fever, joint inflammation, and a nonpruritic rash. Knowing that these are signs of rheumatic fever, the nurse should ask the parents: 1. "Has your child recently been exposed to other children with rheumatic fever?" 2. "Has your child had strep throat recently?" 3. "Does your child have a congenital heart defect?" 4. "Is your child's Haemophilus influenzae vaccine up to date?"

4. Practicing thorough hand washing RATIONALE: Both ALL and its treatment cause immunosuppression. Therefore, thorough hand washing is the single most effective way to prevent infection in an immunosuppressed client. Reverse isolation doesn't significantly reduce the incidence of infection in immunosuppressed clients; furthermore, isolation may cause psychological stress. Standard precautions are intended mainly to protect caregivers from contact with infectious matter, not to reduce the client's risk of infection. Staff and others needn't wear masks when visiting because most infections are transmitted by direct contact. Instead of relying on masks and other barrier methods, the nurse should keep persons with known infections out of the client's room.

A school-age child is admitted to the facility with a diagnosis of acute lymphocytic leukemia (ALL). The nurse formulates a nursing diagnosis of Risk for infection. What is the most effective way for the nurse to reduce the child's risk of infection? 1. Implementing reverse isolation 2. Maintaining standard precautions 3. Requiring staff and visitors to wear masks 4. Practicing thorough hand washing

1. The child should stay on penicillin and return for a follow-up appointment. RATIONALE: A child with rheumatic fever, which is caused by group A beta-hemolytic streptococci, should stay on penicillin — either oral daily or an injection monthly — to prevent a recurrence. A follow-up appointment is needed to determine how the child is responding to treatment. Neither bed rest nor monthly blood tests will be ordered for all children. Rheumatic fever is caused by group A beta-hemolytic streptococci, so the source of the infection is already known.

A school-age child is being discharged with a diagnosis of rheumatic fever. Which instructions should be included in the teaching plan for the family? 1. The child should stay on penicillin and return for a follow-up appointment. 2. At home, be sure to keep the child on bed rest. 3. All children with rheumatic fever need monthly blood tests. 4. The child should stay out of school until the source of the infection is determined.

3. Telling the child in the next room, who also suffered abuse, so the two children can talk to each other RATIONALE: Children have a right to privacy and confidentiality when it comes to their medical condition, treatment plans, and even the fact that they are hospitalized. Therefore, telling another child about the abuse (even if they have that in common) is a breach of confidentiality. A nurse is required by law to report suspected child abuse to the proper local authorities. The attending physician is part of the health care team and needs to be informed about the suspected abuse. These actions don't breach the child's right to confidentiality.

A school-age child reveals to the nurse that his father has been abusing him. What constitutes a breach of the child's right to confidentiality? 1. Telling the child you're required by law to report the abuse 2. Informing the child's attending physician about the conversation 3. Telling the child in the next room, who also suffered abuse, so the two children can talk to each other 4. Informing local authorities and reporting the case

4. Early detection and treatment of streptococcal infections RATIONALE: Rheumatic fever is a systemic inflammatory disease that follows a group A streptococcal infection. Therefore, early detection and treatment of streptococcal infections help prevent the development of rheumatic fever. Hepatitis B vaccine provides immunity against the hepatitis B virus — not streptococci. Because rheumatic fever isn't contagious, isolation measures aren't necessary. Prophylactic antibiotics are used before invasive procedures only in clients with a history of carditis to prevent bacterial endocarditis.

A school-age child with fever and joint pain has just received a diagnosis of rheumatic fever. The child's parents ask the nurse whether anything could have prevented this disorder. Which intervention is effective in preventing rheumatic fever? 1. Immunization with the hepatitis B vaccine 2. Isolation of individuals with rheumatic fever 3. Use of prophylactic antibiotics for invasive procedures 4. Early detection and treatment of streptococcal infections

1. "I will keep the child in light clothing." RATIONALE: Evidence-based practice recommends keeping a child with a fever in cool clothing and a comfortable environment. Therefore, the mother exhibits understanding by saying she will keep the child in light clothing. A child with a fever needs increased fluids and a proper diet. It isn't necessary to take the child with a temperature of 103° F to the ED. The current recommendation is to call the child's physician and then go to the ED if the child has a temperature greater than 105° F (40.5° C). Acetaminophen should be given every 4 hours and ibuprofen every 6 to 8 hours to prevent hepatotoxicity. Giving the child ibuprofen 2 hours after acetaminophen would be too soon according to these guidelines.

A small child is admitted to the facility with a fever. Which statement made by the child's mother indicates understanding of the nurse's teaching? 1. "I will keep the child in light clothing." 2. "I will starve a fever and feed a cold." 3. "I should bring the child back to the emergency department (ED) if his temperature reaches 103° F (39.4° C)." 4. "If acetaminophen doesn't reduce the fever, I can give Motrin in 2 hours."

2. a 9-year-old child receiving subcutaneous (subQ) insulin for treatment of diabetes mellitus. RATIONALE: The nurse-manager should assign the LPN to the child with diabetes mellitus. Because he's receiving subQ insulin rather than I.V. insulin, his diabetes is likely stable. Reye's syndrome is an acute condition with the potential to progress into respiratory depression, seizures, loss of deep tendon reflexes, or other neurologic deficits. This child will require frequent nursing assessments. The child who had a tonsillectomy remains at risk for hemorrhage during the first 24 hours following surgery. Legg-Calve'-Perthes Disease is associated with impaired circulation to the femoral capital epiphysis. This condition requires aggressive monitoring.

A staffing agency is sending a licensed practical nurse (LPN) to cover a shift for a pediatric nurse who called out sick. The unit's nurse-manager isn't familiar with the LPN's clinical background or comfort level with pediatric clients. The nurse-manager should assign the LPN to: 1. an 8-year-old child admitted that morning with suspected Reye's syndrome. 2. a 9-year-old child receiving subcutaneous (subQ) insulin for treatment of diabetes mellitus. 3. a 10-year-old child who had a tonsillectomy that morning. 4. a 9-year-old child with Legg-Calve'-Perthes disease.

4. thrombocytopenia. RATIONALE: A child with thrombocytopenia or neutropenia shouldn't receive rectal medication because of the increased risk of infection and bleeding that may result from tissue trauma. No contraindications exist for administering rectal medication to a child with sepsis, leukocytosis, or anemia.

A toddler has a temperature above 101° F (38.3° C). The physician orders acetaminophen (Tylenol), 120 mg suppository, to be administered rectally every 4 to 6 hours. The nurse should question an order to administer the medication rectally if the child has a diagnosis of: 1. sepsis. 2. leukocytosis. 3. anemia. 4. thrombocytopenia.

4. Performing postural drainage RATIONALE: The child with cystic fibrosis is at risk for frequent respiratory infections secondary to increased viscosity of mucus gland secretions. To help prevent respiratory infections, caregivers must perform postural drainage several times daily to loosen and drain secretions. Because exocrine gland dysfunction, not an allergic response, causes bronchial obstruction in cystic fibrosis, allergy-proofing the home isn't necessary. Oxygen therapy may be indicated, but only during acute disease episodes. Also, such therapy must be supervised closely; home oxygen therapy is inappropriate because chronic hypoxemia poses the risk of oxygen toxicity. If steatorrhea can't be controlled, the child should reduce, but not eliminate, dietary fat intake.

A toddler is admitted to the facility for treatment of a severe respiratory infection. The child's recent history includes fatty stools and failure to gain weight steadily. The physician diagnoses cystic fibrosis. By the time of the child's discharge, the child's parents must be able to perform which task independently? 1. Allergy-proofing the home 2. Maintaining the child in an oxygen tent 3. Maintaining the child on a fat-free diet 4. Performing postural drainage

1. Proteinuria RATIONALE: In nephrotic syndrome, the glomerular membrane of the kidneys becomes permeable to proteins, resulting in massive proteinuria. Nephrotic syndrome typically doesn't cause glycosuria or ketonuria. Because the syndrome causes fluids to shift from plasma to interstitial spaces, it's more likely to decrease urine output than to cause polyuria (excessive urine output).

A toddler is admitted to the facility with nephrotic syndrome. The nurse carefully monitors the toddler's fluid intake and output and checks urine specimens regularly with a reagent strip (Labstix). Which finding is the nurse most likely to see? 1. Proteinuria 2. Glycosuria 3. Ketonuria 4. Polyuria

1. The attending physician RATIONALE: The child's physician is legally responsible for obtaining consent and making sure the parents are well informed. This step includes telling the parents why the child needs the procedure, providing accurate information about the procedure, and explaining the risks involved. The floor nurse may serve as a witness to the parent's signature, and is obligated to inform the physician if the parent doesn't seem informed. The operating room nurse must make sure that the informed consent form has been signed; however, it isn't her responsibility to obtain the consent. Nursing students aren't legally allowed to obtain consent, nor should they act as witnesses.

A toddler is being prepared for surgery. Who is responsible for obtaining informed consent? 1. The attending physician 2. The floor nurse 3. The operating room nurse 4. The nursing student

1. a barium enema. RATIONALE: A nurse should expect the physician to order a barium enema because this test is commonly used to confirm and correct intussusception. Performing a suprapubic aspiration or inserting an NG tube or an indwelling urinary catheter wouldn't help diagnose or treat this disorder.

A toddler is brought to the emergency department with sudden onset of abdominal pain, vomiting, and stools that look like red currant jelly. To confirm intussusception, the suspected cause of these findings, the nurse expects the physician to order: 1. a barium enema. 2. suprapubic aspiration. 3. nasogastric (NG) tube insertion. 4. indwelling urinary catheter insertion.

4. Not crying when moved RATIONALE: Not crying when moved most strongly suggests child abuse because a victim of child abuse typically doesn't complain of pain, even with obvious injuries, for fear of further displeasing the abuser. Trying to sit up on the stretcher is a typical response. Trying to move away from the nurse indicates fear of strangers, which is normal in a toddler. Difficulty answering the nurse's questions is expected in a toddler because of poorly developed cognitive skills.

A toddler is diagnosed with a dislocated right shoulder and a simple fracture of the right humerus. Which behavior suggests that the child's injuries stem from abuse? 1. Trying to sit up on the stretcher 2. Trying to move away from the nurse 3. Not answering the nurse's questions 4. Not crying when moved

2. "Give the elixir with water or juice." RATIONALE: Because iron preparations may stain the teeth, the nurse should instruct the parents to give the elixir with water or juice. The iron preparation shouldn't be given with milk because milk impedes iron absorption. This preparation may darken the stools and cause constipation, not diarrhea; parental instruction regarding increased fluid intake and fiber intake can relieve constipation. To prevent GI upset, the nurse should instruct the parents to mix the iron preparation with water or fruit juice and have the child take it with, not before, meals. (Giving it with fruit juice may be preferable because vitamin C enhances iron solubility and absorption.)

A toddler is diagnosed with iron deficiency anemia. When teaching the parents about using supplemental iron elixir, the nurse should provide which instruction? 1. "Give the iron preparation with milk." 2. "Give the elixir with water or juice." 3. "Monitor the child for episodes of diarrhea." 4. "Give the iron preparation before meals."

4. The parents should be encouraged to visit frequently and should be welcomed by the staff. RATIONALE: Abusive parents should be encouraged to visit their child frequently and should be welcomed by the staff. Many abusive parents love their children but lack effective parenting skills. The child's hospitalization offers an opportunity for the staff to demonstrate appropriate parenting behaviors to the parents.

A toddler is hospitalized for treatment of injuries that the staff believes were caused by child abuse. A staff member states that the parents "shouldn't be allowed to visit because they caused the child's injuries." When responding to this staff member, the nurse should base the comments on which understanding? 1. The parents shouldn't be allowed to visit the child. 2. The parents shouldn't visit until the child is ready for discharge. 3. The parents should visit on a schedule established by the health care team and should be supervised during visits. 4. The parents should be encouraged to visit frequently and should be welcomed by the staff.

2. Report the incident to the proper authorities. RATIONALE: Reporting the incident to the proper authorities should be done first because the nurse is required by law to report all incidents of suspected child abuse. When the appropriate authorities have been notified, the child can be placed under protective custody. Later, the nurse may need to prepare the child for foster care placement and refer the parent to a support group. After reporting suspected abuse, the nurse should allow the parent to visit and help care for the child; during these visits, the nurse should exhibit and reinforce positive parenting behaviors.

A toddler is hospitalized with multiple injuries. Although the parent states that the child fell down the stairs, the child's history and physical findings suggest abuse as the cause of the injuries. What should the nurse do first? 1. Refer the parent to a support group such as Parents Anonymous. 2. Report the incident to the proper authorities. 3. Prepare the child for foster care placement. 4. Restrict the parent from the child's room.

4. Performing the tongue-jaw lift and removing the foreign object only if it's visible. RATIONALE: When checking for a foreign object in the airway of a child younger than age 8, the rescuer should perform the tongue-jaw lift and remove the object only if it's visible. Neck hyperextension may occlude the airway; the head tilt/chin lift method is the correct way to open the airway. After checking for a foreign object, the rescuer should open the airway and attempt to deliver two rescue breaths. A blind finger sweep is contraindicated because it may push the object into the airway. Abdominal thrusts (the Heimlich maneuver) are indicated only for children older than age 1. In a child younger than age 1, such thrusts may injure the abdominal organs; back blows and chest thrusts should be used instead.

A toddler requires emergency intervention for an obstructed airway. Which nursing intervention is appropriate? 1. Hyperextending the child's neck to open the airway and delivering four rescue breaths 2. Placing the child on the side and using a blind finger sweep to remove the foreign object 3. Delivering five back blows followed by five chest thrusts 4. Performing the tongue-jaw lift and removing the foreign object only if it's visible.

4. Respiratory isolation RATIONALE: Because bacterial meningitis is transmitted by droplets from the nasopharynx, the nurse should prepare to use respiratory isolation. This type of isolation involves wearing a gown and gloves during direct client care and ensuring that everyone who enters the child's room wears a mask. Reverse isolation is unnecessary because it's used for immunosuppressed clients who are at high risk for acquiring infection. Strict hand washing and standard precautions are insufficient for this client because they don't require the use of a mask.

A toddler with bacterial meningitis is admitted to the inpatient unit. Which infection control measure should the nurse be prepared to use? 1. Reverse isolation 2. Strict hand washing 3. Standard precautions 4. Respiratory isolation

3. Frequently assessing the child's level of consciousness (LOC) RATIONALE: In hemophilia, one of the factors required for blood clotting is absent, significantly increasing the risk of hemorrhage after injury. Therefore, the nurse must assess the child frequently for signs and symptoms of intracranial bleeding, such as an altered LOC, slurred speech, vomiting, and headache. To manage hemophilia, the absent blood clotting factor is replaced via I.V. infusion of factor, cryoprecipitate, or fresh frozen plasma; this may be done prophylactically or after a traumatic injury. Platelet transfusions aren't necessary. Clients with hemophilia aren't at increased risk for infection. Discussing a safe play environment with the parents is important but isn't the highest priority.

A toddler with hemophilia is hospitalized with multiple injuries after falling off a sliding board. X-rays reveal no bone fractures. When caring for the child, what is the nurse's highest priority? 1. Administering platelets as ordered 2. Taking measures to prevent infection 3. Frequently assessing the child's level of consciousness (LOC) 4. Discussing a safe play environment with the parents

3. Parents' expression of feelings of inadequacy in providing for their child's needs RATIONALE: Expression of feelings of inadequacy in providing for their child's needs is a defining characteristic of Parental role conflict related to child's hospitalization. Supportive child-parent interaction, parents' active participation in the child's care, and evidence of adaptation to parental role changes don't suggest this diagnosis.

After assessing a newly admitted 5-year-old child, the nurse makes the nursing diagnosis of Parental role conflict related to child's hospitalization. Which defining characteristic suggests this diagnosis? 1. Supportive child-parent interaction (speaking, listening, touching, and eye-to-eye contact) 2. Parents' active participation in child's physical or emotional care 3. Parents' expression of feelings of inadequacy in providing for their child's needs 4. Evidence of adaptation to parental role changes

4. Liver RATIONALE: Phenytoin is metabolized in the liver. The pancreas isn't involved in the pharmacokinetic activity of phenytoin. The stomach absorbs orally administered phenytoin, which is excreted by the kidneys in the urine.

After a series of tests, a 6-year-old client weighing 50 lb (22.7 kg) is diagnosed with complex partial seizures. The physician orders phenytoin (Dilantin), 125 mg by mouth twice per day. After the nurse administers phenytoin, where is the drug metabolized? 1. Pancreas 2. Kidneys 3. Stomach 4. Liver

1. Removing the restraints every 2 hours RATIONALE: Removing one elbow restraint at a time every 2 hours for about 5 minutes allows exercise of the arms and inspection for skin irritation. To prevent the infant from touching and disrupting the suture line, the nurse should use the restraints when the infant is asleep and awake. The nurse should maintain the elbow restraints from the time the infant recovers from anesthesia until the suture line is healed.

After an infant undergoes surgical repair of a cleft lip, the physician orders elbow restraints. For this infant, the postoperative care plan should include which nursing action? 1. Removing the restraints every 2 hours 2. Removing the restraints while the infant is asleep 3. Keeping the restraints on both arms only while the child is awake 4. Using the restraints until the infant recovers fully from anesthesia

1. Provide extra oxygen by using a ventilator or through manual bagging. RATIONALE: Providing extra oxygen before suctioning is the first step because it helps prevent hypoxemia. Insertion of a suction catheter is performed after preoxygenation. Instilling a few drops of sterile saline solution is no longer part of routine suctioning. ET and tracheal suctioning require sterile technique and sterile gloves, not just clean gloves.

After gathering all necessary equipment and setting up the supplies, what should be the first step in performing endotracheal (ET) or tracheal suctioning in an infant? 1. Provide extra oxygen by using a ventilator or through manual bagging. 2. Insert a suction catheter to the appropriate measured length. 3. Insert a few drops of sterile saline solution. 4. Put on clean gloves.

4. Gown, gloves, mask, and eye goggles or eye shield RATIONALE: The transmission of SARS isn't fully understood. Therefore, all modes of transmission must be considered possible, including airborne, droplet, and direct contact with the virus. For protection from contracting SARS, any health care worker providing care for a person with SARS should wear a gown, gloves, mask, and eye goggles or an eye shield.

An 11-year-old child contracted severe acute respiratory syndrome (SARS) when traveling abroad with her parents. The nurse knows she must put on personal protective equipment to protect herself while providing care. Based on the mode of SARS transmission, which personal protective equipment should the nurse wear? 1. Gloves 2. Gown and gloves 3. Gown, gloves, and mask 4. Gown, gloves, mask, and eye goggles or eye shield

4. "About 3 to 5 years." RATIONALE: Most children with scoliosis must wear a brace until the spine matures — typically between ages 14 and 16. Therefore, this 11-year-old child will need to wear the brace for 3 to 5 years.

An 11-year-old child is diagnosed with scoliosis and scheduled for brace application. The mother asks the nurse how long her child will have to wear the brace. How should the nurse respond? 1. "About 6 to 8 weeks." 2. "About 6 months." 3. "About 1 to 2 years." 4. "About 3 to 5 years."

3. "Do you like yourself physically?" RATIONALE: Role and relationship patterns focus on body image and the client's relationship with others, which commonly interrelate with food intake. Therefore, asking the adolescent whether she likes herself physically is appropriate. Questions about activities and food preferences elicit information about health promotion and health protection behaviors, not role and relationship patterns. Questions about food allergies elicit information about health and illness patterns.

An 16-year-old girl is brought to the clinic for evaluation for a suspected eating disorder. To best assess the effects of role and relationship patterns on her nutritional intake, the nurse should ask: 1. "What activities do you engage in during the day?" 2. "Do you have any allergies to foods?" 3. "Do you like yourself physically?" 4. "What kinds of foods do you like to eat?"

2. Playing with a pounding board RATIONALE: Playing with a pounding board is a developmentally appropriate diversional activity for a toddler because it not only promotes physical development but also provides an acceptable energy outlet during immobilization. A child younger than age 3 accidentally may swallow Tinker toys and other toys with small parts. Whereas a pull toy is appropriate for a toddler, it isn't appropriate for one who's immobilized. Playing board games is too advanced for a toddler's developmental stage.

An 18-month-old child immobilized with traction to the legs has a nursing diagnosis of Deficient diversional activity related to immobility. Which diversional activity is most appropriate for the nurse to include in the care plan? 1. Playing with Tinker toys 2. Playing with a pounding board 3. Playing with a pull toy 4. Playing board games

4. Have the parents stay with the child and participate in his care. RATIONALE: Allowing the parents to stay and participate in the child's care can provide support to both the parents and the child. Asking the parents not to visit, asking the physician to explain why the child needs to stay, and telling the child to act like an adult won't address the child's diagnosis and may exacerbate the problem.

An 8-year-old child enters a health care facility. During assessment, the nurse discovers that the child is experiencing the anxiety of separation from his parents. The nurse makes the nursing diagnosis of Fear related to separation from familiar environment and family. Which nursing intervention is likely to help the child cope with fear and separation? 1. Ask the parents not to visit the child until he is adjusted to the new environment. 2. Ask the physician to explain to the child why he needs to stay in the health care facility. 3. Tell the child that he must act like an adult while he's in the facility. 4. Have the parents stay with the child and participate in his care.

2. appropriately sedated. RATIONALE: Moderate sedation is an induced state of depressed consciousness. While under moderate sedation, the child should maintain protective reflexes (such as the gag reflex), maintain a patent airway independently, and respond to physical stimuli or verbal commands such as, "Open your eyes." In this scenario, the nurse assesses that the child is under moderate sedation. An undersedated child would likely be anxious and would complain of pain. In deep sedation, the child isn't as easily aroused and doesn't have protective reflexes or the ability to maintain a patent airway; this type of sedation is closer to general anesthesia. With oversedation, the child is difficult to rouse; however, he is able to maintain a patent airway independently.

An 8-year-old child is receiving moderate sedation for a medical procedure. The nurse is assessing the child's level of sedation. His gag reflex is intact, he's breathing comfortably on his own, and he opens his eyes on verbal request. The nurse recognizes that the child is: 1. undersedated. 2. appropriately sedated. 3. deeply sedated. 4. oversedated.

3. Explore the child's knowledge of the procedure and his prior experiences with surgery. RATIONALE: By exploring the child's knowledge of the procedure and his prior experiences with surgery, the nurse may be better able to identify the etiology of his feelings about the procedure. Children can't provide informed consent; parents or guardians do so. Explaining the surgical procedure in detail and informing the child that he could die if he doesn't have the surgery would probably make him more fearful. Telling the child that other children have had the surgery and have done well offers false reassurance.

An 8-year-old child is refusing to have a scheduled appendectomy even though his parents have given informed consent for the surgery. Which action is most appropriate for the nurse to take? 1. Cancel the surgery until the child gives informed consent. 2. Explain the surgery in detail, telling the child that he might die if he doesn't have the operation. 3. Explore the child's knowledge of the procedure and his prior experiences with surgery. 4. Assure the child that other children have had the surgery and have done very well postoperatively.

3. Pediatric pain specialist RATIONALE: Children and adolescents hospitalized with sickle cell crisis are commonly in excruciating pain. Therefore, the pediatric pain specialist should be consulted first to help relieve the adolescent's pain. The adolescent also requires hydration with I.V. fluids, but consulting a nutritionist isn't important at this time. Bed rest is commonly ordered to minimize energy expenditure and oxygen demand; therefore, consulting a physical therapist isn't necessary at this time. It isn't necessary to consult the case manager first; pain relief is most important at this time.

An adolescent admitted to the adolescent unit with pain caused by sickle cell crisis. Who should be consulted first about this adolescent's care? 1. Nutritionist 2. Physical therapist 3. Pediatric pain specialist 4. Case manager

3. Leg ulcers RATIONALE: In sickle cell anemia, sickling of red blood cells leads to increased blood viscosity and impaired circulation. Diminished peripheral circulation makes the adolescent or adult with sickle cell anemia susceptible to chronic leg ulcers. In children younger than age 2 who have sickle cell anemia (not adolescents), swelling of the hands and feet (hand-foot syndrome) commonly occurs during a vaso-occlusive crisis as a result of infarction of short tubular bones. Petechiae aren't associated specifically with sickle cell anemia. Hemangiomas, benign tumors of dilated blood vessels, aren't linked to sickle cell anemia.

An adolescent admitted with sickle cell anemia is most at risk for developing which complication? 1. Swelling of the hands and feet 2. Petechiae 3. Leg ulcers 4. Hemangiomas

4. Chronic hypoxia and iron overload RATIONALE: Thalassemia major increases destruction of red blood cells (RBCs), shortens the life span of RBCs, and causes anemia. The body responds by increasing RBC production, but it can't produce adequate numbers of mature cells. This process results in chronic hypoxia. In addition, children with thalassemia major require multiple transfusions of packed RBCs. The combination of excessive RBC destruction and multiple transfusions deposits excess iron that damages organs and tissues. Thalassemia major doesn't place the adolescent at risk for hypertrophy of the thymus or thyroid or polycythemia vera, which involves excessive RBC production that can lead to thrombosis.

An adolescent diagnosed with thalassemia major (Cooley's anemia) is at risk for which condition? 1. Hypertrophy of the thyroid 2. Hypertrophy of the thymus 3. Polycythemia vera and thrombosis 4. Chronic hypoxia and iron overload

2. Have a female health care worker present. RATIONALE: A female health care provider should be present to observe an examination performed by a male health care provider. Leaving the door open and informing the girl's friends about her condition violates her right to privacy and confidentiality. Although the suspected attacker should be kept away from the examination room, having a female health care worker present during the examination best protects the girl's rights.

An adolescent female arrives in the emergency department after a physical assault. How could the male nurse best protect her rights during the physical examination? 1. Leave the door open. 2. Have a female health care worker present. 3. Keep the suspected attacker away from the examination room. 4. Keep the girl's friends (who are waiting in the lounge area) informed of her medical condition.

1. Give him more pain medication to control his pain and suffering. RATIONALE: The adolescent is in severe pain and requires more pain medication. The goal of treatment at this stage of terminal cancer is to make the adolescent as comfortable as possible. Increased tolerance and addiction potential aren't concerns. Strict timing of medication administration doesn't always coincide with an individual's fluctuating pain. The nurse should give the medication even if the adolescent's need for it doesn't match the administration schedule. Pain is what a client says it is; a nurse shouldn't withhold medication or make judgments about a client's pain threshold.

An adolescent in the terminal stage of leukemia cries out for more pain medicine. What is the best action for a nurse to take in caring for this dying adolescent? 1. Give him more pain medication to control his pain and suffering. 2. Withhold pain medication because he may become addicted to it. 3. Maintain a strict medication administration schedule. 4. Withhold medication because the adolescent has a low pain threshold.

2. Respiratory system RATIONALE: The primary concern with petroleum distillate ingestion is its effect on the respiratory system. Aspiration or absorption of petroleum distillates can cause severe chemical pneumonitis and impaired gas exchange. The GI, neurologic, and cardiovascular systems may also be affected if the petroleum contains additives such as pesticides, but the respiratory system is the priority assessment.

An adolescent is brought to the facility by friends after accidentally ingesting gasoline while siphoning it from a car. Based on the nurse's knowledge of petroleum distillates, which system should be the priority assessment? 1. GI system 2. Respiratory system 3. Neurologic system 4. Cardiovascular system

4. Principles of infection control and ergonomics RATIONALE: Properly cleaning the monitoring equipment involves infection control. Properly placing and securing the monitor uses ergonomic principles. The principles of geometry and mathematics aren't relevant to safety.

An infant requires cardiorespiratory monitoring. A nurse must locate and clean the necessary equipment, move it into the infant's room, and secure it to the bedside wall-mounting device. Which principles should a nurse use to complete this task safely? 1. Principles of geometry and mathematics 2. Principles of ergonomics and geometry 3. Principles of sterile technique and mathematics 4. Principles of infection control and ergonomics

2. "I remove white patches from my tongue and cheeks with my toothbrush." RATIONALE: White patches on the tongue and oral mucosa indicate infection; the adolescent should report the patches, not remove them. Using a soft toothbrush is appropriate because it prevents injury to the fragile oral mucosa. Rinsing his mouth every 2 to 4 hours with a nonirritating solution, such as baking soda and water or normal saline solution helps prevent stomatitis. Avoiding commercial mouthwashes is appropriate because they may contain alcohol, which may dry the oral mucosa.

An adolescent is receiving chemotherapy for lymphoma. Which statement by the adolescent supports a nursing diagnosis of Deficient knowledge related to mouth care? 1. "I use a soft toothbrush to clean my teeth." 2. "I remove white patches from my tongue and cheeks with my toothbrush." 3. "I rinse my mouth every 2 to 4 hours with a solution of baking soda and water." 4. "I don't use commercial mouthwashes."

2. show the adolescent to a private examination room. RATIONALE: The nurse should take the client to an examination room to provide privacy. Federal law states that adolescents may obtain treatment for sexually transmitted diseases without parental notification. This adolescent is guaranteed the same confidentiality as older clients. It isn't appropriate for the nurse to ask the adolescent if her parents know she's promiscuous; doing so could undermine the therapeutic relationship.

An adolescent presents to a community clinic for treatment of vulvar lesions associated with Type 2 herpes simplex. The nurse should: 1. call the adolescent's parents and ask permission to treat their daughter. 2. show the adolescent to a private examination room. 3. inform the adolescent that she can't guarantee her confidentiality. 4. ask the adolescent if her parents know she's promiscuous.

1. Lyme disease. RATIONALE: Lyme disease, which results from a tick bite, is characterized by a large round ring with a raised swollen border at the site of the bite. Treatment at this stage can prevent systemic involvement that could lead to cardiac, neurologic, and musculoskeletal symptoms. Cutaneous anthrax is characterized by a skin lesion that originates as a papule, then develops into a depressed area of black eschar. Impetigo is a clustering of vesicles that ooze and form a crust on the skin. Adolescents rarely develop scarlet fever, which is characterized by rough, red pinpoint lesions concentrated on the trunk and in skin folds.

An adolescent presents with a large round ring with a swollen border on his left arm. He states that he often plays football in a field behind the school. The nurse suspects that he has: 1. Lyme disease. 2. anthrax. 3. impetigo. 4. scarlet fever.

3. "I'll call your physician for the order. No one will tell your parents." RATIONALE: Federal laws state that adolescents may be tested for sexually transmitted diseases without their parents' permission. The rules of confidentiality apply to this adolescent; his parents won't be told of his condition unless he agrees. The adolescent doesn't have to speak with a lawyer before the test. HIV can be contracted at any age, even during infancy and childhood.

An adolescent with pneumonia is admitted to the pediatric unit. After his parents leave the unit for the evening, he tells the nurse he may have contracted human immunodeficiency virus (HIV). He wants to be tested, but he doesn't want his parents to know about the test. What should the nurse say? 1. "Sorry, you need a parent's permission for the test." 2. "You'll have to talk with the hospital lawyer." 3. "I'll call your physician for the order. No one will tell your parents." 4. "You're too young to have HIV."

2. Administering multiple doses of insulin RATIONALE: During an adolescent growth spurt, a regimen of multiple insulin doses achieves better control of the blood glucose level because it more closely simulates endogenous insulin release. A single daily dose of insulin wouldn't control his blood glucose level as effectively. Limiting dietary fat intake wouldn't help the body use glucose at the cellular level. An adolescent with type 1 diabetes doesn't produce insulin and therefore can't receive an oral antidiabetic agent instead of insulin.

An adolescent with type 1 diabetes is experiencing a growth spurt. Which treatment approach would be most effective? 1. Administering insulin once per day 2. Administering multiple doses of insulin 3. Limiting dietary fat intake 4. Substituting an oral antidiabetic agent for insulin

4. Delayed sexual maturation RATIONALE: In children and adolescents with ulcerative colitis, frequent diarrhea and poor nutrient absorption from the bowel lead to malnutrition. Nausea, vomiting, and anorexia may further compromise nutritional status. Malnutrition, in turn, may cause growth retardation and delayed sexual maturation. Corticosteroid therapy, which is commonly used to treat ulcerative colitis, may also cause growth retardation and delayed sexual maturation. Jaundice isn't associated with ulcerative colitis. Because this disease causes increased bowel motility, bowel sounds may be hyperactive, not decreased. Perianal lesions are rare in clients with ulcerative colitis.

An adolescent with ulcerative colitis who is taking corticosteroids is at risk for which complication? 1. Jaundice 2. Decreased bowel sounds 3. Perianal lesions 4. Delayed sexual maturation

1. eating a snack before each gymnastics practice. RATIONALE: Because exercise decreases the blood glucose level, the nurse should instruct him to eat a snack before engaging in physical activity to prevent a hypoglycemic episode. Measuring his urine glucose level before each gymnastics practice is incorrect because the urine glucose level doesn't reflect the current blood glucose level. To prevent hypoglycemia, the blood glucose level should be measured before the activity, not after the activity. Increasing his morning dosage of intermediate-acting insulin may lead to hypoglycemia during gymnastics practice; to avoid this condition, the adolescent may need to decrease, not increase, his morning dosage of intermediate-acting insulin.

An adolescent with well-controlled type 1 diabetes has assumed complete management of his disease and wants to participate in gymnastics after school. To ensure safe participation, the nurse should instruct him to adjust his therapeutic regimen by: 1. eating a snack before each gymnastics practice. 2. measuring his urine glucose level before each gymnastics practice. 3. measuring his blood glucose level after each gymnastics practice. 4. increasing his morning dosage of intermediate-acting insulin.

2. Providing small, frequent meals RATIONALE: Clients with ulcerative colitis, an inflammatory bowel disorder (IBD), tolerate small, frequent meals better than a few large meals daily. Eating large amounts of food may exacerbate the abdominal distention, cramps, and nausea IBD typically causes. Frequent meals also provide the additional calories needed to restore nutritional balance. This adolescent doesn't lack digestive enzymes and therefore doesn't need enzyme supplementation. Antibiotics are contraindicated because they may interfere with the actions of other ordered drugs and because ulcerative colitis isn't caused by bacteria. High-fiber foods may irritate the bowel further.

An adolescent, age 14, is hospitalized for nutritional management and drug therapy after experiencing an acute episode of ulcerative colitis. Which nursing intervention is appropriate? 1. Administering digestive enzymes before meals as ordered 2. Providing small, frequent meals 3. Administering antibiotics with meals as ordered 4. Providing high-fiber snacks

4. It occurs more commonly in infants who sleep in the prone position. RATIONALE: More infants who sleep in the prone position are affected by SIDS. Because of the pooling of blood that occurs in the child with SIDS, child abuse is sometimes suspected. Although premature infants are at a higher risk for SIDS, SIDS isn't exclusive to them. No correlation between SIDS and lung disease exists.

An infant arrives at the emergency department in full cardiopulmonary arrest. Efforts at resuscitation fail, and he's pronounced dead. The cause of death is sudden infant death syndrome (SIDS). Which statement regarding the etiology of SIDS is true? 1. It occurs in suspected child abuse cases. 2. It occurs primarily in infants with congenital lung problems. 3. It occurs only in premature infants. 4. It occurs more commonly in infants who sleep in the prone position.

1. ask to see a copy of the advance directive. RATIONALE: In order to have information about how to proceed, the nurse must evaluate the advance directive. Until the nurse evaluates the legitimacy and content of the advance directive, it's inappropriate for her to administer oxygen or provide palliative care. The nurse should ask to see the advanced directive before proceeding with care; contacting the nursing supervisor isn't the most appropriate initial response.

An infant is brought to the emergency department. The infant is limp and has central cyanosis, a heart rate of 60 beats/minute, and a respiratory rate of 12 breaths/minute. The parents state that they have an advance directive for their infant, who has a terminal illness. A nurse's initial action should be to: 1. ask to see a copy of the advance directive. 2. administer oxygen to the infant while awaiting the physician's orders. 3. provide palliative care for the infant and his family. 4. contact the nursing supervisor for assistance.

4. Hip RATIONALE: To assess for Ortolani's sign, the nurse abducts the infant's hips while flexing the legs at the knees. This is performed on all infants to assess for congenital hip dislocation. The examiner listens and feels for a "click" as the femoral head enters the acetabulum during the examination. This finding indicates a congenitally dislocated hip.

An infant is having his 2-month checkup at the pediatrician's office. The physician tells the parents that she's assessing for Ortolani's sign. The nurse explains that the presence of Ortolani's sign indicates dislocation of what joint? 1. Shoulder 2. Elbow 3. Knee 4. Hip

1. Ineffective airway clearance RATIONALE: Ineffective airway clearance has the highest priority in the immediate postoperative period. The infant's airway must be carefully assessed and frequent suctioning may be necessary to remove mucus while taking care not to pass the catheter as far as the suture line. Assess breath sounds, respiratory rate, skin color, and ease of breathing. Because of the risk of edema and airway obstruction, keep a laryngoscope and endotracheal intubation equipment readily available. Imbalanced nutrition, Interrupted breast-feeding, and Hypothermia are also important during the postoperative period but only after a patent airway is ensured.

An infant undergoes surgery to correct an esophageal atresia and tracheoesophageal fistula. Which nursing diagnosis has the highest priority during the first 24 hours postoperatively? 1. Ineffective airway clearance 2. Imbalanced nutrition: Less than body requirements 3. Interrupted breast-feeding 4. Hypothermia

3. Bulging fontanels RATIONALE: Because an infant's fontanels remain open, the skull may expand in response to increased intracranial pressure, a possible postoperative complication. Decreased urine output and sunken eyeballs (signs of dehydration) and a decrease in heart rate are rarely seen as postoperative complications of myelomenigocele removal.

An infant undergoes surgery to remove a myelomeningocele. To detect complications as early as possible, the nurse should stay alert for which postoperative finding? 1. Decreased urine output 2. Increased heart rate 3. Bulging fontanels 4. Sunken eyeballs

1. The foster mother RATIONALE: When children are minors and aren't emancipated, their parents or designated legal guardians are responsible for providing consent for medical procedures. Therefore, the foster mother is authorized to give consent for the blood transfusion. The social worker, the nurse, and the nurse manager have no legal rights to give consent in this scenario.

An infant who has been in foster care since birth requires a blood transfusion. Who is authorized to give written, informed consent for the procedure? 1. The foster mother 2. The social worker who placed the infant in the foster home 3. The registered nurse caring for the infant 4. The nurse manager

2. instruct the mother to place the food at the back and toward the side of the infant's mouth. RATIONALE: The nurse should instruct the mother to place the food at the back and toward the side of the infant's mouth because it encourages swallowing. Tongue thrusting is a physiologic response to food placed incorrectly in the mouth. Offering pureed foods wouldn't encourage swallowing, which is a learned behavior. Force-feeding is inappropriate because it may be frustrating for both the mother and child and may cause the child to gag and choke when attempting to reject the undesired food; also, it may lead to a higher-than-normal caloric intake, resulting in obesity.

An infant, age 10 months, is brought to the well-baby clinic for a follow-up visit. The mother tells the nurse that she has been having trouble feeding her infant solid foods. To help correct this problem, the nurse should: 1. point out that tongue thrusting is the infant's way of rejecting food. 2. instruct the mother to place the food at the back and toward the side of the infant's mouth. 3. advise the mother to puree foods if the child resists them in solid form. 4. suggest that the mother force-feed the child if necessary.

4. Rubber dropper RATIONALE: An infant with a surgically repaired cleft lip must be fed with a rubber dropper or Breck feeder to prevent sucking or suture line trauma. A single-hole nipple, a plastic spoon, and a paper straw wouldn't prevent these actions.

An infant, age 3 months, undergoes surgical repair of a cleft lip. After surgery, the nurse should use which equipment to feed the infant? 1. Single-hole nipple 2. Plastic spoon 3. Paper straw 4. Rubber dropper

2. 14 lb (6.4 kg) RATIONALE: Birth weight typically doubles by age 6 months and triples by age 12 months. Therefore, an infant who weighed 7 lb (3.2 kg) at birth should weigh 14 lb (6.4 kg) at age 6 months.

An infant, age 6 months, is brought to the clinic for a well-baby visit. The mother reports that the infant weighed 7 lb (3.2 kg) at birth. Based on the nurse's knowledge of infant weight gain, which current weight would be within the normal range for this infant? 1. 10.5 lb (4.8 kg) 2. 14 lb (6.4 kg) 3. 17.5 lb (7.9 kg) 4. 21 lb (9.5 kg)

4. Seated upright RATIONALE: For the most accurate results, the nurse should seat the infant upright to assess the fontanels and should perform this assessment when the infant is quiet. Pressure from postural changes or intense crying may cause the fontanels to bulge or seem abnormally tense. When the infant is in a recumbent position, the fontanel is less flat than it is normally, creating the false impression that intracranial pressure is increased.

An infant, age 6 weeks, is brought to the clinic for a well-baby visit. To assess the fontanels, how should the nurse position the infant? 1. Supine 2. Prone 3. In the left lateral position 4. Seated upright

2. To help the girl and the nurse analyze how much food she is eating and to identify the circumstances in which she eats RATIONALE: Keeping a food diary allows this adolescent to use the cognitive level of formal operations to help her identify and evaluate eating behaviors of which she may not be aware. The food diary isn't intended to keep the girl busy and focused on losing weight. She needs to engage in other activities instead of focusing on her diet. Using the food diary to check for cheating represents a punitive approach, which is relatively ineffective. The food diary is primarily for the girl's benefit, although the nurse can use it, too.

An overweight girl, age 15, has lost 12 lb (5.4 kg) in 8 weeks by dieting. Now, after reaching a weight plateau, she has become discouraged. She and the nurse decide she should keep a food diary. What is the primary purpose of keeping such a diary? 1. To help the girl stay busy and more focused on losing weight 2. To help the girl and the nurse analyze how much food she is eating and to identify the circumstances in which she eats 3. To help the nurse and the girl determine whether the the girl has been cheating on her diet 4. To provide a written record for the nurse

4. record facts surrounding each incident. RATIONALE: The main goal of an incident report following an adventitious event isn't punishment for those involved in the incident. The purpose of an incident report is threefold: to identify ways to prevent recurrences of incidents, to identify patterns of care problems, and to identify facts surrounding each incident. An incident report doesn't protect the nurse from a lawsuit.

As an adolescent is receiving care, he's inadvertently injured with a warm compress. The nurse completes an incident report, knowing the report's goal is to: 1. reprimand staff for their actions. 2. protect the nurse from a lawsuit. 3. place the blame on the adolescent. 4. record facts surrounding each incident.

3. A check of the pH of fluid aspirated from the tube RATIONALE: Intestinal, gastric, and respiratory fluids have different pH values. Therefore, checking the pH of fluid aspirated from the tube is the most reliable technique for checking proper NG tube placement without taking X-rays before each feeding. X-rays can't be performed multiple times a day on a daily basis. Because auscultation of air can be heard when the tube is in the esophagus as well as in the stomach, this isn't the best test for checking placement. Observing the insertion measurement mark isn't a good check either because the mark may remain the same even though the tube has migrated up or down into the esophagus, lungs, or intestines.

Before administering a tube feeding to a toddler, which method should the nurse use to check the placement of a nasogastric (NG) tube? 1. Abdominal X-rays 2. Injection of a small amount of air while listening with a stethoscope over the abdominal area 3. A check of the pH of fluid aspirated from the tube 4. Visualization of the measurement mark on the tube made at the time of insertion

4. "The baby's stools are bright yellow and soft." RATIONALE: Breast-fed infants typically have soft, bright yellow or light green stools with no offensive odor. Formula-fed infants typically have pale yellow, semiformed stools with a strong odor. A neonate's first stools typically are dark green to black, sticky, and odorless (representing meconium, usually present for the first 3 days). By the fourth day, yellowish green transitional stools appear. Green, watery stools indicate diarrhea.

During a visit to the clinic, a mother who's breast-feeding her 2-month-old infant expresses concern over the infant's bowel movements. Which statement by the mother would lead the nurse to believe that the infant's bowel movements are normal? 1. "The baby's stools are yellow and semiformed." 2. "The baby's stools are dark green and sticky." 3. "The baby's stools are green and watery." 4. "The baby's stools are bright yellow and soft."

2. "This vaccine protects against serious bacterial infections, such as meningitis and bacterial pneumonia." RATIONALE: The Hib vaccine provides protection against serious childhood infections caused by H. influenzae type B virus, such as meningitis and bacterial pneumonia. The Hib vaccine doesn't prevent infection by the influenza virus, hepatitis B virus, or the varicella virus (chickenpox). The influenza virus vaccine provides immunity to various strains of the influenza virus. The Heptavax vaccine prevents infection by the hepatitis B virus. The varicella vaccine prevents the chickenpox.

During a well-baby visit, a 2-month-old infant receives diphtheria, tetanus toxoids, and acellular pertussis (DTaP) vaccine, inactivated poliovirus vaccine, hepatitis B vaccine, pneumococcal vaccine, and Haemophilus influenzae b (Hib) vaccine. The parents ask why the baby must have the Hib vaccine. How should the nurse respond? 1. "This vaccine prevents infection by various strains of the influenza virus." 2. "This vaccine protects against serious bacterial infections, such as meningitis and bacterial pneumonia." 3. "This vaccine prevents infection by the hepatitis B virus." 4. "This vaccine prevents chickenpox."

2. expiratory wheezing. RATIONALE: Expiratory wheezing is common during an acute asthma attack and results from narrowing of the airway caused by edema. Acute asthma rarely causes apneic periods. Inspiratory stridor more commonly accompanies croup. The child may have some fine crackles but wheezing is much more common in an acute asthma attack.

For a child who's admitted to the emergency department with an acute asthma attack, nursing assessment is most likely to reveal: 1. apneic periods. 2. expiratory wheezing. 3. inspiratory stridor. 4. fine crackles throughout.

3. The child exhibits clear breath sounds. RATIONALE: The nurse should expect clear breath sounds because this outcome indicates an improved respiratory status and airway clearance. A respiratory rate of 44 breaths/minute is high and indicates a respiratory problem. An arterial oxygen saturation of 85% is abnormally low. Decreased, not increased, anxiety would indicate effective airway clearance.

For a child with tracheobronchitis, the nurse formulates a nursing diagnosis of Ineffective airway clearance related to thick secretions. After implementing interventions, the nurse expects which client outcome? 1. The child exhibits a respiratory rate of 44 breaths/minute. 2. The child exhibits an arterial oxygen saturation of 85%. 3. The child exhibits clear breath sounds. 4. The child exhibits increased anxiety.

2. Jack-in-the-box RATIONALE: According to Piaget's theory of cognitive development, an 8-month-old child will look for an object once it disappears from sight to develop the cognitive skill of object permanence. Therefore, a jack-in-the-box would promote cognitive development. Finger paint and small balls are potentially dangerous because infants frequently put their fingers or objects in their mouths. Anything strung across a crib, such as a play gym, is a safety hazard — especially to a child who may use it to pull up to a standing position.

For an 8-month-old infant, which toy promotes cognitive development? 1. Finger paint 2. Jack-in-the-box 3. A small rubber ball 4. A play gym strung across the crib

2. an arched, side-lying position, avoiding flexion of the neck onto the chest. RATIONALE: For a lumbar puncture, the nurse should place the infant in an arched, side-lying position to maximize the space between the third and fifth lumbar vertebrae. The nurse's hands should rest on the back of the infant's shoulders to prevent neck flexion, which could block the airway and cause respiratory arrest. The infant should be placed at the edge of the bed or table during the procedure, and the nurse should speak quietly to calm the infant. A mummy restraint would limit access to the lumbar area because it involves wrapping the child's trunk and extremities snugly in a blanket or towel. A prone position isn't appropriate because it wouldn't cause separation of the vertebral spaces.

For an infant who's about to undergo a lumbar puncture, the nurse should place the infant in: 1. an arched, side-lying position, with the neck flexed onto the chest. 2. an arched, side-lying position, avoiding flexion of the neck onto the chest. 3. a mummy restraint. 4. a prone position, with the head over the edge of the bed.

1. Separation from the family RATIONALE: For infants through preschoolers, separation from the family is the major stressor posed by hospitalization. To minimize the effects of separation, the nurse may suggest that a family member stay with the child as much as possible. Reducing this stressor may help a young child withstand other possible stressors of hospitalization, such as fear of bodily injury, loss of control, and fear of pain.

For children from infancy through the preschool years, what is the major stressor posed by hospitalization? 1. Separation from the family 2. Fear of bodily injury 3. Loss of control 4. Fear of pain

3. the inside of the infant's mouth. RATIONALE: The nurse should pay close attention to the inside of the infant's mouth for white patches. Signs of thrush, these patches are common in children with C. albicans infections and should be reported to the physician. Although the other assessments should be performed as a part of an infant evaluation, they aren't the nurse's primary focus in this situation.

For the last 6 days, a 7-month-old infant has been receiving amoxicillin trihydrate (Amoxil) to treat an ear infection. Now the parents report redness in the diaper area and small, red patches on the infant's inner thighs and buttocks. After diagnosing Candida albicans, the physician orders topical nystatin (Mycostatin) to be applied to the perineum four times daily. The nurse should focus her assessment on: 1. the infant's heart and respiratory rate. 2. the infant's fontanels. 3. the inside of the infant's mouth. 4. the infant's height and weight.

4. immature kidney function. RATIONALE: Because of immature kidneys, an infant's glomerular filtration and absorption are inadequate, not reaching adult levels until age 1 to 2 years. An infant actually has a greater percentage of body water as well as higher daily fluid requirements than an adult. Although the infant's respiratory rate is higher, causing insensible water loss, immature kidney function is more responsible for fluid balance in an infant.

In a 3-month-old infant, fluid and electrolyte imbalance can occur quickly, primarily because an infant has: 1. a lower percentage of body water than an adult. 2. a lower daily fluid requirement than an adult. 3. a more rapid respiratory rate than an adult. 4. immature kidney function.

1. Siblings RATIONALE: When a brother or sister is ill, siblings frequently experience jealousy and resentment of the increased attention given to the ill child, embarrassment and shame, fear of becoming ill, and guilt at causing the illness. Parents may experience grieving, denial, overprotectiveness, rejection, and overcompensation. The ill child may regress to a previous developmental stage and feel anxiety, depression, and anger. Both the child's and the siblings' reactions are influenced by the parents' response. Grandparents may experience ambivalence, disappointment, and grief.

In a family with a 7-year-old child with a chronic illness, which family members feel jealousy, resentment, embarrassment, shame, fear of becoming ill, and guilt at causing the illness? 1. Siblings 2. Parents 3. Child with the illness 4. Grandparents

3. on an empty stomach. RATIONALE: Most oral pediatric medications are administered on an empty stomach. They aren't usually administered with milk or formula because these can affect gastric pH and alter drug absorption. Because a child's meals usually contain milk or a milk product, the nurse wouldn't administer the drugs with meals or even ½ hour after meals.

Most oral pediatric medications are administered: 1. with the nighttime formula. 2. ½ hour after meals. 3. on an empty stomach. 4. with meals.

4. Anemia RATIONALE: The nurse should be concerned about anemia because it isn't a typical adverse effect of morphine. This sign should be investigated if it's discovered during treatment. Constipation, nausea and vomiting, and pruritus are all treatable adverse effects of morphine and don't necessitate discontinuation of the medication.

When administering morphine to a school-age child, which sign or symptom should cause the nurse to be concerned? 1. Constipation 2. Nausea and vomiting 3. Pruritus 4. Anemia

2. Using gestures to express desires RATIONALE: Using gestures instead of verbal communication to express desires — especially in a child older than age 15 months — may indicate a hearing or communication impairment. Stuttering is normal in children ages 2 to 4, especially boys. Continuous babbling is a normal phase of speech development in young children. In fact, its absence, not presence, would be cause for concern. Parallel play — playing alongside peers without interacting — is typical of toddlers. However, in an older child, difficulty interacting with peers or avoiding social situations may indicate a hearing deficit.

Parents of a 2-year-old child with chronic otitis media are concerned that the disorder has affected their child's hearing. Which behavior suggests that the child has a hearing impairment? 1. Stuttering 2. Using gestures to express desires 3. Babbling continuously 4. Playing alongside rather than interacting with peers

1. complementary therapy is an alternative to conventional medical therapies. RATIONALE: The nurse should tell the parents that complementary therapy is a form of alternative medicine. This type of therapy can include diet, exercise, herbal remedies, and prayer. Answering the parents' questions builds rapport and trust. The nurse shouldn't dismiss the parents' idea by telling them complementary therapy wouldn't help their child. The nurse doesn't need to direct the parents to the physician. She can provide the basic information and let the parents determine if they'd like to seek further assistance. Studies indicate that complementary therapies are beneficial to the child and the parents.

Parents of a 4-year-old child with acute leukemia ask a nurse to explain the concept of complementary therapy. The nurse should tell the parents that: 1. complementary therapy is an alternative to conventional medical therapies. 2. complementary therapy wouldn't help their child. 3. the physician should talk with them about it. 4. there's no research that indicates that complementary therapies are effective.

4. Genetic counselor RATIONALE: A genetic counselor can educate the couple about an inherited disorder, as well as screening tests and treatments that can be done; the counselor can also provide emotional support. Clergy are available to provide spiritual support. A social worker can provide emotional support and help with referrals for financial problems. A nurse-midwife cares for women during pregnancy and birth.

Parents of a 4-year-old with sickle cell anemia tell the nurse that they would like to have other children, but they're concerned about passing sickle cell anemia on to them. Which health care team member would be the most appropriate person for the nurse to refer them to? 1. Clergy 2. Social worker 3. Certified nurse-midwife 4. Genetic counselor

3. Typical absence RATIONALE: This child is probably having typical absence seizures. Typical absence seizures have an onset between ages 3 and 12. This type of seizure is exhibited by an abrupt loss of consciousness, amnesia, or unawareness characterized by staring and a 3-cycle/second spike and waveform on an EEG. The attack lasts from 10 to 30 seconds and may occur as frequently as 50 to 100 times a day. No postictal or confused state follows the attack. A complex partial seizure causes a brief impairment of consciousness. A myoclonic seizure occurs in older children and is exhibited by lightning jerks without loss of consciousness. An abrupt increase in muscle tone, loss of consciousness, and marked autonomic signs and symptoms characterize the tonic seizure.

Parents of a 6-year-old tell a physician that the child has been having periods of unawareness with short periods of staring. Based on his history, the child is probably having which type of seizure? 1. Complex partial 2. Myoclonic 3. Typical absence 4. Tonic

4. "At this developmental stage, most children have an adult concept of death and should be encouraged to discuss it." RATIONALE: By age 9 or 10, most children have an adult concept of death. Therefore, caregivers should discuss death with them in terms consistent with their developmental stage. In addition, school-age children respond well to concrete explanations about death and dying. Preschoolers, not school-age children, typically view death as temporary and reversible. While school-age children may fantasize about the unknown aspects of death, these fantasies may actually increase their anxiety. Although a child may fear death, accurate information about death can ease anxiety.

Parents of a 9-year-old child in the terminal phase of a fatal illness ask the nurse for guidance in discussing death with their child. Which response is appropriate? 1. "Children of that age view death as temporary and reversible, which makes it hard to explain." 2. "Children of that age typically fantasize about what dying will be like, which is much better than knowing the truth." 3. "At this developmental stage, children are afraid of death, so it's best not to discuss it with them." 4. "At this developmental stage, most children have an adult concept of death and should be encouraged to discuss it."

4. Not perform the transfusion but provide comfort measures for the child. RATIONALE: Jehovah's Witnesses believe that a blood transfusion is the same as oral intake of blood, which they regard as a sin. The nurse caring for the child shouldn't perform the transfusion, but she should provide comfort measures for the child. It isn't appropriate for the nurse to call social services because this situation is an ethical matter. The nurse shouldn't ask the parents to reconsider their decision because it violates their cultural beliefs, which the nurse should uphold.

Parents of a preschooler are told their child needs a blood transfusion to treat hypovolemia. A nurse contacts a physician with the information that the parent's are Jehovah's Witnesses and refuse to sign the consent form. The physician tells the nurse to perform the transfusion. He states that he isn't going to let the child's parents allow him to die. What should the nurse do next? 1. Contact social services and allow that agency to manage the situation. 2. Perform the blood transfusion as directed by the physician. 3. Inform the boy's parents of the physician's decision and ask them to reconsider. 4. Not perform the transfusion but provide comfort measures for the child.

3. Reye's syndrome RATIONALE: Research shows a correlation between the use of aspirin in children with flulike symptoms and the development of Reye's syndrome (a disorder characterized by brain and liver toxicity). Therefore, the nurse should instruct the parents to avoid administering aspirin or other products that contain salicylates and to consult the physician or pharmacist before administering any medication to a child with chickenpox. No research has found a link between aspirin use, chickenpox, and the development of Guillain-Barré syndrome, rheumatic fever, or scarlet fever.

Parents of a preschooler with chickenpox ask the nurse about measures to make their child comfortable. The nurse instructs the parents to avoid administering aspirin or any other product that contains salicylates. When given to children with chickenpox, aspirin has been linked to which disorder? 1. Guillain-Barré syndrome 2. Rheumatic fever 3. Reye's syndrome 4. Scarlet fever

4. conservation skills. RATIONALE: According to Piaget, a school-age child acquires cognitive operations to understand concepts related to objects, including conservation skills, classification skills, and combinational skills. Magical thinking and transductive reasoning are characteristic of the preschooler's preoperational thought. Abstract thought is characteristic of the adolescent's period of formal operations.

Parents of a school-age child request anticipatory guidance. When developing a care plan to address this matter, the nurse should keep in mind that this child's cognitive development is characterized by: 1. magical thinking. 2. transductive reasoning. 3. abstract thought. 4. conservation skills.

4. Taking prophylactic drugs before the activity can prevent asthma attacks and enable the child to engage in most sports. RATIONALE: Although exercise may trigger asthma attacks, the nurse should tell the parents that taking prophylactic asthma drugs before beginning the activity can prevent attacks, enabling the child to engage in most sports. To say asthma attacks are triggered by allergens, but not exercise, isn't appropriate because asthma attacks may be triggered by various factors, including allergens, exercise, medications, upper respiratory tract infections, and psychological stress. Provided the asthma is under control, most children can participate in sports and other physical activities; in fact, they benefit from exercise. Activity restrictions actually hamper peer interaction, which is essential to the development of the school-age child. A child with asthma may tolerate intermittent activities better than continuous ones.

Parents of a school-age child with asthma express concern about letting the child participate in sports. What should the nurse tell the parents about the relationship between exercise and asthma? 1. Asthma attacks are triggered by allergens, not exercise. 2. The child should avoid exercise because it may trigger asthma attacks. 3. Continuous activities such as jogging are less likely to trigger asthma than intermittent activities such as baseball. 4. Taking prophylactic drugs before the activity can prevent asthma attacks and enable the child to engage in most sports.

1. Increased respiratory rate RATIONALE: Increased respiratory and heart rates are the earliest signs of heart failure. Decreased urine output and increased weight are later signs.

Which assessment finding is an early sign of heart failure in a toddler? 1. Increased respiratory rate 2. Increased urine output 3. Decreased weight 4. Decreased heart rate

4. "We'll read her a story and let her play quietly in her bed until she falls asleep." RATIONALE: The parents stating that they'll read the child a story and let her play quietly demonstrates effective teaching because spending time with the parents and playing quietly are positive bedtime routines that provide security and prepare a child for sleep. Saying that they will let their daughter fall asleep in their room reflects ineffective teaching because the child should sleep in her own bed. Locking the door is frightening and may cause insecurity. Active play before bedtime stimulates the child and increases the time needed to settle down for sleep; therefore, a statement about running games would demonstrate ineffective teaching.

Parents report that their daughter, age 4, resists going to bed at night. After instruction by the nurse, which statement by the parents indicates effective teaching? 1. "We'll let her fall asleep in our room, then move her to her own room." 2. "We'll lock her in her room if she gets up more than once." 3. "We'll play running games with her before bedtime to tire her out, and then she'll fall asleep easily." 4. "We'll read her a story and let her play quietly in her bed until she falls asleep."

1. tell the children not to bite their fingernails. RATIONALE: Pinworms come out of the intestine through the anus at night to lay eggs, causing perianal itching. The child wakes up and may begin scratching. Eggs under the fingernails are carried to the mouth if the child chews on his nails, and the life cycle of the pinworm continues. In addition to teaching children not to bite their fingernails, parents should keep the nails short and encourage hand washing before food preparation and eating. Sharing hairbrushes contributes to the spread of head lice, not pinworms. Although covering the mouth and nose are hygienic practices to reduce the spread of infections from respiratory droplets, doing so doesn't affect the spread of pinworms. There are no immunizations to protect against pinworms.

Several children in a kindergarten class have been treated for pinworm. To prevent the spread of pinworm, the school nurse meets with the parents and explains that they should: 1. tell the children not to bite their fingernails. 2. not let children share hairbrushes. 3. tell the children to cover their mouths and noses when they cough or sneeze. 4. have their children immunized.

1. A nurse who just discharged two clients with newly diagnosed diabetes RATIONALE: Having just discharged two clients, this nurse has a low client load and she's able to accept a new assignment. The client with asthma requires constant monitoring by the nurse until his situation is resolved. Simple tasks and procedures are commonly more time-consuming when clients with paralysis are involved because these clients can't directly aid in their own care. Additional time must also be allotted for the nurse about to undertake a complicated procedure, such as a wet-to-damp dressing change.

The charge nurse on the adolescent unit must decide which nurse should admit a new client. Based on the present client care assignments, who is the best candidate to admit the client? 1. A nurse who just discharged two clients with newly diagnosed diabetes 2. A nurse whose patient with asthma has decreasing oxygen saturation levels 3. A nurse caring for a client who is paralyzed and has no visiting family 4. A nurse who is about to start a complicated wet-to-damp dressing change

3. "Sucking is important to the baby." RATIONALE: Stating that sucking is the infant's chief pleasure indicates effective teaching. However, thumb-sucking may cause malocclusion if it persists after age 4. Many fetuses begin sucking on their fingers in utero and, as infants, refuse a pacifier as a substitute, so the mother who states she'll give the infant a pacifier instead requires more teaching. A young child is likely to chew on a bandage, possibly leading to airway obstruction.

The mother of a 12-month-old child expresses concern about the effects of her child's frequent thumb-sucking. After the nurse provides instruction on this topic, which response by the mother indicates that teaching has been effective? 1. "Thumb-sucking should be discouraged at age 12 months." 2. "I'll give my baby a pacifier instead." 3. "Sucking is important to the baby." 4. "I'll wrap the baby's thumb in a bandage."

4. "Bring your daughter into the emergency department immediately before her appendix has a chance to rupture." RATIONALE: Abdominal pain, low-grade fever, and vomiting are cardinal signs of appendicitis. The nurse should instruct the mother to take the girl to the emergency department. Telling the mother to give the girl a laxative is inappropriate because if appendicitis is the cause of the pain the appendix may rupture as a result of the drug. Appendicitis can occur at any age. Rebound tenderness is a symptom of appendicitis, but this finding would be found in the right lower quadrant, not the left.

The mother of a 16-year-old girl calls the emergency department, suspecting her daughter's abdominal pain may be appendicitis. In addition to pain, her daughter has a temperature of 100° F (37.7° C) and has vomited twice. What should the nurse tell the mother? 1. "Give your daughter a laxative to rule out the possibility that constipation is causing the pain." 2. "Gently press on the lower left quadrant of your daughter's abdomen to test for rebound tenderness." 3. "It's most likely the flu because your daughter is too young to have appendicitis." 4. "Bring your daughter into the emergency department immediately before her appendix has a chance to rupture."

3. is a normal finding in a 6-month-old infant. RATIONALE: The Denver Developmental Screening Test evaluates the developmental level of social, motor, and language skills in children ages 1 month to 6 years. An infant doesn't develop the ability to use a pincer grasp until about 9 months, so the lack of such a grasp in a 6-month-old infant is a normal finding. A neurologic evaluation or more developmental testing isn't indicated.

The nurse is administering the Denver Developmental Screening Test to a 6-month-old infant during a well-baby checkup. She notes that the child is unable to use a pincer grasp. The nurse notes that this finding: 1. suggests the infant needs a neurologic evaluation. 2. indicates the need for further developmental testing. 3. is a normal finding in a 6-month-old infant. 4. indicates the infant is ahead in developmental milestones.

4. Providing age-appropriate reading materials RATIONALE: Because adolescents absorb less information through reading than through demonstration or discussion, providing age-appropriate reading materials is the least effective way to teach parenting skills to an adolescent. The other options engage more than one of the senses and therefore serve as effective teaching strategies.

The nurse is preparing a teaching plan for a 15-year-old adolescent who is 7 months pregnant. The nurse should reevaluate her teaching plan if she includes which teaching strategy? 1. Providing a one-on-one demonstration and requesting a return demonstration, using a live infant model 2. Initiating a teenage-parent support group with first- and second-time mothers 3. Using audiovisual aids that show discussions of feelings and skills 4. Providing age-appropriate reading materials

1. allow adequate time for the nursing assistant to complete the task, then follow-up with her. RATIONALE: The nurse remains accountable for all of the client's care, including tasks that have been delegated to the nursing assistant. The nurse should allow the nursing assistant ample time to complete the task, then follow up with her to make sure she has completed the task. Documentation occurs after the task has been completed satisfactorily. When a task is delegated, it's important to allow team members the authority to complete the assigned task. However, the nurse should follow up with the nursing assistant to make sure she has completed the task satisfactorily; the nurse can't assume that has been done.

The nurse on the adolescent unit delegates a task to the nursing assistant. After delegating the task, the nurse should: 1. allow adequate time for the nursing assistant to complete the task, then follow-up with her. 2. document in the chart that the task has been completed. 3. keep asking the nursing assistant if she has completed the task. 4. assume the nursing assistant has completed the task to her satisfaction.

4. "The child should be aware of the impending surgery so he can develop coping strategies and his questions can be answered." RATIONALE: Advance awareness of the surgery and its significance offers a school-age child time to develop coping strategies and formulate questions. Failure to inform the child about the surgery may result in fear or mistrust of health care workers or the health care system. A school-age child can't give operative consent. Although hospital requirements may require the nurse to inform a child of impending surgery, this response doesn't best reflect the nurse's promotion of the child's rights.

The parents of a 9-year-old child who is scheduled to have surgery ask the nurse not to tell him about the surgery until he's taken to the operating room. Which response best demonstrates the nurse's role in supporting the child's rights? 1. "I agree that the child shouldn't be told about the surgery until it's absolutely necessary to avoid unnecessary stress." 2. "The child should be aware of the impending surgery so he can give informed consent." 3. "I must inform the child because the hospital requires that he be made aware of the surgery." 4. "The child should be aware of the impending surgery so he can develop coping strategies and his questions can be answered."

2. 25% RATIONALE: To manifest, or express, an autosomal recessive disorder, a child must inherit the trait from both parents. A heterozygous person carries one normal gene and one affected gene and doesn't express the disorder. Therefore, a child of two heterozygous parents has a one-in-four (25%) chance of manifesting an autosomal recessive disorder. Also, outcomes of previous pregnancies don't influence the probability of subsequent offspring expressing the genetic disorder.

The parents of a child with cystic fibrosis, an autosomal recessive disorder, are considering having a second child. Each parent is heterozygous for the cystic fibrosis trait. What is the chance that their second child will manifest the disorder? 1. 0% 2. 25% 3. 50% 4. 100%

1. striving to prevent pain by routine administration of pain medication. RATIONALE: When providing comfort measures for a child, the nurse should strive to prevent pain by providing routine pain medication. Although the nurse should administer pain medication promptly, the goal of treatment should be to prevent pain rather than simply respond to it. Assessing pain with an age-appropriate tool is important; however, the effective assessment of pain shouldn't take precedence over the effective treatment of pain. Alternating stronger opioid medications with nonopioid medications may be effective, but the nurse should individualize the treatment to meet the child's needs.

The parents of a school-age child with a brain tumor have elected to have only comfort measures instituted for their dying child. The child has been experiencing significant discomfort and has been receiving pain medication. A nurse knows that the pain-management principle most effective in controlling the child's pain is: 1. striving to prevent pain by routine administration of pain medication. 2. administering pain medication promptly when the child requests it. 3. using an age-appropriate tool for effectively assessing pain. 4. alternating stronger opioid pain medications with nonopioid agents.

3. Expression of feelings, such as sorrow and anger, about the girl's condition RATIONALE: The ability to express feelings and relate them to the diagnosis is the first step in accepting the situation. Failing to recognize the seriousness of the girl's condition despite physical evidence, intellectualizing about the illness in areas unrelated to the girl's condition, and avoiding staff, family members, or the girl herself are all avoidance behaviors that represent a parent's inability to cope with the situation.

The parents of an adolescent girl have recently learned that their daughter has a terminal illness. At first, as they try to cope, they display avoidance behaviors. Then they demonstrate behaviors that indicate possible acceptance of the diagnosis. Which behavior indicates acceptance? 1. Failure to recognize the seriousness of the girl's condition despite physical evidence 2. Intellectualization about the illness in areas unrelated to the girl's condition 3. Expression of feelings, such as sorrow and anger, about the girl's condition 4. Avoidance of staff, family members, or the girl herself.

3. Atropine RATIONALE: Succinylcholine is an ultra-short-acting depolarizing agent used for rapid-sequence intubation. Bradycardia can occur, especially in children. Atropine is the drug of choice in treating or preventing succinylcholine-induced bradycardia. Lidocaine is used in adults only. Epinephrine bolus and isoproterenol aren't used in rapid-sequence intubation because of their profound cardiac effects.

To decrease the likelihood of bradyarrhythmias in children during endotracheal intubation, succinylcholine (Anectine) is used with which agent? 1. Epinephrine (Adrenalin) 2. Isoproterenol (Isuprel) 3. Atropine 4. Lidocaine (Xylocaine)

2. recumbent height with the infant supine. RATIONALE: For the most accurate measurement, the nurse should place the infant in a supine position and then measure recumbent height. Measuring recumbent height with the infant lying on the side would yield an inaccurate result. Measuring recumbent height with the infant prone would yield an inaccurately long result because it includes the length of the foot. Measuring standing height with the infant held upright would also yield an inaccurate result, at least until the child no longer needs assistance to stand up straight.

To obtain the most accurate measurement of an infant's height (length), the nurse should measure: 1. recumbent height with the infant lying on the side. 2. recumbent height with the infant supine. 3. recumbent height with the infant prone. 4. standing height with the infant held upright.

3. "I will avoid using soap and water on the affected area and will apply an emollient cream on this area frequently." RATIONALE: A parent stating he will avoid using soap and water reflects effective teaching because such washing removes moisture from the horny layer of the skin. Applied in a thin layer, emollient cream holds moisture in the skin, provides a barrier to environmental irritants, and helps prevent infection. Stating he will spread a thick coat of hydrocortisone shows ineffective teaching because topical steroid creams such as hydrocortisone should be applied sparingly as a light film; the affected area should be cleaned gently with water before the cream is applied. Scraping or abrading the skin may actually increase the risk of infection and alter drug absorption. Excessive application of steroidal creams may result in systemic absorption and Cushing's syndrome. Frequent washing dries the skin, making it more susceptible to cracking and further breakdown.

To treat a child's atopic dermatitis, a physician orders a topical application of hydrocortisone cream twice daily. After medication instruction by the nurse, which statement by the parent indicates effective teaching? 1. "I will spread a thick coat of hydrocortisone cream on the affected area and will wash this area once a week." 2. "I will gently scrape the skin before applying the cream to promote absorption." 3. "I will avoid using soap and water on the affected area and will apply an emollient cream on this area frequently." 4. "I will apply a moisturizing cream sparingly and will wash the affected area frequently."

2. Profuse diarrhea RATIONALE: Ulcerative colitis causes profuse diarrhea. Intense abdominal cramps, anal fissures, and abdominal distention are more common in Crohn's disease.

What is the most common assessment finding in a child with ulcerative colitis? 1. Intense abdominal cramps 2. Profuse diarrhea 3. Anal fissures 4. Abdominal distention

4. Permethrin (Elimite) RATIONALE: Permethrin, supplied in a cream, is the treatment of choice for children younger than age 1. However, its safety hasn't been established for clients younger than 2 months. Griseofulvin and tolnaftate are used to treat ringworm, not scabies. Thiabendazole is used to treat hookworm, roundworm, threadworm, and whipworm.

What is the recommended treatment for scabies in a child who's younger than age 1? 1. Griseofulvin (Grifulvin V) 2. Tolnaftate (Tinactin) 3. Thiabendazole (Mintezol) 4. Permethrin (Elimite)

4. Move the equipment out of reach. RATIONALE: Moving the equipment out of reach ensures a safe environment because toddlers are curious and may try to play with items within their reach. Toddlers in a strange hospital environment still need the security of a crib. Stacking toys don't need to be moved out of reach because they don't present a safety hazard and are appropriate for this age-group. Padded crib rails are necessary only if seizure activity is present.

What should a nurse do to ensure a safe hospital environment for a toddler? 1. Place the child in a youth bed. 2. Move stacking toys out of reach. 3. Pad the crib rails. 4. Move the equipment out of reach.

3. 20 ml/kg RATIONALE: Fluid volume replacement must be calculated using the child's weight to avoid overhydration. Initial fluid bolus is administered at 20 ml/kg, followed by another 20 ml/kg bolus if there is no improvement in fluid status.

What should be the initial bolus of crystalloid fluid replacement for a child in shock? 1. 10 ml/kg 2. 15 ml/kg 3. 20 ml/kg 4. 30 ml/kg

1. Tell the caller that she can't give out information about a client's condition. RATIONALE: A nurse must uphold her institution's policies and Health Insurance Portability and Accountability Act regulations by not providing information in response to questions about a client's care. Contacting social services would be indicated if the father came to the facility and demanded information. The nurse shouldn't transfer the call to the child's room. She doesn't know anything about the father's relationship with the child, and the contact might distress the child.

When a nurse answers the telephone at the front desk, a caller identifies himself as a child's father and asks how the child is doing. The nurse knows that the child's father hasn't had contact with his son for 2 years. What should the nurse do? 1. Tell the caller that she can't give out information about a client's condition. 2. Report the call to social services. 3. Give the caller a basic update on the child's prognosis. 4. Transfer the call to the child's room.

3. Encourage the parent to stand next to the crib and stay with the child. RATIONALE: The nurse should encourage the parent to stand next to the crib and stay with the child. This approach promotes compliance with treatment while minimizing the toddler's separation anxiety. Because the mist helps thin secretions and make them easier to clear, turning off the mist or letting the toddler sit next to the mist tent defeats the treatment's purpose. To prevent falls, the nurse should keep the side rails up and shouldn't permit the toddler to climb into and out of the crib.

When a toddler with croup is admitted to the facility, a physician orders treatment with a mist tent. As the parent attempts to put the toddler in the crib, the toddler cries and clings to the parent. What should the nurse do to gain the child's cooperation with the treatment? 1. Turn off the mist so the noise doesn't frighten the toddler. 2. Let the toddler sit on the parent's lap next to the mist tent. 3. Encourage the parent to stand next to the crib and stay with the child. 4. Put the side rail down so the toddler can get into and out of the crib unaided.

4. Vastus lateralis RATIONALE: The recommended injection site for an infant is the vastus lateralis or rectus femoris muscle. The deltoid is inappropriate. The dorsogluteal and ventrogluteal sites can be used only in toddlers who have been walking for about 1 year.

When administering an I.M. injection to an infant, the nurse should use which site? 1. Deltoid 2. Dorsogluteal 3. Ventrogluteal 4. Vastus lateralis

4. Using an oral syringe to place the medication beside the tongue. RATIONALE: Using an oral syringe is the best way to prevent aspiration because it allows controlled administration of a small amount of medication. Administering the medication too quickly could cause aspiration. Putting the drug in a bottle of formula isn't preferred because the infant may not take the entire dose of medication and because the contents of the bottle could interfere with drug absorption or action. Blocking the nasal passages could cause aspiration.

When administering an oral medication to an infant, the nurse should take which action to minimize the risk of aspiration? 1. Administering the oral medication as quickly as possible 2. Placing the medication in the infant's formula bottle 3. Keeping the infant upright with the nasal passages blocked 4. Using an oral syringe to place the medication beside the tongue.

2. 10% glucose RATIONALE: The amount of glucose that is considered safe for peripheral veins while still providing adequate parenteral nutrition is 10%. A glucose amount of 5% isn't sufficient nutritional replacement, although it's safe for peripheral veins. Any amount above 10% glucose, such as 15% and 17%, must be administered via central venous access.

When administering total parenteral nutrition (TPN) through a peripheral I.V. line to a school-age child, what is the lowest amount of glucose that is considered safe and not caustic to small veins that will also provide adequate TPN? 1. 5% glucose 2. 10% glucose 3. 15% glucose 4. 17% glucose

4. Productive cough RATIONALE: Bronchiolitis causes a productive cough. Clubbed fingers and a barrel chest are more likely in a client with chronic respiratory problems. A barking cough is associated with croup.

When assessing a child with bronchiolitis, which finding does the nurse expect? 1. Clubbed fingers 2. Barrel chest 3. Barking cough and stridor 4. Productive cough

3. Joint stiffness RATIONALE: Joint stiffness is an early sign of hemarthrosis. Hemarthrosis doesn't affect pulses and bleeding into the joints can't be observed directly. Hematuria is incorrect because this sign indicates bleeding in the urinary tract.

When assessing a child with hemophilia, the nurse identifies which condition as an early sign of hemarthrosis? 1. Decreased peripheral pulses 2. Active bleeding 3. Joint stiffness 4. Hematuria

1. Goiter RATIONALE: Juvenile hypothyroidism results in goiter, weight gain, sleepiness, and a slow heart rate. It doesn't cause weight loss, insomnia, or tachycardia.

When assessing a child with juvenile hypothyroidism, the nurse expects which finding? 1. Goiter 2. Recent weight loss 3. Insomnia 4. Tachycardia

2. Waddling gait RATIONALE: A waddling, wide-based gait is a sign of muscular dystrophy. A nurse wouldn't expect pain, joint swelling, and limited ROM because they are rare with this disease.

When assessing a child with muscular dystrophy, the nurse expects which finding? 1. Pain 2. Waddling gait 3. Joint swelling 4. Limited range of motion (ROM)

2. A gallop heart rhythm RATIONALE: Heart failure may cause a gallop heart rhythm in a child. Bounding peripheral pulses, widened pulse pressure, and bradycardia aren't associated with heart failure.

When assessing a child, age 3 months, who has been diagnosed with heart failure, the nurse expects which finding? 1. Bounding peripheral pulses 2. A gallop heart rhythm 3. Widened pulse pressure 4. Bradycardia

2. Incompatibility between the history (mechanism) and the injury RATIONALE: The most important criterion on which to base a decision for reporting suspected abuse is an incompatibility between the history and the injury. A maltreated child will rarely betray his parents by saying he has been abused and will, instead, attempt to defend the parent's action and verify the story. The child may even take responsibility for the act in attempt to vindicate them. However, these factors aren't as important as an incompatibility between the history and the injury. A complaint other than the one associated with the signs of abuse (for example, a complaint of being cold when second-degree burns are visible) is a warning sign of abuse but isn't the most important criterion.

When assessing a family suspected of abusing its 4-year-old child, which behavior is the most important criterion that would suggest abuse? 1. Attempts by the child to defend or verify what the parent states 2. Incompatibility between the history (mechanism) and the injury 3. Responsibility taken by the child for the act 4. A complaint other than the one associated with the signs of abuse

3. Can be roused with stimulation RATIONALE: The child is obtunded if he can be aroused with stimulation. If the child shows no motor or verbal response to noxious stimuli, he's comatose. If the child remains in a deep sleep and is responsive only to vigorous and repeated stimulation, he's stuporous. If the child has limited spontaneous movement and sluggish speech, he's lethargic.

When assessing a preschooler who has sustained a head trauma, the nurse notes that the child appears to be obtunded. Which finding supports this level of consciousness? 1. No motor or verbal response to noxious (painful) stimuli 2. Remains in a deep sleep; responsive only to vigorous and repeated stimulation 3. Can be roused with stimulation 4. Limited spontaneous movement; sluggish speech

2. Autonomy RATIONALE: The toddler's developmental task is to achieve autonomy while overcoming shame and doubt. Developing initiative is the preschooler's task whereas developing trust is the infant's task. Developing industry is the task of the school-age child.

When assessing a toddler's growth and development, the nurse understands that a child in this age-group displays behavior that fosters which developmental task? 1. Initiative 2. Autonomy 3. Trust 4. Industry

3. Positive Babinski's reflex RATIONALE: A nurse should interpret Babinski's reflex as a sign of neurologic dysfunction because this reflex should disappear by age 12 months. The gag reflex, tonic neck reflex, and corneal reflex are normal findings for a toddler.

When assessing a toddler, age 18 months, the nurse should interpret which reflex as a sign of a neurologic dysfunction? 1. Positive gag reflex 2. Positive tonic neck reflex 3. Positive Babinski's reflex 4. Positive corneal reflex

3. "Your child's height and weight must be checked again in 1 month." RATIONALE: Although the growth rate usually slows between ages 1 and 3, it normally doesn't drop as dramatically as this child's. Therefore, the nurse should advise the mother to have the child's growth rate monitored frequently, such as every month. Asking the mother what she feeds her child implies that the mother is at fault for the child's slow growth. Telling the mother not to worry is inappropriate because it doesn't address the mother's concern about the child. Asking about pregnancy weight gain is inappropriate because maternal weight gain during pregnancy wouldn't affect a child's growth rate at 18 months.

When assessing an 18-month-old child, the nurse determines that the child's height and weight fall below the 5th percentile on the growth chart. In all previous visits, the child's height and weight fell between the 30th and 40th percentiles. The child's mother expresses concern about the slowed growth rate. How should the nurse respond? 1. "What do you feed your child?" 2. "Don't worry. Your child is bound to have a growth spurt soon." 3. "Your child's height and weight must be checked again in 1 month." 4. "How much weight did you gain when you were pregnant with this child?"

1. No action is needed; this is a normal finding. RATIONALE: No action is needed by the nurse because in an infant, the anteroposterior diameter is normally twice the lateral diameter (a ratio of 1:2).

When assessing the chest of a 4-month-old infant, the nurse identifies the ratio of the anteroposterior-to-lateral diameter as 1:2. What action should the nurse take next? 1. No action is needed; this is a normal finding. 2. Inform the physician of the finding and obtain an order for a chest X-ray. 3. Instruct the parents to bring the infant back in 1 month for reevaluation. 4. Check the infant for signs of respiratory distress.

4. Promoting the nurse's personal values and beliefs if she considers the family's to be inappropriate RATIONALE: If the nurse attempts to force her beliefs on the family, the family may interpret this as a lack of understanding, which could lead to distrust of the nurse. Becoming familiar with the family's spiritual beliefs and practices, seeking assistance or referrals to the facility chaplain or other resources, and being open to the family's and the child's expressions of spiritual concerns are all ways to help children and their families cope with a life-threatening illness.

When attempting to facilitate spiritual support for a school-age child with a life-threatening disease and his family, which action would hinder the nurse-client relationship? 1. Becoming familiar with the family's spiritual beliefs and practices 2. Seeking assistance or referrals to the facility chaplain or other resources 3. Being open to the family's and the child's expressions of spiritual concerns 4. Promoting the nurse's personal values and beliefs if she considers the family's to be inappropriate

2. autonomy. RATIONALE: According to Erikson's theory of development, a 2-year-old child is at the stage of autonomy versus shame and doubt. Offering the child choices about some aspects of care encourages autonomy. An infant is at the stage of trust versus mistrust; a school-age child, industry versus inferiority; and a preschooler, initiative versus guilt.

When caring for a 2-year-old child, the nurse should offer choices, when appropriate, about some aspects of care. According to Erikson, offering choices helps the child achieve: 1. trust. 2. autonomy. 3. industry. 4. initiative.

2. Draw blood for cultures as ordered. RATIONALE: Osteomyelitis, an infectious bone disease, typically results from Staphylococcus aureus or Haemophilus influenzae. Blood cultures must be obtained to identify the causative organism and determine its sensitivity to antimicrobial agents. Although treatment may include high doses of antibiotics, blood cultures must be obtained before antibiotic therapy begins. Hepatic and renal studies are obtained during the course of antibiotic therapy to monitor the child for adverse effects. Later, surgery may be necessary to drain abscesses.

When caring for a child, age 12, who's diagnosed with osteomyelitis of the left femur, the nurse should take which action first? 1. Administer I.V. antibiotics as ordered. 2. Draw blood for cultures as ordered. 3. Monitor hepatic and renal studies. 4. Prepare the child for immediate surgery.

2. place a tracheotomy tray at the bedside. RATIONALE: Placing a tracheotomy tray at the bedside should take priority because acute epiglottiditis is an emergency situation in which inflammation can cause the airway to swell so that it's unable to rise, totally obstructing the airway. This situation may require tracheotomy or endotracheal intubation. The nurse should never depress the tongue of a child with a tongue blade to examine the throat if signs or symptoms of epiglottiditis are present because this maneuver can cause the swollen epiglottis to completely obstruct the airway. Because the child can't swallow, I.V. fluids are necessary; however, airway concerns are the priority. Only after a patent airway is secured can antibiotics be given to treat Haemophilus influenzae, a common cause of acute epiglottiditis.

When caring for a toddler with epiglottiditis, the nurse should first: 1. examine his throat. 2. place a tracheotomy tray at the bedside. 3. administer I.V. fluids. 4. administer antibiotics.

2. Keeping the adolescent's head in midline position while raising the head of the bed 15 to 30 degrees RATIONALE: Elevating the head of the bed while keeping the adolescent's head in midline position will facilitate venous drainage and avoid jugular compression. Turning the head, hyperextending the neck, and suctioning will increase ICP.

When caring for an adolescent who's at risk for injury related to intracranial pathology, which action would maintain stable intracranial pressure (ICP)? 1. Turning the adolescent's head from side to side frequently 2. Keeping the adolescent's head in midline position while raising the head of the bed 15 to 30 degrees 3. Hyperextending the adolescent's head with a blanket roll 4. Suctioning frequently to maintain a clear airway

1. Comforting the child as quickly as possible RATIONALE: After surgery to repair a cleft lip, the primary goal of nursing care is to maintain integrity of the operative site. Crying causes tension on the suture line, so comforting the child as quickly as possible is the highest nursing priority. Parents may help by cuddling and comforting the child. The prone position is contraindicated after surgery because rubbing on the sheet may disturb the suture line. Elbow restraints may cause agitation; if used to prevent the child from disturbing the suture line, they must be removed, one at a time, every 2 hours so that the child can exercise and the nurse can assess for skin irritation. Crusts forming on the suture line contribute to scarring and must be cleaned carefully.

When developing a postoperative care plan for an infant scheduled for cleft lip repair, the nurse should assign highest priority to which intervention? 1. Comforting the child as quickly as possible 2. Maintaining the child in a prone position 3. Restraining the child's arms at all times, using elbow restraints 4. Avoiding disturbing any crusts that form on the suture line

1. Have the child stand firmly on both feet and bend forward at the hips, with the trunk exposed. RATIONALE: To screen for scoliosis, a lateral curvature of the spine, the nurse has the child stand firmly on both feet with the trunk exposed and examines the child from behind, checking for asymmetry of the shoulders, scapulae, or hips. The nurse then asks the child to bend forward at the hips and inspects for a rib hump, a sign of scoliosis. Listening for a clicking sound while the child abducts the hips is appropriate when screening for congenital hip dysplasia. The heel-to-shin test evaluates cerebellar function and having the child shrug the shoulders against mild resistance helps evaluate the integrity of cranial nerve XI.

When examining school-age and adolescent children, the nurse routinely screens for scoliosis. Which statement accurately summarizes how to perform this screening? 1. Have the child stand firmly on both feet and bend forward at the hips, with the trunk exposed. 2. Listen for a clicking sound as the child abducts the hips. 3. Have the child run the heel of one foot down the shin of the other leg while standing. 4. Have the child shrug the shoulders as the nurse applies mild pressure to the shoulders.

3. 2 months, 4 months, 6 months, 15 to 18 months, and 4 to 6 years RATIONALE: According to the American Academy of Pediatrics and the Committee on Infectious Diseases, the DTaP vaccine should be administered at 2 months, 4 months, 6 months, 15 to 18 months, and 4 to 6 years (before the start of school).

Which of the following is the recommended immunization schedule for diphtheria, tetanus toxoids, and acellular pertussis (DTaP)? 1. Birth, 2 months, 6 months, 15 to 18 months, and 10 to 12 years 2. 1 month, 2 months, 6 months, 15 to 18 months, and 4 to 6 years 3. 2 months, 4 months, 6 months, 15 to 18 months, and 4 to 6 years 4. Birth, 3 months, 6 months, 12 months, and 4 to 6 years

3. Assess the neonate to determine if other apparent abnormalities are present. RATIONALE: Although low-set ears are an abnormal finding, the presence of this abnormality by itself isn't cause for immediate concern. The nurse should continue to assess the neonate to determine if other abnormalities are present. It's appropriate to note the abnormality in the medical record; however, it's even more important to continue the assessment. It's outside the scope of nursing practice to order a diagnostic test, such as an ultrasound, and there's no indication for this test.

When performing a physical examination on a neonate, the nurse notes low-set ears. What action should the nurse perform next? 1. Call the pediatrician for an immediate evaluation of the infant. 2. Note the findings in the medical record. 3. Assess the neonate to determine if other apparent abnormalities are present. 4. Order an ultrasound of the head to determine if the brain is normal.

3. Brachial artery RATIONALE: The brachial artery is the best location for evaluating the pulse of an infant younger than age 1. A child of this age has a very short and often fat neck, so the carotid artery is inaccessible. The femoral artery is usually inaccessible because of clothing and diapers. The radial artery may not be palpable if cardiac output is low, even if there is a heart beat.

When performing cardiopulmonary resuscitation on a 7-month-old infant, which location would the nurse use to evaluate the presence of a pulse? 1. Carotid artery 2. Femoral artery 3. Brachial artery 4. Radial artery

3. assess the child's current developmental level and plan care accordingly. RATIONALE: Nursing care should be planned at the developmental age of a child with Down syndrome, not the chronological age. Because children with Down syndrome can vary from mildly to severely mentally challenged, each child should be individually assessed. Directing all teaching to parents isn't appropriate because a child with Down syndrome is capable of learning, especially one with mild limitations.

When planning care for a 7-year-old boy with Down syndrome, the nurse should: 1. plan interventions at the developmental level of a 7-year-old because that is the child's age. 2. plan interventions at the developmental level of a 5-year-old because the child will have developmental delays. 3. assess the child's current developmental level and plan care accordingly. 4. direct all teaching to the parents because the child can't understand.

1. Ineffective airway clearance RATIONALE: Because airway obstruction is a life-threatening complication of epiglottiditis, Ineffective airway clearance takes highest priority. Fear, Ineffective thermoregulation, and Risk for disproportionate growth are important but don't take precedence over Ineffective airway clearance and ensuring airway patency.

When planning care for a child with epiglottiditis, the nurse should assign highest priority to which nursing diagnosis: 1. Ineffective airway clearance 2. Fear 3. Ineffective thermoregulation 4. Risk for disproportionate growth

4. An infant's kidneys excrete drugs more slowly than an adult's. RATIONALE: Because an infant has immature kidney function, drugs excreted by the kidneys are excreted more slowly, significantly altering drug effects. An infant has a faster metabolic rate, slower drug detoxification, and faster systemic drug circulation than an adult.

When planning to administer medication to a 3-month-old infant, the nurse should keep which consideration in mind? 1. An infant's metabolic rate is slower than an adult's. 2. An infant's liver detoxifies drugs faster than an adult's. 3. An infant's systemic drug circulation is slower than an adult's. 4. An infant's kidneys excrete drugs more slowly than an adult's.

4. Be sure the child is ready before starting to toilet train. RATIONALE: All of the instructions are appropriate, but knowing whether the child is ready to toilet train is initially most appropriate. Many 17-month-olds don't have the neuromuscular control to be able to be trained. Waiting a few more months until the child is closer to age 2 years allows the child to develop more control. The mother should be taught the signs of readiness for toilet training.

When teaching a mother of a 17-month-old about toilet training, which instruction would initially be most appropriate? 1. Place the toddler on the potty chair every 2 hours for 10 minutes. 2. Offer a reward every time the child has a bowel movement in the potty chair. 3. Remove the diaper and use training pants to begin the process. 4. Be sure the child is ready before starting to toilet train.

1. Fifth disease is transmitted by respiratory secretions. RATIONALE: Fifth disease is transmitted by respiratory secretions. The transmission mode for roseola is unknown. Rubella is transmitted by respiratory secretions, stool, and urine. Intestinal parasitic conditions, such as giardiasis and pinworm infection, are transmitted by stool.

When teaching parents about fifth disease (erythema infectiosum) and its transmission, the nurse should provide which information? 1. Fifth disease is transmitted by respiratory secretions. 2. Fifth disease has an unknown transmission mode. 3. Fifth disease is transmitted by respiratory secretions, stool, and urine. 4. Fifth disease is transmitted by stool.

4. "Try to maintain your child's usual lifestyle to promote normal development." RATIONALE: The nurse should encourage the parents of a child with a congenital heart defect to treat the child normally and allow self-limited activity. Telling the parents to reduce the child's caloric intake isn't appropriate because doing so wouldn't necessarily reduce cardiac demand. Telling the parents to alter disciplinary patterns and deliberately prevent crying or interactions with other children could foster maladaptive behaviors. Contact with peers promotes normal growth and development.

When teaching parents of a toddler with congenital heart disease, the nurse should explain all medical treatments and emphasize which instruction? 1. "Reduce your child's caloric intake to decrease cardiac demand." 2. "Relax discipline and limit-setting to prevent crying." 3. "Make sure your child avoids contact with small children to reduce overstimulation." 4. "Try to maintain your child's usual lifestyle to promote normal development."

RATIONALE: School-age children are subject to peer pressure, and they would rather not participate in a sport if they must wear safety apparel that provokes taunts from peers. Therefore, the nurse should discuss stylishness, comfort, and social acceptance because these are major determinants of compliance. School-age children like to swim and may work hard to perfect that skill. This age-group will usually listen to reasons for not taking illegal drugs and will adhere to group rules for not tolerating drug use. Regarding stranger danger, this age-group simply needs to be reminded of potential dangers.

When teaching school-age children important injury prevention strategies, the nurse must use creativity to gain cooperation because children tend not to comply with: 1. wearing safety apparel (helmets, knee pads, elbow pads). 2. learning to swim. 3. saying "no" when offered illegal or dangerous drugs. 4. learning "stranger danger."

1. use simple terms. RATIONALE: When explaining a procedure to a 4-year-old child, the nurse must use simple terms that the child can understand. Speaking loudly may provoke anxiety. Distracting the child with a toy is more appropriate during the procedure rather than before it. Because preschoolers have a limited attention span, the nurse should provide only the necessary basic facts — not every detail — to prevent anxiety.

When telling a 4-year-old child about an upcoming procedure, the nurse's most important consideration is to: 1. use simple terms. 2. speak loudly and clearly. 3. offer a toy to keep the child happy. 4. include every detail.

3. Consulting with health care providers to make sure the child is following the critical pathway RATIONALE: Case managers follow a group of clients, ensuring that their care follows the appropriate critical pathway. These pathways contain a timeline designed to coordinate the multidisciplinary team toward a common goal of providing a short, safe, and healthy length of stay in the hospital. Registered nurses handle most of the direct bedside client care, whereas physicians and nurse practitioners are responsible for writing medical orders. The circulating nurse and scrub nurse work in the operating room, assisting the orthopedic surgeon.

Which action illustrates the responsibilities of a pediatric case manager on the pediatric orthopedic unit? 1. Providing direct child care 2. Writing orders in the medical chart 3. Consulting with health care providers to make sure the child is following the critical pathway 4. Assisting the orthopedic surgeon in the operating room

1. Allow the infant to rest before feeding. RATIONALE: Because feeding requires so much energy, an infant with heart failure should rest before feeding. Bathing and weighing the infant and administering medications should be scheduled around feedings. An infant expends energy when crying; therefore, it's best if the infant doesn't cry.

Which action should a nurse include in the care plan for a 2-month-old infant with heart failure? 1. Allow the infant to rest before feeding. 2. Bathe the infant and administer medications before feeding. 3. Weigh and bathe the infant before feeding. 4. Feed the infant when he cries.

2. Begin I.V. fluids after obtaining the child's history. RATIONALE: The nurse should obtain the child's history and then begin I.V.fluids. Fluids are one of the most important components of therapy for sickle cell crisis; they help increase blood volume and prevent sickling and thrombosis. A child experiencing a sickle cell crisis commonly has severe pain requiring the use of I.V. analgesics such as morphine, which would be administered after fluid therapy has been started. Instructing the parents about what to expect during hospitalization is important, but it isn't the first action the nurse should take. Oxygen therapy is used only if the child is hypoxic.

Which action should the nurse take first when admitting an 11-year-old child in sickle cell crisis? 1. Administer oral pain medication while obtaining the child's history. 2. Begin I.V. fluids after obtaining the child's history. 3. Instruct the parents about what to expect during this hospitalization. 4. Start oxygen therapy as soon as the child's vital signs are taken.

4. Remove a garment. RATIONALE: According to the Denver Developmental Screening Test, most 2-year-olds are able to remove one garment. A 2½-year-old can build a tower of eight cubes and point out a picture. A 3-year-old can wash and dry his hands.

Which activity should a 2-year-old child be able to do? 1. Build a tower of eight cubes. 2. Point out a picture. 3. Wash and dry his hands. 4. Remove a garment.

1. Insist that the child remain seated while eating. RATIONALE: A child should remain seated while eating. The risk of aspiration increases if the child is running, jumping, or talking with food in his mouth. Television and toys are a dangerous distraction to toddlers and young children and should be avoided during meals. A child needs constant supervision and should be monitored while eating snacks and meals.

Which activity should a nurse recommend to prevent foreign body aspiration in a child during meals? 1. Insist that the child remain seated while eating. 2. Give the child toys to play with while eating. 3. Allow the child to watch television while eating. 4. Allow the child to eat in a separate room.

3. Talk to the mother first and then to the toddler. RATIONALE: When dealing with a crying toddler, the best approach is to talk to the mother first then to the toddler. This approach helps the toddler get used to the nurse before she attempts any procedures. It also gives the toddler an opportunity to see that the mother trusts the nurse. Ignoring the crying and screaming may be the second step. The nurse should encourage the mother to hold the toddler because it will likely help the situation. The last resort is to bring in assistance so the procedure can be completed quickly.

Which approach by a nurse is the best for trying to take a crying toddler's temperature? 1. Ignore the crying and screaming. 2. Tell the mother not to hold the child. 3. Talk to the mother first and then to the toddler. 4. Bring extra help so it can be done quickly.

4. The spaces between the ribs (intercostal) are delineated during inspiration. RATIONALE: The presence of intercostal retractions is a sign of respiratory distress from an obstruction or a disease such as pneumonia, which causes the infant to have to work to breathe. Infants and children up to age 7 are abdominal breathers; after that age, they change to an adult pattern of breathing, which uses the diaphragmatic and thoracic muscles. A normal respiratory rate for an infant up to age 1 is 20 to 40 breaths/minute; a rate between 30 and 35 breaths/minute is within this normal range. An infant's skin can become mottled if the infant is left uncovered during the examination; this change isn't a cause for concern.

Which assessment finding in a 4-month-old infant is a concern? 1. The abdominal wall is rising with inspiration. 2. The respiratory rate is between 30 and 35 breaths/minute. 3. The infant's skin is mottled during examination. 4. The spaces between the ribs (intercostal) are delineated during inspiration.

4. A history of steroid-dependent asthma RATIONALE: The child's history of steroid-dependent asthma is a contributing factor to making him at high risk for a severe exacerbation. The nurse must treat the situation as a severe exacerbation regardless of the severity of the current episode. Decreased oxygen saturation, cyanosis, retractions, and increase (not mild) work of breathing are all assessments of an asthma exacerbation, not risk factors for it. These findings should be treated with oxygen, nebulized respiratory treatments, and steroids. However, if a significant history of high-risk factors is absent, the episode can be treated without hospitalization and followed up with the pediatrician.

Which assessment should alert a nurse that a hospitalized 7-year-old child is at high risk for a severe asthma exacerbation? 1. Oxygen saturation of 95% 2. Mild work of breath 3. Intercostal or substernal retractions 4. A history of steroid-dependent asthma

4. Overindulgence RATIONALE: Parents who feel guilty about a child's illness may overindulge the child. Anger, sadness, and shock are common in parents of chronically ill children but don't necessarily indicate feelings of guilt.

Which behavior exhibited by parents of a chronically ill child may indicate feelings of guilt about the child's illness? 1. Anger 2. Sadness 3. Shock 4. Overindulgence

4. The parents will call immediate attention to undesirable behavior. RATIONALE: Calling immediate attention to undesirable behavior reflects effective teaching. This approach helps the child learn socially acceptable behavior and maintain self-esteem and a positive self-concept while learning to adapt to the rules of the larger group and society. Rules should be established clearly and enforced consistently. To reinforce desirable behavior, parents should voice requests for behavior in positive terms and use a normal speaking voice and tone when talking to or reprimanding the child. Screaming and shouting should be minimized.

Which desired outcome demonstrates effective parent teaching about disciplining a toddler? 1. The parents will set flexible rules. 2. The parents will verbalize requests for behavior in negative terms. 3. The parents will raise their voices when reprimanding the child. 4. The parents will call immediate attention to undesirable behavior.

4. Inefficient liver function RATIONALE: Inefficient liver function will most likely decrease drug metabolism during infancy. As the liver matures during the first year of life, drug metabolism improves. Decreased glomerular filtration and increased tubular secretion may affect drug excretion rather than metabolism; reduced protein-binding ability may affect drug distribution but not metabolism.

Which factor will most likely decrease drug metabolism during infancy? 1. Decreased glomerular filtration 2. Reduced protein-binding ability 3. Increased tubular secretion 4. Inefficient liver function

1. Oral electrolyte replacement solutions, breast milk, or lactose-free formula RATIONALE: Oral electrolyte replacement solutions, breast milk, or lactose-free formula may be given in small amounts to replace fluid and electrolyte losses in an infant with mild diarrhea and vomiting. I.V. fluids are usually reserved for clients experiencing severe vomiting and dehydration. Fruit juices, carbonated soft drinks, and the BRAT diet, which are high in carbohydrates and low in electrolytes, aren't recommended.

Which intervention should be included in the care plan for a 6-month-old infant with a nursing diagnosis of Deficient fluid volume related to excessive GI losses in stool and emesis? 1. Oral electrolyte replacement solutions, breast milk, or lactose-free formula 2. I.V. fluid replacement therapy 3. Clear fluids, such as fruit juices, carbonated soft drinks, and gelatin 4. Delayed introduction of food for several days followed by the BRAT (bananas, rice, apples, and toast or tea) diet

3. Interviewing adolescents with their parents present RATIONALE: When possible, adolescents should be interviewed without their parents present to ensure confidentiality and privacy. Interviewing adolescents with their parents present hinders the formation of the nurse-adolescent relationship. Avoiding assumptions, judgments, and lectures will increase the adolescents' comfort in disclosing sensitive information. Begin with less-sensitive questions so the adolescents won't feel threatened and uncomfortable and become uncooperative during the interview. Ask open-ended questions to give adolescents opportunities to share their psychosocial context.

Which interview strategy contributes to a poor nurse-adolescent relationship? 1. Maintaining objectivity by avoiding assumptions, judgments, and lectures 2. Beginning with less-sensitive issues and proceed to more-sensitive ones 3. Interviewing adolescents with their parents present 4. Asking open-ended questions and moving to more directive questions when possible

3. Assisting with intubation RATIONALE: The most important intervention for a child with epiglottiditis is airway management because children are at high risk for developing abrupt airway obstruction. Therefore, intubation should be performed as soon as possible in a controlled environment. Children need supplemental oxygen, but most are so anxious that they will never allow a mask to stay in place. Provide humidified "blow-by" oxygen administered by the parent if possible. The child does need parenteral antibiotics; however, the priority is airway management. The most common rhythm in this client is sinus tachycardia related to compensation. However, monitoring for arrhythmias isn't a priority over airway management.

Which is the priority intervention for a preschool child with epiglottiditis and a deteriorating respiratory status? 1. Administering oxygen by face mask 2. Administering parenteral antibiotics 3. Assisting with intubation 4. Monitoring the electrocardiogram for arrhythmias

2. Check the hospital identification bracelet. RATIONALE: The only safe method for identifying the child is to check the identification band for the client's name and medical record number and then compare that information with the medication record. Children sometimes exchange beds during play, so checking the name on the bed isn't reliable. Infants are unable to give their names, toddlers or preschoolers may admit to any name, and school-age children may deny their identities in an attempt to avoid the medication. Parents aren't always at the bedside, so they shouldn't be relied on for identification.

Which method is reliable for identifying a preschooler before administering a medication? 1. Check the name on the bed. 2. Check the hospital identification bracelet. 3. Ask the child his name. 4. Ask the parents at the bedside.

4. Let the child choose whether to ride to the preoperative area on a stretcher or in a wagon. RATIONALE: Giving the child a choice would promote cooperation, and children commonly prefer a nonthreatening method of travel such as a wagon. Having the child take off his own underwear isn't appropriate because preschoolers commonly have a fear of genital mutilation; the child would likely resist removing his underwear. Children usually won't transfer feelings of security objects to another object such as a hospital blanket. Both parents are encouraged to accompany the child to the preoperative area, so having the child choose one parent isn't appropriate.

Which nursing action would be most successful in gaining a preschooler's cooperation in preparing for surgery? 1. Have the child take off his own underwear. 2. Encourage the child to use the hospital blanket as a transition object so his won't be lost. 3. Let the child choose which parent can accompany him to the preoperative waiting area. 4. Let the child choose whether to ride to the preoperative area on a stretcher or in a wagon.

3. Creating a therapeutic, homelike environment for the infant and his family RATIONALE: The goal of palliative care is to make the infant and his family as comfortable as possible. Maintaining routines and structure doesn't support the principles of palliative care. Clustering care activities may allow the infant more rest, but this action isn't a principle of palliative care. Minimizing noise and disruption isn't specifically related to palliative care.

Which nursing activity supports the principles of palliative care for a dying infant and his family? 1. Maintaining routines and structure for the infant and his family 2. Clustering care activities to provide as much rest as possible for the infant 3. Creating a therapeutic, homelike environment for the infant and his family 4. Minimizing noise and disruption to decrease stress for the infant

3. Ineffective airway clearance related to laryngospasm RATIONALE: Ineffective airway clearance related to laryngospasm is the most appropriate nursing diagnosis for a preschool child with epiglottiditis because complete upper airway obstruction may occur suddenly and be precipitated by improper examination or intervention. The upper airway obstruction is the result of laryngospasm and edema. Anxiety related to separation from parent isn't an appropriate nursing diagnosis because the client is likely anxious because of respiratory distress. The nurse should allow the parent to stay with the child and should encourage the parent to hold and reassure the child. The child will probably be tachycardic, not bradycardic until respiratory failure ensues. The child has impaired gas exchange from impeded airflow, not from a noncompliant lung.

Which nursing diagnosis is the most appropriate for a preschool child with epiglottiditis? 1. Anxiety related to separation from parent 2. Decreased cardiac output related to bradycardia 3. Ineffective airway clearance related to laryngospasm 4. Impaired gas exchange related to noncompliant lungs

1. Risk for infection RATIONALE: Because infection is a serious risk for a client in the early stages of burn recovery, a diagnosis of Risk for infection takes highest priority. Diagnoses of Impaired physical mobility, Disturbed body image, and Constipation may be relevant but take lower priority at this time.

Which nursing diagnosis takes highest priority for a child in the early stages of burn recovery? 1. Risk for infection 2. Impaired physical mobility 3. Disturbed body image 4. Constipation

2. Place the child on a sheepskin. RATIONALE: Placing the child with Reye's syndrome on a sheepskin helps to prevent pressure on prominent areas of the body. Rubbing lotion on the extremities stimulates circulation and helps prevent drying of the skin, and therefore shouldn't be avoided. Keeping extremities flexed can lead to contractures. Placing the child supine is contraindicated because of the risk of aspiration and increasing intracranial pressure. The supine position isn't appropriate because it puts pressure on the sacral and occipital areas.

Which nursing intervention should be included in the care of an unconscious child with Reye's syndrome? 1. Keep his arms and legs flexed. 2. Place the child on a sheepskin. 3. Avoid using lotions on his skin. 4. Place the child in a supine position.

4. Side rails in the halfway position RATIONALE: Side rails in the halfway position pose the biggest threat because the most common accidents in hospitals are falls. To prevent falls, the crib rails always should be raised and fastened securely unless an adult is at the bedside. Crayons and paper and a stuffed teddy bear are safe toys for a 2-year-old child. Although a mobile could pose a safety threat to this child, the threat is less serious than that posed by an incorrectly positioned side rail.

Which of the following objects poses the most serious safety threat to a 2-year-old child in the hospital? 1. Crayons and paper 2. Stuffed teddy bear in the crib 3. Mobile hanging over the crib 4. Side rails in the halfway position

2. Change in behavior RATIONALE: Positive changes in behavior and vital signs are indicators of an effective response to pain medication. Sleeping isn't a reliable indicator of pain relief because the teen may use sleep as a coping mechanism.

Which parameter is an appropriate indicator of pain relief in an adolescent? 1. Intermittent sleeping 2. Change in behavior 3. No change in behavior 4. No change in vital signs

4. Use of an infusion pump to regulate the flow rate RATIONALE: Use of an infusion pump to regulate the flow rate is the appropriate safeguard because infants and children with compromised cardiopulmonary status are particularly vulnerable to I.V. fluid overload. Administering fluid at the slowest possible rate may not benefit the infant. Using a gravity infusion set or a small I.V. infusion set won't protect against fluid overload when I.V. administration is too rapid.

Which safeguard should a nurse employ with I.V. fluid administration for an infant? 1. Administration of fluid at the slowest possible rate 2. Use of a gravity infusion set 3. Use of a small I.V. infusion set 4. Use of an infusion pump to regulate the flow rate

2. Tachycardia RATIONALE: The earliest sign of heart failure in infants is tachycardia (sleeping heart rate greater than 160 beats/minute) as a direct result of sympathetic stimulation. Tachypnea (respiratory rate greater than 60 breaths/minute in infants) occurs later in response to decreased lung compliance. Poor weight gain is a result of the increased energy demands to the heart and breathing efforts, not an early sign of heart failure itself. Pulmonary edema occurs as the left ventricle fails and blood volume and pressure increase in the left atrium, pulmonary veins, and lungs; it isn't an early sign of heart failure.

Which sign is an early indicator of heart failure in an infant with a congenital heart defect? 1. Tachypnea 2. Tachycardia 3. Poor weight gain 4. Pulmonary edema

1. Conflicting stories about the accident or injury from the parents RATIONALE: Conflicting stories about the accident or injury from the parents is a warning sign of abuse. A history consistent with the child's injuries, a disheveled appearance and low socioeconomic status, and an appropriate emotional response by the caregiver aren't indicators of expected or potential abuse.

Which sign is likely to indicate abuse in a 4-year-old child? 1. Conflicting stories about the accident or injury from the parents 2. History consistent with the child's injuries 3. Disheveled parental appearance and low socioeconomic status 4. Appropriate emotional response by the caregiver

1. Two nurses talk about the adolescent on an elevator on their way to lunch. RATIONALE: The elevator isn't a secure area in which to talk about any client, including an adolescent; anyone could overhear the nurses' conversation. A client isn't breaching his own confidentiality if he volunteers information about himself. When a client is present for the conversation, he can object at any time to the content of the conversation. Physicians and other health care providers are expected to discuss clients and cases, as long as they do so within the context of a professional relationship and the discussion is necessary for the course of treatment.

Which situation violates a hospitalized adolescent's right to confidentiality? 1. Two nurses talk about the adolescent on an elevator on their way to lunch. 2. The adolescent talks about his disease to someone in the hallway. 3. A physician discusses treatment plans with the adolescent in his mother's presence. 4. A physician discusses a new medication for the adolescent while on the phone with the pharmacist.

3. "I've been checking the urine for protein so I'll be able to do it at home." RATIONALE: The mother stating that she'll check her toddler's urine for protein indicates effective teaching because such testing helps detect the progression of nephrotic syndrome. The child doesn't need to be kept quiet and usually isn't placed on a specific diet. How the child feels will dictate the child's activity level. Most children return to normal soon but may relapse.

Which statement by a mother of a toddler with nephrotic syndrome indicates that the nurse's discharge teaching was effective? 1. "I know that I'll need to keep my child as quiet as possible." 2. "I just went out and bought all I'll need for the special diet." 3. "I've been checking the urine for protein so I'll be able to do it at home." 4. "I'm sure that my child will be back to normal soon and I won't have to worry about this anymore."

1. "My wife and I feel that our real daughter has moved on even though her body is still functioning." RATIONALE: Statements indicating that the family has accepted the grave condition of their child is a green light for approaching them about organ donation. Statements that represent the family's nonacceptance of the child's prognosis, the lack of understanding of treatments that are being given, or the misunderstanding of organ and tissue donation are indications that the family isn't ready to be approached or to make a decision.

Which statement indicates that a family of a dying 4-year-old may be ready to consider organ donation? 1. "My wife and I feel that our real daughter has moved on even though her body is still functioning." 2. "Those physicians aren't doing everything they can for our daughter. I know she's still in there." 3. "When will our daughter wake up and be with us?" 4. "How can some parents allow their children to be cut up like a piece of meat and given away?"

4. Verify the physician order. RATIONALE: The nurse should first verify the physician's order. Next, the nurse should make sure she has the right drug, dose, route, and time. She should then make sure she has the right client by checking the infant's armband. After these steps, the nurse should hold the infant securely in the crook of her arm and raise the infant's head to about a 45-degree angle. Then, the nurse should place the dropper at the corner of the infant's mouth so the drug runs into the pocket between the infant's cheek and gum. Doing this keeps him from spitting out the drug and reduces the risk of aspiration.

Which step should a nurse take first when administering a liquid medication to an infant? 1. Hold the infant securely in the crook of her arm and raise the infant's head to about a 45-degree angle. 2. Place the dropper at the corner of the infant's mouth so the drug runs into the pocket between the cheek and gum. 3. Identify the infant by checking the armband. 4. Verify the physician order.

3. Hand washing RATIONALE: Hand washing is the single most important measure for preventing infection transmission. Isolating the child and using infection control precautions are required for certain diseases, such as varicella, diphtheria, mumps, pertussis, measles, and meningitis. Standard precautions, which include hand washing, are guidelines for treating all clients as potentially infectious. A needleless syringe system will prevent transmission through needle sticks but not from body fluid contact.

Which technique is most effective in preventing nosocomial infection transmission when caring for a preschooler? 1. Client isolation 2. Standard precautions 3. Hand washing 4. Needleless syringe system

2. A puzzle with large pieces RATIONALE: A puzzle is the most appropriate toy because, at age 3, children like to color, draw, and put together puzzles. A bicycle is appropriate for a 5- or 6-year-old child; a pull toy, for a toddler; and a computer game, for a school-age child.

Which toy is appropriate for a 3-year-old child? 1. A bicycle 2. A puzzle with large pieces 3. A pull toy 4. A computer game

1. To substitute for observation RATIONALE: Restraints should never be used as a punishment or as a substitute for observation because if a child is at risk for harming himself when left alone, the child requires one-on-one observation. Ensuring the child's comfort or safety (restraining him to keep an I.V., drainage tube, or orthopedic device in place), facilitating examination, and carrying out procedures are all valid reasons for restraint. Restraining devices aren't without risk and must be checked and documented every 1 to 2 hours.

Which use of restraints in a school-age child should the nurse question? 1. To substitute for observation 2. To ensure the child's comfort or safety 3. To facilitate examination 4. To aid in carrying out procedures

3. administer five back blows. RATIONALE: The nurse should clear the airway with back blows and chest thrusts. Attempting rescue breaths is futile because they can't be administered until the airway is patent. After two attempts to establish an airway, the nurse can assume the airway is blocked. The nurse can't attempt to ventilate the infant with a handheld resuscitation bag until the airway is patent.

While assessing a 2-month-old infant's airway, the nurse finds that he isn't breathing. After two unsuccessful attempts to establish an airway, the nurse should: 1. attempt rescue breaths. 2. attempt to establish an airway a third time. 3. administer five back blows. 4. attempt to ventilate with a handheld resuscitation bag.

2. Talk with the child's parents when they arrive. RATIONALE: A nurse who suspects child abuse should talk with the parents and get additional details about the injuries and compare their story with that of the child. Telling the physician to call the police or contacting Child Protective Services isn't the best action to take at this time. If further investigation continues to raise questions about abuse, these steps may be appropriate. The nurse needn't continue questioning the child.

While doing the shift assessment on a 5-year-old boy, a nurse notices several bruises on his back and arms. The bruises are different colors and sizes. When she asks the child how he got them, he states, "I fell off of my bike." What should the nurse do next? 1. Contact the physician and tell him to call the police. 2. Talk with the child's parents when they arrive. 3. Contact Child Protective Services to report the injuries. 4. Continue to ask the child how he received the injuries.

1. notify the physician. RATIONALE: Because the anterior fontanel normally closes between ages 12 and 18 months, the nurse should notify the physician promptly of this abnormal finding. An open fontanel doesn't indicate abuse and isn't associated with Tay-Sachs disease.

While examining a 2-year-old child, the nurse sees that the anterior fontanel is open. The nurse should: 1. notify the physician. 2. look for other signs of abuse. 3. recognize this as a normal finding. 4. ask about a family history of Tay-Sachs disease.

3. determine the caller's identity before responding. RATIONALE: The nurse must identify the caller before giving information or refusing to give information. Client confidentiality is mandatory and isn't negated by the concept of public relations. The caller's identity and relationship to the infant may make it appropriate for the nurse to divulge information over the phone. The nurse doesn't need to transfer the call.

While providing care for a hospitalized infant, a nurse is summoned to the phone. The caller requests information about the infant's condition. The nurse should: 1. update the caller in the interest of good public relations. 2. protect the infant's confidentiality by divulging no information to the caller. 3. determine the caller's identity before responding. 4. transfer the call to the infant's room.

1. Knee-to-chest RATIONALE: TOF involves four defects: pulmonary stenosis, right ventricular hypertrophy, ventricular-septal defect (VSD), and dextroposition of the aorta with overriding of the VSD. Pulmonary stenosis decreases pulmonary blood flow and right-to-left shunting via the VSD, causing desaturated blood to circulate. The nurse should place the child in the knee-to-chest position because this position reduces venous return from the legs and increases systemic vascular resistance, maximizing pulmonary blood flow and improving oxygenation status. Fowler's, Trendelenburg's, and the prone positions don't improve oxygenation.

A 10-month-old infant with tetralogy of Fallot (TOF) experiences a cyanotic episode. To improve oxygenation during such an episode, the nurse should place the infant in which position? 1. Knee-to-chest 2. Fowler's 3. Trendelenburg's 4. Prone

4. Colonoscopy with biopsy RATIONALE: Crohn's disease is an inflammatory bowel disorder characterized by inflammation, ulceration, and edema of the bowel wall (typically involving the terminal ileum). Colonoscopy with biopsy are the primary procedures used to establish the diagnosis; a barium enema also may be indicated. Although genetics may play a role in Crohn's disease, genetic testing isn't part of the diagnostic workup. Cystoscopy visualizes the bladder and urinary tract and isn't indicated for this client. Myelography is a radiographic procedure used to evaluate the spinal cord.

A 13-year-old girl is being evaluated for possible Crohn's disease. The nurse expects to prepare her for which diagnostic study? 1. Genetic testing 2. Cystoscopy 3. Myelography 4. Colonoscopy with biopsy

1. A bedtime snack of an 8-oz glass of milk and graham crackers with peanut butter RATIONALE: Milk is a readily absorbed form of carbohydrate and will elevate blood glucose level rapidly, thus alleviating hypoglycemia. Crackers and peanut butter contain complex carbohydrates and will maintain blood glucose level. Decreased activity and sleep aren't effective for hypoglycemia. Glucagon should be reserved for more severe signs of hypoglycemia, such as disorientation and unconsciousness. To avoid rapid deterioration, steps should be taken whenever hypoglycemia is suspected, regardless of who performed the measurement.

A 14-year-old adolescent with type 1 diabetes checks his blood glucose level at 9:00 p.m. before going to bed. It has been 4 hours since his dinner and his regular insulin dose. His blood glucose level is 60 mg/dl, and he states that he feels a little shaky. What should the nurse suggest? 1. A bedtime snack of an 8-oz glass of milk and graham crackers with peanut butter 2. Going to sleep to decrease the metabolic demands on the body 3. Taking a dose of glucagon 4. Doing nothing because the glucose level is unreliable because the adolescent measured it himself

1. Appendicitis RATIONALE: Right lower quadrant pain, rebound tenderness, nausea, vomiting, a positive psoas sign, and a low-grade fever are findings consistent with acute appendicitis. Pancreatitis, cholecystitis, and constipation may mimic appendicitis; however, the pain of pancreatitis is usually localized in the left upper quadrant. Cholecystitis is associated with right upper quadrant pain. Constipation wouldn't cause a fever.

A 14-year-old male reports having right lower quadrant pain, nausea, vomiting, and a low-grade fever for the past 12 hours. A physical examination reveals rebound tenderness and a positive psoas sign. Based on these findings, what should the nurse suspect? 1. Appendicitis 2. Pancreatitis 3. Cholecystitis 4. Constipation

1. Minor symptoms can be treated with acetaminophen (Tylenol). 3. Call the office if the toddler develops a temperature above 103° F (39.4° C), seizures, or difficulty breathing. 4. Soreness at the immunization site and mild fever are common. RATIONALE: The nurse should tell the parents that minor symptoms, such as soreness at the immunization site and mild fever, can be treated with acetaminophen or ibuprofen. Aspirin should be avoided in children because of its association with Reye's syndrome. The parents should notify the clinic if serious complications (such as a temperature above 103° F, seizures, or difficulty breathing) occur. Minor discomforts, such as soreness and mild fever, are common after immunizations. Immunizing the child decreases the health risks associated with contracting certain diseases; it doesn't prevent the toddler from acquiring them. Although the child may prefer to rest after immunizations, it isn't necessary to restrict his activity.

A 15-month-old toddler has just received his routine immunizations, including diphtheria, tetanus, and acellular pertussis; inactivated polio vaccine; measles, mumps, and rubella; varicella; and pneumococcal conjugate vaccine. What information should the nurse give to the parents before they leave the office? Select all that apply. 1. Minor symptoms can be treated with acetaminophen (Tylenol). 2. Minor symptoms can be treated with aspirin (A.S.A.). 3. Call the office if the toddler develops a temperature above 103° F (39.4° C), seizures, or difficulty breathing. 4. Soreness at the immunization site and mild fever are common. 5. The immunizations prevent the toddler from contracting their associated diseases. 6. The toddler should restrict his activity for the remainder of the day.

3. Respiratory acidosis RATIONALE: A pH less than 7.35 and a PaCO2 greater than 45 mm Hg indicate respiratory acidosis. Status asthmaticus is a medical emergency that's characterized by respiratory distress. Persistent hypoventilation leads to the accumulation of carbon dioxide, resulting in respiratory acidosis.

A 4-year-old child is being treated for status asthmaticus. His arterial blood gas analysis reveals a pH of 7.28, PaCO2 of 55 mm Hg, and HCO3− of 26 mEq/L. What condition do these findings indicate? 1. Respiratory alkalosis 2. Metabolic acidosis 3. Respiratory acidosis 4. Metabolic alkalosis

3. provide oral and I.V. fluids. RATIONALE: Initial nursing interventions for the child in a sickle cell crisis include providing hydration and oxygenation to prevent more sickling. Pain relief is also a concern. However, painful joints are treated with analgesics and warm packs because cold packs may increase sickling. Antibiotics will be given to treat a sickle cell crisis if it's thought to be bacterial but only after hydration and oxygenation have been addressed. Daily supplements of folic acid will help counteract anemia but they aren't a priority during sickle cell crisis.

A 4-year-old child is having a sickle cell crisis. The initial nursing intervention should be to: 1. place ice packs on the client's painful joints. 2. administer antibiotics. 3. provide oral and I.V. fluids. 4. administer folic acid supplements.

3. Irrigate the NG tube to ensure patency. RATIONALE: The nurse should first irrigate the NG tube because if the tube isn't draining properly or is kinked, the child will experience nausea. There's no reason to notify the physician immediately because a nurse should be able to handle the situation. Giving the child an antiemetic doesn't really address the problem. Encouraging the mother to calm the child is always a good intervention but isn't the first thing to do in this case.

A 4-year-old has just returned from surgery. He has a nasogastric (NG) tube in place and is attached to intermittent suction. The child says to the nurse, "I'm going to throw up." What should the nurse do first? 1. Notify the physician because the child has an NG tube. 2. Immediately give the child an antiemetic I.V. 3. Irrigate the NG tube to ensure patency. 4. Encourage the mother to calm the child down.

2 teaspoons RATIONALE: To perform this dosage calculation, the nurse should first convert the child's weight from pounds to kilograms: 44 lb ÷ 2.2 lb/kg = 20 kg Then she should calculate the total daily dose for the child: 20 kg × 0.2 mg/kg/day = 4 mg Next, the nurse should calculate the amount to be given at each dose: 4 mg ÷ 4 doses = 1 mg/dose The available elixir contains 0.5 mg of drug per 5 ml (which is equal to 1 teaspoon). Therefore, to give 1 mg of the drug, the nurse should administer 2 teaspoons (10 ml) to the child for each dose.

A 44-lb preschooler is being treated for inflammation. The physician orders 0.2 mg/kg/day of dexamethasone (Decadron) by mouth to be administered every 6 hours. The elixir comes in a strength of 0.5 mg/5 ml. How many teaspoons of dexamethasone should the nurse give this client per dose? Record your answer using a whole number. Answer: teaspoons

4. The pedal pulse of the right leg isn't detectable. RATIONALE: Using the femoral vein during catheterization can cause the affected blood vessels to spasm or cause a blood clot to develop, altering circulation in the leg. The inability to detect the pedal pulse in the affected leg is an ominous sign and requires immediate intervention. Small amounts of coolness or pallor are normal. These findings should improve. Although the nurse should continue to monitor a dressing with a small amount of blood on it, this finding isn't the priority in this situation.

A 5-year-old child returns to the pediatric unit following a cardiac catheterization using the right femoral vein. The child has a thick elastoplast dressing. Which assessment finding requires immediate intervention? 1. One leg is slightly cooler than the other leg. 2. The leg used for the catheter insertion is slightly paler than the other leg. 3. A small amount of bright red blood is seen on the dressing. 4. The pedal pulse of the right leg isn't detectable.

3. Notify the physician and request assistance from the interdisciplinary team. RATIONALE: The child's clinical presentation and the mother's behavior suggest Munchausen syndrome by proxy, a condition in which an individual fabricates or induces symptoms of a disorder in another person. Suspicion of this condition mandates a coordinated evaluation by the health care team. Rather than asking the parent to leave, the nurse should establish a rapport with her. Doing so will prevent the parent from becoming suspicious and leaving the health care organization, which would potentially allow the cycle to continue. The nurse must contact authorities when she obtains additional evidence.

A 6-year-old child is admitted to the pediatric unit for evaluation of recurrent abdominal pain. The child has been admitted to the pediatric unit with similar complaints several times in the past few months. The child's symptoms are vague, yet his mother provides detailed information about the problem. The nurse is suspicious of the situation. What should the nurse do next? 1. Request that the parent leave the hospital unit immediately. 2. Ask to speak with the child without the parent being present. 3. Notify the physician and request assistance from the interdisciplinary team. 4. Contact the authorities immediately.

4. Select appropriate injection site with the child. 3. Clean site with an alcohol pad; loosen needle cover. 1. Pinch the skin around the injection site 6. Uncover needle; insert at 45- to 90- degree angle. 2. Release the skin and give the injection. 5. Cover the site with an alcohol pad. RATIONALE: To give a subcutaneous injection of insulin to a child, the nurse should first select an appropriate injection site, being sure to discuss the selection with the child to ensure that injection sites are rotated. She should then clean the injection site with an alcohol pad and loosen the needle cover. The next step is to pinch the skin around the site. She should then uncover the needle and insert the needle at a 45- to 90-degree angle, release the skin, and give the injection. When finished, the nurse should cover the injection site with an alcohol pad and avoid rubbing the site.

A 9-year-old boy with diabetes mellitus tests his glucose level in the nurse's office before lunch. According to this sliding scale of insulin, he's due for 1 unit of regular insulin. What steps should a nurse follow after confirming the medication order, washing her hands, drawing up the appropriate dose, verifying the boy's identity, and putting on gloves? Put the following steps in chronological order. 1. Pinch the skin around the injection site 2. Release the skin and give the injection. 3. Clean site with an alcohol pad; loosen needle cover. 4. Select appropriate injection site with the child. 5. Cover the site with an alcohol pad. 6. Uncover needle; insert at 45- to 90- degree angle.

2. 1 to 3 years old (a toddler). RATIONALE: Toddlers show fear of separation from their parents, the dark, loud or sudden noises, injury, strangers, certain persons, certain situations, animals, large objects or machines, and change in environment. Infants show fear of strangers, the sudden appearance of unexpected and looming objects (including people), animals, and heights. School-age children show fear of supernatural beings, injury, storms, the dark, staying alone, separation from parents, things seen on television and in the movies, injury, tests and failure in school, consequences related to unattractive physical appearance, and death. Adolescents show fear of inept social performance, social isolation, sexuality, drugs, war, divorce, crowds, gossip, public speaking, plane and car crashes, and death.

A child has just been admitted to the facility and is displaying fear related to separation from his parents, the room being too dark, being hurt while in the hospital, and having many different staff members come into the room. Based on the nurse's knowledge of growth and development, the child is likely: 1. 7 to 12 months old (an infant). 2. 1 to 3 years old (a toddler). 3. 6 to 12 years old (a school-age child). 4. 12 to 18 years old (an adolescent).

2. Sadness RATIONALE: Clinical depression is diagnosed if the child exhibits a depressed mood (sadness) or loss of interest. Irritability isn't diagnostic for depression. Although a depressed child may gain weight and report fatigue, these findings aren't essential to the diagnosis.

A child is admitted with a tentative diagnosis of clinical depression. Which assessment finding is most significant in confirming this diagnosis? 1. Irritability 2. Sadness 3. Weight gain 4. Fatigue

2. monitoring the blood glucose level closely. RATIONALE: Most TPN solutions contain a high glucose content, placing the client at risk for hyperglycemia. Therefore, the most important nursing action is to monitor the child's blood glucose level closely. A child receiving TPN isn't likely to require vital sign assessment every 30 minutes or elevation of the head of the bed. A daily bath isn't a priority.

A child is receiving total parenteral nutrition (TPN). During TPN therapy, the most important nursing action is: 1. assessing vital signs every 30 minutes. 2. monitoring the blood glucose level closely. 3. elevating the head of the bed 60 degrees. 4. providing a daily bath.

1. The parent verbalizes the need to stay away from persons with known infections. RATIONALE: Preventing infections through proper hand washing and staying away from persons with known infections is an important measure in preventing sickle cell crises. Dietary restrictions aren't significant in preventing these crises. The client should maintain adequate hydration, not restrict fluid intake, and should avoid strenuous activity such as aerobics.

A child who was hospitalized for sickle cell crisis is being discharged. Which parent outcome demonstrates effective teaching regarding prevention of future crises? 1. The parent verbalizes the need to stay away from persons with known infections. 2. The parent verbalizes appropriate dietary restrictions. 3. The parent verbalizes the need to restrict fluid intake. 4. The parent participates in an aerobic exercise program.

3. Elevate the affected arm and apply ice to the injury site. RATIONALE: Severe joint pain in a child with hemophilia indicates bleeding; therefore, the nurse should first elevate the affected extremity and apply ice to the injury site to promote vasoconstriction. ROM exercises may worsen discomfort and bleeding. Massage and warm compresses also may increase bleeding. The nurse should notify the physician only after taking measures to stop the bleeding.

A child with hemophilia is hospitalized after falling. Now the child complains of severe pain in the left wrist. What should the nurse do first? 1. Perform passive range-of-motion (ROM) exercises on the wrist. 2. Massage the wrist and apply a warm compress. 3. Elevate the affected arm and apply ice to the injury site. 4. Notify the physician.

1. "The vitamin C in the citrus juice helps with iron absorption." RATIONALE: Administering an oral iron supplement such as ferrous sulfate with citrus juice or another vitamin C source enhances its absorption. Preferably, doses should be administered between meals because gastric acidity and absence of food promote iron absorption. Although citrus juice may improve the taste of an oral iron supplement, this isn't the primary reason for mixing the two together. Telling the mother that there isn't a specific reason for mixing the supplement with citrus juice is inappropriate and inaccurate.

A child with iron deficiency anemia is ordered ferrous sulfate (Ferralyn), an oral iron supplement. When teaching the child and parent how to administer this preparation, the mother asks why she needs to mix the supplement with citrus juice. Which response by the nurse is best? 1. "The vitamin C in the citrus juice helps with iron absorption." 2. "Having food and juice in the stomach helps with iron absorption." 3. "The citrus juice counteracts the unpleasant taste of the iron." 4. "There isn't a specific reason for it."

0.2 milliliters RATIONALE: The nurse should calculate the volume to be given using this equation: 2 mg/X ml = 10 mg/1 ml 10X = 2 X = 0.2 ml

A child with sickle cell anemia is being treated for a crisis. The physician orders morphine sulfate, 2 mg I.V. The concentration of the vial is 10 mg/1 ml of solution. How many milliliters of solution should the nurse administer? Record your answer using one decimal place. Answer: milliliters

2. provide the family with the drug's name, dosage, route, and frequency of administration. RATIONALE: Before the child is discharged, the nurse should provide the family with essential facts: the drug's name, dosage, route, and frequency of administration. Generally the physician, not the family or nurse, adjusts dosages. It's unrealistic and unsafe to expect a child to take responsibility for ensuring timely administration of any drug. A child has a right to know the reasons for taking the drug.

A child's physician orders a drug for home use. Before the child is discharged, the nurse should: 1. teach the family how to adjust the drug dosage according to the child's needs. 2. provide the family with the drug's name, dosage, route, and frequency of administration. 3. instruct the family to encourage the child to take responsibility for ensuring timely drug administration. 4. tell the family to avoid explaining the purpose of the medication to the child.

3. "My child's abdomen seems bigger, and his diapers are much tighter." RATIONALE: The most common presenting sign of a Wilms' tumor is abdominal swelling or an abdominal mass. Therefore, the mother's observation that her child's abdomen seems bigger suggests a Wilms' tumor. A rapid increase in length (height) isn't associated with this type of tumor. Although lethargy may accompany a Wilms' tumor, abdominal swelling is a more specific sign. Children with a Wilms' tumor usually have a decreased, not increased, appetite.

A child, age 15 months, is admitted to the health care facility. During the initial nursing assessment, which statement by the mother most strongly suggests that the child has a Wilms' tumor? 1. "My child has grown 3" in the past 6 months." 2. "My child seems to be napping for longer periods." 3. "My child's abdomen seems bigger, and his diapers are much tighter." 4. "My child's appetite has increased so much lately."

4. Increased interest in play RATIONALE: A behavioral change is one of the most valuable clues to pain. A child who's pain-free likes to play. In contrast, a child in pain is less likely to play or to consume food or fluids. An increased heart rate may indicate increased pain. Decreased urine output may signify dehydration.

A child, age 15 months, is recovering from surgery to remove a Wilms' tumor. Which finding best indicates that the child is free from pain? 1. Decreased appetite 2. Increased heart rate 3. Decreased urine output 4. Increased interest in play

4. "Wear gloves when you're likely to come into contact with the child's blood or body fluids." RATIONALE: HIV is transmitted by blood and body fluids. Therefore, the nurse should respond by telling family members they should wear gloves when anticipating contact with the child's blood or body fluids. Standard household methods for cleaning dishes and utensils are adequate, so the child needn't use disposable plates and utensils. To disinfect HIV-contaminated surfaces, the nurse should instruct the foster parents to use a solution of 1 part bleach to 10 parts water. The child may share toys; any toys that become soiled with the child's blood or body fluids should be disinfected with the bleach solution.

A child, age 3, who tests positive for the human immunodeficiency virus (HIV) is placed in foster care. The foster parents ask the nurse how to prevent HIV transmission to other family members. How should the nurse respond? 1. "Make sure the child uses disposable plates and utensils." 2. "Use isopropyl alcohol to clean surfaces contaminated with the child's blood or body fluids." 3. "Don't let the child share toys with other children." 4. "Wear gloves when you're likely to come into contact with the child's blood or body fluids."

3. blindness. RATIONALE: Neurotoxicity, the primary adverse effect of vincristine, may manifest as blindness that the parents must report promptly. Neurotoxicity may also cause peripheral neuropathy. Hair loss and moon face are expected adverse effects of this chemotherapy regimen and will resolve once therapy ends. Bone pain is common in clients with ALL and results from invasion of the periosteum by leukemic cells.

A child, age 5, has acute lymphocytic leukemia (ALL) and is receiving induction chemotherapy consisting of vincristine (Oncovin), asparaginase (L-asparaginase [Elspar]), and prednisone (Deltasone). When teaching the parents about the adverse effects of this regimen, the nurse should stress the importance of promptly reporting: 1. hair loss. 2. moon face. 3. blindness. 4. bone pain.

1. Meats RATIONALE: The nurse should instruct the parents to restrict meats because they contain a large amount of protein. Dairy products, carbohydrates, and fats are appropriate food choices for this child.

A child, age 5, is diagnosed with chronic renal failure. When teaching the parents about diet therapy, the nurse should instruct them to restrict which foods from the child's diet? 1. Meats 2. Carbohydrates 3. Fats 4. Dairy products

3. Anxiety over school absences RATIONALE: The school-age child is becoming industrious and attempts to master school-related activities. Therefore, school absences are likely to cause extreme anxiety for a school-age child who's chronically ill. Mutilation anxiety is more common in adolescents. Anticipatory grief is rare in a school-age child. Fear of hospital procedures is most pronounced in preschool-age children.

A chronically ill school-age child is most vulnerable to which stressor? 1. Mutilation anxiety 2. Anticipatory grief 3. Anxiety over school absences 4. Fear of hospital procedures

4. Right to privacy RATIONALE: This adolescent is exhibiting her right to privacy when she requests that she doesn't want a male nurse to care for her. She also has a right to competent care, the right to have an advance directive on file, and a right to confidentiality. However, she isn't exercising these rights in this scenario.

A female adolescent client refuses to allow male nurses to care for her while she's hospitalized. Which of these health care rights is this adolescent exerting? 1. Right to competent care 2. Right to have an advance directive on file 3. Right to confidentiality of her medical record 4. Right to privacy

1. Deliver five back blows. RATIONALE: If rescue breathing is unsuccessful in a child younger than age 1, the nurse should deliver five back blows, followed by five chest thrusts, to try to expel the object from the obstructed airway. The nurse shouldn't perform chest compressions because the infant has a pulse and because chest compressions are ineffective without a patent airway for ventilation. The nurse shouldn't use abdominal thrusts for a child younger than age 1 because they can injure the abdominal organs.

A hospitalized infant, age 10 months, begins to choke while eating and quickly becomes unconscious. A foreign object isn't visible in the infant's airway, but respirations are absent and the pulse is 50 beats/minute and thready. The nurse attempts rescue breathing, but the ventilations are unsuccessful. What should the nurse do next? 1. Deliver five back blows. 2. Deliver five chest thrusts. 3. Perform chest compressions. 4. Deliver five abdominal thrusts.

1. to talk with her daughter about what she should do if a stranger talks to her. RATIONALE: Preschoolers can begin to take a role in their own safety. They must be taught what a stranger is and what to do if a stranger approaches them. Living in a safe town doesn't eliminate the need to warn a child about talking to strangers. Although it's appropriate for the mother to talk with her daughter about strangers and have the daughter tell her if a stranger approaches her, the child needs to be aware of what to do at the time that the situation occurs, not only afterward. Contacting social services isn't appropriate because the nurse is capable of answering the mother's questions.

A mother of a 4-year-old child asks the nurse how to talk with her daughter about strangers. The little girl is very friendly and her mother is concerned that her child could be abducted. The nurse should tell the mother: 1. to talk with her daughter about what she should do if a stranger talks to her. 2. that she lives in a safe town and shouldn't worry. 3. to talk with her daughter about bad people and remind her to tell Mommy if someone she doesn't know talks to her. 4. contact social services, which is better equipped to respond to her questions.

2. latex. RATIONALE: If a child is sensitive to bananas, kiwifruit, and chestnuts, she's likely to be allergic to latex. Children with spina bifida commonly develop an allergy to latex and shouldn't be exposed to it. Some children are allergic to dyes in foods and other products, but dyes aren't a factor in a latex allergy.

A mother tells the nurse that her preschool-age daughter with spina bifida sneezes and gets a rash when playing with brightly colored balloons, and that recently she had an allergic reaction after eating kiwifruit and bananas. The nurse would suspect that the child may have an allergy to: 1. bananas. 2. latex. 3. kiwifruit. 4. color dyes.

3. Kidneys RATIONALE: The kidneys are most responsible for drug excretion in children. Less commonly, some drugs may be excreted via the lungs or liver. Drugs are never excreted by the heart in children or adults.

A nurse has just administered a drug to a child. Which organ is most responsible for drug excretion in children? 1. Heart 2. Lungs 3. Kidneys 4. Liver

1. may not disclose information regarding the child's condition. RATIONALE: According to Health Insurance Portability and Accountability Act standards, a nurse can't provide information regarding a child's care unless the child's parent or guardian authorizes her to do so. It wouldn't be appropriate for the nurse to contact an attorney at this time. Although not legally wrong, it wouldn't be appropriate for the nurse to make a statement about her feelings about the situation.

A nurse in the pediatric intensive care unit is caring for the only survivor of a house fire that killed seven people. Reporters from local newspapers and television stations are at the hospital, trying to obtain information about the child's condition. The nurse knows that she: 1. may not disclose information regarding the child's condition. 2. may disclose the child's condition, but not his name. 3. may make a statement about how sad she feels for the little boy's family and friends. 4. should contact an attorney because of the legal issues involved in caring for the child.

2. Edematous lips RATIONALE: A child who has ingested a caustic poison such as lye (found in toilet bowl cleaners) may develop edema, ulcers of the lips and mouth, pain in the mouth and throat, excessive salivation, dysphagia, and burns of the mouth, lips, esophagus, and stomach. Bleeding from burns in the GI tract can lead to pallor, hypotension (not hypertension), tachypnea, and tachycardia. The nurse would not expect to find reddish colored skin and lower abdominal pain because they don't commonly occur in caustic poisoning.

A nurse is assessing a 3-year-old child who has ingested toilet bowl cleaner. What finding should the nurse expect? 1. Reddish colored skin 2. Edematous lips 3. Hypertension 4. Lower abdominal pain

4. A preoccupation with death RATIONALE: An adolescent who demonstrates a preoccupation with death (such as by talking frequently about death) should be considered at high risk for suicide. Although depression, excessive sleepiness, and a history of cocaine use may occur in suicidal adolescents, they also occur in adolescents who aren't suicidal. Verbal and emotional withdrawal, not excessive talking, are signs of possible depression and suicide risk in an adolescent.

A nurse is assessing a severely depressed adolescent. Which finding indicates a risk of suicide? 1. Excessive talking 2. Excessive sleepiness 3. A history of cocaine use 4. A preoccupation with death

1. Babinski's RATIONALE: The nurse should be able to elicit the Babinski's reflex because it may be present the entire first year of life. The startle reflex actually disappears around 4 months of age; the Moro's reflex, by 3 or 4 months of age; and the dance reflex, after the third or fourth week.

A nurse is conducting an examination of a 6-month-old baby. During the examination, the nurse should be able to elicit which reflex? 1. Babinski's 2. Startle 3. Moro's 4. Dance

2. Pureed fruits 4. Rice cereal 5. Strained vegetables RATIONALE: The first food provided to a neonate is breast milk or formula. Between ages 4 and 6 months, rice cereal can be introduced, followed by pureed or strained fruits and vegetables, then strained, chopped or ground meat. Infants shouldn't be given whole milk until they are at least age 1. Fruit drinks provide no nutritional benefit and shouldn't be encouraged.

A nurse is conducting an infant nutrition class for parents. Which foods are appropriate to introduce during the first year of life? Select all that apply. 1. Sliced beef 2. Pureed fruits 3. Whole milk 4. Rice cereal 5. Strained vegetables 6. Fruit juice

2. Place the infant in an upright position when giving a bottle. RATIONALE: Feeding an infant a bottle in an upright position reduces the pooling of formula or breast milk in the nasopharynx. Formula, in particular, provides a good medium for the growth of bacteria, which can travel easily through the short, horizontal eustachian tubes. Administering antibiotics whenever the infant has a cold, avoiding getting the ears wet, and cleaning the external ear canal daily don't reduce the risk of an infant developing otitis media.

A nurse is developing a plan to teach a mother how to reduce her infant's risk of developing otitis media. Which direction should the nurse include in the teaching plan? 1. Administer antibiotics whenever the infant has a cold. 2. Place the infant in an upright position when giving a bottle. 3. Avoid getting the infant's ears wet while bathing or swimming. 4. Clean the infant's external ear canal daily.

2. Reading books RATIONALE: During the acute phase of rheumatic fever, the child should be placed on bed rest to reduce the workload of the heart and prevent heart failure. Therefore, an appropriate activity for this child would be reading books. Playing ping-pong, climbing on play equipment, and ambulating without restrictions are too strenuous during the acute phase.

A nurse is planning care for a 10-year-old child in the acute phase of rheumatic fever. Which activity is most appropriate for the nurse to schedule in the care plan? 1. Playing ping-pong 2. Reading books 3. Climbing on play equipment in the playroom 4. Ambulating without restrictions

4. "The special medicine will feel warm when it's put in the tubing." RATIONALE: To prepare a 4-year-old child without increasing anxiety, the nurse should provide concrete information in small amounts about nonthreatening aspects of the procedure. Therefore, saying the special medicine will feel warm is most appropriate. Saying that it won't hurt may prevent the child from trusting the nurse in the future. Explaining the time needed for the procedure wouldn't provide sufficient information. Stating that the child will need to sleep isn't true and could provoke anxiety.

A nurse is preparing a child, age 4, for cardiac catheterization. Which explanation of the procedure is appropriate? 1. "Don't worry. It won't hurt." 2. "The test usually takes an hour." 3. "You must sleep the whole time that the test is being done." 4. "The special medicine will feel warm when it's put in the tubing."

4. Increased carbohydrate need RATIONALE: Increased carbohydrate need is most likely because healing and repair of tissue requires more carbohydrates. Increased — not decreased — protein catabolism is present and decreased appetite — not increased — is a problem. Digestive enzymes are decreased — not increased.

A nurse is providing care to a 5-year-old child with a fractured femur whose nursing diagnosis is Imbalanced nutrition: Less than body requirements. Which change is most likely to occur with this condition? 1. Decreased protein catabolism 2. Increased calorie intake 3. Increased digestive enzymes 4. Increased carbohydrate need

3. Administer oxygen at a rate of 4 L/minute using a nonhumidified nasal cannula. RATIONALE: Oxygen should be humidified to assure that irritation of the mucosa doesn't occur. This adolescent's platelet level is decreased, so she's at risk for bleeding. The nose is a vascular region that can bleed easily if the mucosa is dried by the oxygen. Therefore, the nurse should revise the care plan to reflect use of humidified oxygen. A sign to remind others to avoid needle sticks and to not give anything via the rectum, the presence of two peripheral I.V.s, and the use of a tympanic temperature device are all aspects of care that would decrease the adolescent's risk of bleeding.

A nurse is reviewing a care plan for an adolescent girl who's receiving chemotherapy for leukemia who was admitted for pneumonia. The adolescent's platelet count is 50,000 μl. Which item in the care plan should the nurse revise? 1. Keep a sign over the bed that reads "NO NEEDLE STICKS AND NOTHING PER RECTUM." 2. Use two peripheral I.V. intermittent infusion devices, one for blood draws and one for infusions. 3. Administer oxygen at a rate of 4 L/minute using a nonhumidified nasal cannula. 4. Use a tympanic membrane sensor to measure her temperature at the bedside.

2. Lay the infant on his back or side to sleep. 3. Sit the infant up for each feeding. 5. Clean the suture line after each feeding by dabbing it with saline solution. 6. Give the infant extra care and support. RATIONALE: The nurse should instruct the parents to lay the infant on his back or side to sleep to prevent trauma to the surgery site. She should also instruct them to feed the infant in the upright position with a syringe and attached tubing to prevent stress to the suture line from sucking. In addition, to prevent crusts and scarring, the suture line should be cleaned after each feeding by dabbing it with half-strength hydrogen peroxide or saline solution. The parents should give the infant extra care and support because he can't meet emotional needs by sucking. Extra attention may also prevent crying, which stresses the suture line. Offering a pacifier isn't appropriate. Pacifiers shouldn't be used during the healing process because they stress the suture line. Arm restraints keep the infant's hands away from his mouth. They should be loosened every 2 hours, not every 4 hours.

A nurse is reviewing a teaching plan with parents of an infant undergoing repair for a cleft lip. Which instructions are the most appropriate for the nurse to give? Select all that apply. 1. Offer a pacifier as needed. 2. Lay the infant on his back or side to sleep. 3. Sit the infant up for each feeding. 4. Loosen the arm restraints every 4 hours. 5. Clean the suture line after each feeding by dabbing it with saline solution. 6. Give the infant extra care and support.

2. A second measles-mumps-rubella (MMR) vaccine RATIONALE: A second MMR vaccine is a recommended immunization for an adolescent. A Td vaccine is given 10 years after the most recent childhood DTaP vaccination (not 7 years after). A hepatitis B vaccine is recommended only if the adolescent hasn't received one earlier. A tuberculin skin test is necessary for adolescents who have been exposed to active tuberculosis, have lived in a homeless shelter, have been incarcerated, have lived in or come from an area with a high prevalence of tuberculosis, or are currently working in a health care setting. It isn't routinely administered every other year.

A nurse is reviewing an adolescent's immunization record. Which immunization is inappropriate for an adolescent as a component of preventative care? 1. A tetanus-diphtheria (Td) vaccine, given 7 years after the most recent childhood diphtheria, tetanus toxoids, and acellular pertussis (DTaP) vaccine 2. A second measles-mumps-rubella (MMR) vaccine 3. A tuberculin skin test every other year 4. The hepatitis B vaccine, if not received earlier

2. An 11-month-old infant receiving chemotherapy through a central venous catheter RATIONALE: The nurse should assess the 11-month-old infant with a central venous catheter first. This child takes priority because he has an invasive line and is receiving chemotherapy, which may cause toxic effects. Next, the nurse should assess the 5-month-old infant with an I.V. infusion and then the 14-year-old postoperative child. Because he's the most stable, the nurse can assess the 8-year-old child in traction last.

A nurse is reviewing her shift assignment. Which child should she assess first? 1. A 5-month-old infant with I.V. fluids infusing 2. An 11-month-old infant receiving chemotherapy through a central venous catheter 3. An 8-year-old child in traction with a femur fracture 4. A 14-year-old child who is postoperative and has a nasogastric tube and an indwelling urinary catheter

1. "I'll give the antibiotics for the full 10-day course of treatment." RATIONALE: The mother demonstrates understanding of antibiotic therapy by stating she'll give the full 10-day course of treatment. Antibiotics must be given for the full course of therapy, even if the child feels well. Otherwise, the infection won't be eradicated. Antibiotics should be taken at ordered intervals to maintain blood levels and not as needed for pain. A reexamination at the end of the course of antibiotics is necessary to confirm that the infection is resolved.

A nurse is teaching a parent how to administer antibiotics at home to a toddler with acute otitis media. Which statement by the parent indicates that teaching has been successful? 1. "I'll give the antibiotics for the full 10-day course of treatment." 2. "I'll give the antibiotics until my child's ear pain is gone." 3. "Whenever my child is cranky or pulls on an ear, I'll give a dose of antibiotics." 4. "If the ear pain is gone, there's no need to see the physician for another examination of the ears."

1. Acne 2. Hirsutism 3. Mood swings 4. Osteoporosis 6. Adrenal suppression RATIONALE: Adverse effects of corticosteroids include acne, hirsutism, mood swings, osteoporosis, and adrenal suppression. Steroid use in children and adolescents may cause delayed growth, not growth spurts.

A nurse is teaching an adolescent with inflammatory bowel disease about treatment with corticosteroids. Which adverse effects are concerns for this client? Select all that apply. 1. Acne 2. Hirsutism 3. Mood swings 4. Osteoporosis 5. Growth spurts 6. Adrenal suppression

RATIONALE: Making sure all medications are kept in containers with childproof safety caps is the most appropriate guideline because poisoning accidents are common in toddlers owing to the toddler's curiosity and his increasing mobility and ability to climb. When riding in a car, a toddler should be strapped into a car seat, not a seat belt. A seat belt is an appropriate guideline for a school-age child. Never leaving a child alone on a bed is an appropriate guideline for parents of infants. Toddlers already have the ability to climb on and off of beds and other furniture by themselves. Note, however, that toddlers should never be left unattended on high surfaces, such as an examining table in a physician's office. Teaching the rules of the road for bicycle safety is an appropriate safety measure for a school-age child. Toddlers shouldn't be allowed in the road unsupervised.

A nurse is teaching parents about accident prevention for a toddler. Which guideline is most appropriate? 1. Always make the toddler wear a seat belt when riding in a car. 2. Make sure all medications are kept in containers with childproof safety caps. 3. Never leave a toddler unattended on a bed. 4. Teach rules of the road for bicycle safety.

2. A 6-month-old infant can usually roll from prone to supine and supine to prone positions. 3. A teething ring is appropriate for a 6-month-old infant. 6. Lack of visual coordination usually resolves by age 6 months. RATIONALE: Gross motor skills of the 6-month-old infant include rolling from front to back and back to front. Teething usually begins around age 6 months; therefore, a teething ring is appropriate. Visual coordination is usually resolved by age 6 months. At age 6 months, fine motor skills include purposeful grasps. Stranger anxiety normally peaks at 8 months of age. The 6-month-old infant also should have good head control and no longer display head lag when pulled up to a sitting position.

A nurse is teaching the parents of a 6-month-old infant about usual growth and development. Which statements about infant development are true? Select all that apply. 1. A 6-month-old infant has difficulty holding objects. 2. A 6-month-old infant can usually roll from prone to supine and supine to prone positions. 3. A teething ring is appropriate for a 6-month-old infant. 4. Stranger anxiety usually peaks at 12 to 18 months. 5. Head lag is commonly noted in infants at age 6 months. 6. Lack of visual coordination usually resolves by age 6 months.

1. Caring for the same child from admission to discharge RATIONALE: Primary care nursing requires that the primary nurse care for the same child (to whom she's assigned) during her scheduled shift. The associate nurse is assigned to the child care assignment when the primary nurse has a day off or during the evening and night shifts. Caring for different children each shift doesn't promote continuity of care. Taking vital signs for every child on the floor is an example of team nursing, in which each member of the team is assigned one specific task for each child. The charge nurse may be directly involved in child care.

A nurse is working on the pediatric unit. Which assignment best demonstrates primary care nursing? 1. Caring for the same child from admission to discharge 2. Caring for different children each shift to gain nursing experience 3. Taking vital signs for every child hospitalized on the unit 4. Assuming the charge nurse role instead of participating in direct child care

2. Proximodistal RATIONALE: Proximodistal development progresses from the center of the body to the extremities, such as from the arm to the fingers. Cephalocaudal development occurs along the body's long axis; for example, the infant develops control over the head, mouth, and eye movements before the upper body, torso, and legs. Mass-to-specific development, sometimes called differentiation, occurs as the child masters simple operations before complex functions and moves from broad, general patterns of behavior to more refined ones.

A nurse notes that an infant develops arm movement before fine-motor finger skills and interprets this as an example of which pattern of development? 1. Cephalocaudal 2. Proximodistal 3. Differentiation 4. Mass-to-specific

2. Parallel play RATIONALE: Two-year-olds engage in parallel play, in which they play side by side but rarely interact. Associative play is characteristic of preschoolers, in which they are all engaged in a similar activity but there is little organization. School-age children engage in cooperative play, which is organized and goal-directed. Therapeutic play is a technique that can be used to help understand a child's feelings; it consists of energy release, dramatic play, and creative play.

A nurse observes a 2½-year-old child playing with another child of the same age in the playroom on the pediatric unit. What type of play should the nurse expect the children to engage in? 1. Associative play 2. Parallel play 3. Cooperative play 4. Therapeutic play

1. Immunoglobulin E RATIONALE: The nurse would expect elevated immunoglobulin (Ig) E levels because IgE is predominantly found in saliva and tears as well as intestinal and bronchial secretions and, therefore, may be found in allergic disorders. IgD's physiologic function is unknown and constitutes only 1% of the total number of circulating immunoglobulins. IgG is elevated in the presence of viral and bacterial infections. IgM is the first antibody activated after an antigen enters the body, and is especially effective against gram-negative organisms.

A nurse practicing in a nurse-managed clinic suspects that an 8-year-old child's chronic sinusitis and upper respiratory tract infections may result from allergies. She orders an immunoglobulin assay. Which immunoglobulin would the nurse expect to find elevated? 1. Immunoglobulin E 2. Immunoglobulin D 3. Immunoglobulin G 4. Immunoglobulin M

2. The infant's right to privacy RATIONALE: All clients are entitled to privacy; providing it doesn't represent individualization of nursing care. Nurses provide privacy to minors without regard to their parents' expectations. Provision of privacy is every client's right and isn't specifically related to institutional liability.

A nurse provides privacy to the infants in her care. This approach is an example of which international concept? 1. Individualization of nursing care 2. The infant's right to privacy 3. The parental expectation for nursing behavior 4. The hospital's liability protection

4. "We try to be united and consistent in our approach to discipline." RATIONALE: To deal with misbehavior most successfully, parents should be firm and consistent when taking appropriate disciplinary action. Usually, parents should begin setting limits and implementing discipline, such as using time-outs for inappropriate behavior, around age 1, or when the child begins to crawl and explore the environment. Rigidly enforcing rules wouldn't allow the development of autonomy and could lead to self-doubt. The parent should never be encouraged to withdraw attention or affection as a result of the child's behavior, or any other reason.

A parent asks the nurse for advice on disciplining a 3-year-old child. Which statement made by a parent indicates understanding of accepted discipline techniques? 1. "I don't think children younger than 5 understand the purpose of time-out." 2. "My husband uses one form of punishment and I use a different form." 3. "I don't listen to excuses." 4. "We try to be united and consistent in our approach to discipline."

3. Tetracycline RATIONALE: Tetracycline should be avoided in children younger than age 8 because it may cause enamel hypoplasia and permanent yellowish gray to brownish tooth discoloration. Penicillin, erythromycin, and amoxicillin don't discolor the teeth.

A physician orders an antibiotic for a child, age 6, who has an upper respiratory tract infection. To avoid tooth discoloration, the nurse expects the physician to avoid prescribing which drug? 1. Penicillin 2. Erythromycin 3. Tetracycline 4. Amoxicillin

2. Bradycardia RATIONALE: Bradycardia is a sign of late shock in a child. Cardiovascular dysfunction and impairment of cellular function lead to lowered perfusion pressures, increased precapillary arteriolar resistance, and venous capacitance. Decreased cardiac output occurs in late shock if the circulating volume isn't replaced. Sympathetic nervous innervation has limited compensation mechanisms if the volume isn't replaced. Tachycardia and irritability occur during the early phase of shock as compensatory mechanisms are implemented to increase cardiac output. Normal pediatric urine output is 1 to 2 ml/kg/hour; volumes less than this would indicate a decrease in renal perfusion and activation of the renin-angiotensin-aldosterone system to decrease water and sodium excretion.

A preschool child presents with a history of vomiting and diarrhea for 2 days. Which assessment finding indicates that the child is in the late stages of shock? 1. Tachycardia 2. Bradycardia 3. Irritability 4. Urine output 1 to 2 ml/kg/hour

2. prepare to ventilate the child. RATIONALE: The nurse should recognize these physical findings as signs and symptoms of impending respiratory collapse. Therefore, the nurse's top priority is to assess airway, breathing, and circulation, and prepare to ventilate the child if necessary. The nurse should then notify the emergency medical systems to transport the child to a local hospital. Because the child's condition requires immediate intervention, simply monitoring pulse oximetry would delay treatment. This child shouldn't be returned to class. When the child's condition allows, the nurse can notify the parents or guardian.

A school nurse is evaluating a 7-year-old child who is having an asthma attack. The child is cyanotic and unable to speak, with decreased breath sounds and shallow respirations. Based on these physical findings, the nurse should first: 1. monitor the child with a pulse oximeter in her office. 2. prepare to ventilate the child. 3. return the child to class. 4. contact the child's parent or guardian.

1. allowing the child to change into a gown while she isn't in the room. RATIONALE: School-age children tend to be very modest. The nurse should allow them to change into gowns while she isn't in the examination room. Children shouldn't have to take off their underwear for routine medical examinations. Playing with medical equipment is characteristic of younger children. The nurse shouldn't ask parents to leave the room unless the child requests that they not be present. A school-age child may feel too old to hold a stuffed animal during the examination.

A school-age child presents to the office for a routine examination. Given the child's developmental level, a nurse should give highest priority to: 1. allowing the child to change into a gown while she isn't in the room. 2. allowing the child to play with medical equipment before the examination begins. 3. asking the parents to leave the room during the child's examination. 4. encouraging the child to hold a stuffed animal during the examination.

2. "Let's see about further developmental testing." RATIONALE: Stating that further developmental testing is necessary is appropriate because at age 12 months a child should be sitting up and rolling over. Therefore, this child may have developmental problems. Saying the infant's behavior is abnormal or suggesting that the mother hasn't seen her infant do these milestones isn't therapeutic and can cut off communication with the mother. Telling the mother that the infant's behavior is normal misleads the mother with false reassurance.

A teenage mother brings her 1-year-old child to the pediatrician's office for a well-baby checkup. She says that her infant can't sit alone or roll over. An appropriate response by the nurse would be: 1. "This is very abnormal. Your child must be sick." 2. "Let's see about further developmental testing." 3. "Don't worry, this is normal for her age." 4. "Maybe you just haven't seen her do it."

1. 50 mg RATIONALE: The dose is 5 mg/kg and the child weighs 10 kg. To determine the dose, the nurse would calculate: 5 mg/1 kg × 10 kg = 50 mg per dose.

A toddler develops acute otitis media and is ordered cefpodoxime proxetil (Vantin) 5 mg/kg P.O. every 12 hours. If the child weighs 22 lb (10 kg), how many milligrams will the nurse administer with each dose? 1. 50 mg 2. 100 mg 3. 110 mg 4. 220 mg

4. "It must be difficult for you when your child is ill and hospitalized." RATIONALE: Expressing concern is the most appropriate response because it acknowledges the parents' feelings. False reassurance, such as telling parents not to worry, isn't helpful because it doesn't acknowledge their feelings. Encouraging parents to look at how ill other children are also isn't helpful because the focus of the parents is on their own child. Asking what the concern is merely reinforces the parents' concern without addressing it.

A toddler is in the hospital. The parents tell the nurse they're concerned about the seriousness of the child's illness. Which response to the parents is most appropriate? 1. "Please try not to worry. Your child will be fine." 2. "If you look around, you'll see other children who are much sicker." 3. "What seems to concern you about your child being hospitalized?" 4. "It must be difficult for you when your child is ill and hospitalized."

3. Clamp the catheter. RATIONALE: First, the nurse must clamp the catheter to prevent air entry, which could lead to air embolism. If an air embolism occurs, the nurse should position the child on the side after clamping the catheter. The nurse may turn off the infusion pump after ensuring the child's safety. If blood has backed up in the catheter, the nurse may need to flush the catheter with heparin; however, this isn't the initial priority.

A toddler is receiving an infusion of total parenteral nutrition via a Broviac catheter. As the child plays, the I.V. tubing becomes disconnected from the catheter. What should the nurse do first? 1. Turn off the infusion pump. 2. Position the child on the side. 3. Clamp the catheter. 4. Flush the catheter with heparin.

3. Industry versus inferiority RATIONALE: In middle childhood, the 6- to 12-year-old child is mastering the task of industry versus inferiority. The trust versus mistrust task is in infancy (birth to 1 year). In early childhood, the 1- to 3-year-old child is in the stage of initiative versus guilt. Identity versus role confusion occurs during adolescence.

According to Erikson's psychosocial theory of development, an 8-year-old child would be in which stage? 1. Trust versus mistrust 2. Initiative versus guilt 3. Industry versus inferiority 4. Identity versus role confusion

2. "Long-term steroid therapy may interfere with a child's growth." RATIONALE: Steroids suppress release of adrenocorticotropic hormone from the pituitary gland, stopping production of endogenous hormones by the adrenal cortex. Because prolonged adrenal suppression may cause growth retardation in a child, the duration and dosage of steroid therapy must be kept to a minimum. Steroids also may cause central nervous system effects, such as euphoria, insomnia, and mood swings. Although steroids increase the appetite, this effect isn't the reason for limiting their use in children. Steroids are present in the body, so hypersensitivity isn't a problem, and they're likely to cause euphoria, not depression.

After being hospitalized for status asthmaticus, a child, age 5, is discharged with prednisone (Deltasone) and other oral medications. Two weeks later, when the child comes to the clinic for a checkup, the nurse instructs the mother to gradually decrease the dosage of prednisone, which will be discontinued. The mother asks why prednisone must be discontinued. How should the nurse respond? 1. "Steroids increase the appetite, leading to obesity with prolonged use." 2. "Long-term steroid therapy may interfere with a child's growth." 3. "The child may develop a hypersensitivity to steroids with continued use." 4. "Prolonged steroid use may cause depression."

3. 1.08 ml RATIONALE: Because the infant weighs 17 lb (7.7 kg), the safe dosage range is 385 to 578 mg daily. The ordered dosage, 540 mg daily, is safe. To calculate the amount to administer, the nurse may use the following fraction method: 500 mg/2 ml = 270 mg/X ml 500X = 270 × 2 500X = 540 X = 540/500 X = 1.08 ml

An 8-month-old infant is admitted with a febrile seizure. The infant weighs 17 lb (7.7 kg). The physician orders ceftriaxone (Rocephin), 270 mg I.M. every 12 hours. (The safe dosage range is 50 to 75 mg/kg daily.) The pharmacy sends a vial containing 500 mg, to which the nurse adds 2 ml of preservative-free normal saline solution. The nurse should administer how many milliliters? 1. None because this isn't a safe dosage 2. 0.08 ml 3. 1.08 ml 4. 1.8 ml

1. Frequent clearing of the throat 3. Frequent swallowing 6. Bright red vomitus RATIONALE: A classic sign of bleeding after tonsillectomy is frequent swallowing; this sign occurs because blood drips down the back of the throat, tickling it. Other signs include frequent clearing of the throat and vomiting of bright red blood. Vomiting of dark blood may be seen if the child swallowed blood during surgery but doesn't indicate postoperative bleeding. Breathing through the mouth is common because of dried secretions in the nares. Sleeping for long intervals is normal after a client receives sedation and anesthesia. A pulse rate of 98 beats/minute is in the normal range for this age-group.

An 8-year-old child has just returned from the operating room after having a tonsillectomy. The nurse is preparing to do a postoperative assessment. The nurse should be alert for which signs and symptoms of bleeding? Select all that apply. 1. Frequent clearing of the throat 2. Breathing through the mouth 3. Frequent swallowing 4. Sleeping for long intervals 5. Pulse rate of 98 beats/minute 6. Bright red vomitus

3. Kernig's sign. RATIONALE: Signs and symptoms of meningitis include Kernig's sign, stiff neck, headache, and fever. To test for Kernig's sign, the client is in the supine position with knees flexed; a leg is then flexed at the hip so that the thigh is brought to a position perpendicular to the trunk. An attempt is then made to extend the knee. If meningeal irritation is present, the knee can't be extended and attempts to extend the knee result in pain. Cullen's sign is the bluish discoloration of the periumbilical skin caused by intraperitoneal hemorrhage. Koplik's spots are reddened areas with grayish blue centers that are found on the buccal mucosa of a client with measles. Chvostek's sign is elicited by tapping the client's face lightly over the facial nerve, just below the temple. A calcium deficit is suggested if the facial muscles twitch.

An 8-year-old child is suspected of having meningitis. Signs of meningitis include: 1. Cullen's sign. 2. Koplik's spots. 3. Kernig's sign. 4. Chvostek's sign.

2. Metabolic alkalosis RATIONALE: In a client with bulimia nervosa, metabolic alkalosis may occur secondary to hydrogen loss caused by frequent, self-induced vomiting. Typically, the blood glucose level is within normal limits, making hypoglycemia unlikely. In bulimia nervosa, hypokalemia is more common than hyperkalemia and typically results from potassium loss related to frequent vomiting.

An adolescent is admitted for treatment of bulimia nervosa. When developing the care plan, the nurse anticipates including interventions that address which metabolic disorder? 1. Hypoglycemia 2. Metabolic alkalosis 3. Metabolic acidosis 4. Hyperkalemia

3. Ham and eggs RATIONALE: Good sources of dietary iron include red meat, egg yolks, whole wheat breads, seafood, nuts, legumes, iron-fortified cereals, and green, leafy vegetables. Fresh fruits and milk products contain only small amounts of iron. White bread isn't a good iron source.

An adolescent is diagnosed with iron deficiency anemia. After emphasizing the importance of consuming dietary iron, the nurse asks him to select iron-rich breakfast items from a sample menu. Which selection demonstrates knowledge of dietary iron sources? 1. Grapefruit and white toast 2. Pancakes and a banana 3. Ham and eggs 4. Bagel and cream cheese

2. poor hygiene and weight loss. RATIONALE: Signs of neglect include poor hygiene and weight loss because neglect can involve failure to provide food, bed, shelter, health care, or hygiene. Slapping, kicking, pulling hair, hitting, and punching are examples of forms of physical abuse, not neglect. Loud crying and screaming are normal findings in a 3-year-old boy.

An emergency department nurse suspects neglect in a 3-year-old boy admitted for failure to thrive. Signs of neglect in the child would include: 1. slapping, kicking, and punching others. 2. poor hygiene and weight loss. 3. loud crying and screaming. 4. pulling hair and hitting.

1. Cleaning the suture line carefully with a sterile solution after every feeding RATIONALE: To avoid an infection that could adversely affect the cosmetic outcome of the repair, the suture line must be cleaned very gently with a sterile solution after each feeding. Laying an infant on his abdomen after a cleft lip repair isn't appropriate because doing so will put pressure on the suture line, causing damage. The infant can be positioned on his side to drain saliva without affecting the suture line. Crying puts tension on the suture line and should be avoided by anticipating the baby's needs, such as holding and cuddling him. Hard objects such as pacifiers should be kept away from the suture line because they can cause damage.

An infant boy has just had surgery to repair his cleft lip. Which nursing intervention is important during the immediate postoperative period? 1. Cleaning the suture line carefully with a sterile solution after every feeding 2. Laying the infant on his abdomen to help drain fluids from his mouth 3. Allowing the infant to cry to promote lung reexpansion 4. Giving the baby a pacifier to suck for comfort

2. Ortolani's sign. RATIONALE: In a child with a congenital hip dislocation, assessment typically reveals Ortolani's sign, asymmetrical thigh and gluteal folds, limited hip abduction, femoral shortening, and Trendelenburg's sign.

An infant is diagnosed with a congenital hip dislocation. On assessment, the nurse expects to note: 1. symmetrical thigh and gluteal folds. 2. Ortolani's sign. 3. increased hip abduction. 4. femoral lengthening.

4. Maintaining a consistent, structured environment RATIONALE: The nurse caring for an infant with inorganic failure to thrive should strive to maintain a consistent, structured environment because it reinforces a caring feeding environment. Encouraging the infant to hold a bottle would reinforce an uncaring feeding environment. The infant should receive social stimulation rather than be confined to bed rest. The number of caregivers should be minimized to promote consistency of care.

An infant is hospitalized for treatment of inorganic failure to thrive. Which nursing action is most appropriate for this child? 1. Encouraging the infant to hold a bottle 2. Keeping the infant on bed rest to conserve energy 3. Rotating caregivers to provide more stimulation 4. Maintaining a consistent, structured environment

1. Tragus, mastoid process, and helix RATIONALE: Before inserting the otoscope, the nurse should palpate the child's external ear, especially the tragus and mastoid process, and should pull the helix backward to determine the presence of pain or tenderness. The umbo, incus, and malleus (parts of the middle ear) and the cochlea (part of the inner ear) aren't palpable.

Before performing an otoscopic examination on a child, where should the nurse palpate for tenderness? 1. Tragus, mastoid process, and helix 2. Helix, umbo, and tragus 3. Tragus, cochlea, and lobule 4. Mastoid process, incus, and malleus

3. Rice cereal RATIONALE: The nurse should instruct her to introduce rice cereal first because it's easy to digest and is associated with few allergies. Next, the infant can receive pureed fruits, such as bananas, applesauce, and pears, followed by pureed vegetables, egg yolks, cheese, yogurt and, finally, meat. Egg whites shouldn't be given until age 9 months because they may trigger a food allergy.

During a well-baby visit, a mother asks the nurse when she should start giving her infant solid foods. The nurse should instruct her to introduce which solid food first? 1. Applesauce 2. Egg whites 3. Rice cereal 4. Yogurt

4. Held in the bottle-feeding position RATIONALE: The nurse should hold an infant in the bottle-feeding position when administering an oral medication by placing the child's inner arm behind the back, supporting the head in the crook of the elbow, and holding the child's free hand with the hand of the supporting arm. A 4-month-old infant can't sit unsupported in a high chair. Administering medication to an infant lying flat could cause choking and aspiration. Holding the infant in the lap may cause the medication to spill.

How should a nurse position a 4-month-old infant when administering an oral medication? 1. Seated in a high chair 2. Restrained flat in the crib 3. Held on the nurse's lap 4. Held in the bottle-feeding position

1. Identification of neonates, infants, toddlers, children, and adolescents at all times 2. The facility's physical layout 4. Available resources to obtain and maintain the security plan 5. Methods for educating all staff regarding the security plan RATIONALE: When developing a security plan for a pediatric unit, the nurse should consider the identification of neonates, infants, toddlers, children, and adolescents; the facility's physical layout; available resources; and methods for educating staff. She needn't consider the climate in which the hospital is located.

In developing a security plan for a pediatric unit, a nurse must consider which factors? Select all that apply. 1. Identification of neonates, infants, toddlers, children, and adolescents at all times 2. The facility's physical layout 3. The climate in which the hospital is located 4. Available resources to obtain and maintain the security plan 5. Methods for educating all staff regarding the security plan

2. Ask the mother for more information about the infant's sleep patterns. RATIONALE: The nurse needs more information about the infant's sleep patterns to rule out potential problems before determining whether the infant is getting enough sleep. The nurse shouldn't offer advice or reassurance without knowing more about the infant's specific sleep habits.

The mother of an 11-month-old infant reports to the nurse that her infant sleeps much less than other children. The mother asks the nurse whether her infant is getting sufficient sleep. What should be the nurse's initial response? 1. Reassure the mother that each infant's sleep needs are individual. 2. Ask the mother for more information about the infant's sleep patterns. 3. Instruct the mother to decrease the infant's daytime sleep to increase his nighttime sleep. 4. Inform the mother that her infant's growth and development are appropriate for his age, so sleep isn't a concern.

4. ask open-ended questions about the parents' concerns. RATIONALE: Asking open-ended questions about the parents' concerns will help the nurse understand why they're asking for information. Advance directives are rarely prepared for healthy infants. The parents' request for information may indicate distress, and the nurse should obtain more details before giving them information. Although suggesting the parents talk to their attorney or to the physician and providing the parents with a brochure about advance directives are appropriate actions, the nurse must obtain additional information before implementing these choices.

The parents of a healthy infant request information about advance directives. The nurse's best response is to: 1. suggest that the parents discuss the matter with an attorney. 2. tell the parents that they should discuss advance directives with the physician. 3. provide the parents with a brochure about advance directives. 4. ask open-ended questions about the parents' concerns.

2. Elevating the neonate's head and giving nothing by mouth RATIONALE: Because of the risk of aspiration, a neonate with a known or suspected tracheoesophageal fistula should be kept with the head elevated at all times and should receive nothing by mouth (NPO). The nurse should suction the neonate regularly to maintain a patent airway and prevent pooling of secretions. Elevating the neonate's head after feedings or giving glucose water are inappropriate because the neonate must remain on NPO status.

The physician suspects tracheoesophageal fistula in a 1-day-old neonate. Which nursing intervention is most appropriate for this child? 1. Avoiding suctioning unless cyanosis occurs 2. Elevating the neonate's head and giving nothing by mouth 3. Elevating the neonate's head for 1 hour after feedings 4. Giving the neonate only glucose water for the first 24 hours

3. Asking open-ended questions RATIONALE: Open-ended questions allow the adolescent to share information and feelings. Asking personal questions not related to the situation jeopardizes the trust that must be established because the adolescent may feel as though he's being interrogated with unnecessary questions. Writing everything down during the interview can be a distraction and doesn't allow the nurse to observe how the adolescent behaves. Discussing the nurse's thoughts and feelings may bias the assessment and is inappropriate when interviewing any client.

To establish a good interview relationship with an adolescent, which strategy is most appropriate? 1. Asking personal questions unrelated to the situation 2. Writing down everything the teen says 3. Asking open-ended questions 4. Discussing the nurse's own thoughts and feelings about the situation

3. A serum trough and peak level around the third dose RATIONALE: Aminoglycosides such as gentamicin have a narrow range between therapeutic and toxic serum levels. A serum peak and trough level (taken half an hour before the dose and half an hour after the dose has been administered) around the third dose is the most accurate way to determine the correct serum values because the third dose provides enough medication buildup in the blood stream to be measured. A trough level every morning, a serum peak level after the second dose, and serial serum trough levels won't provide sufficient data about the effectiveness of the antibiotic.

When administering gentamicin (Garamicin) to a preschooler, which monitoring schedule is best for determining the drug's effectiveness? 1. A serum trough level every morning 2. A serum peak level after the second dose 3. A serum trough and peak level around the third dose 4. Serial serum trough levels after three doses (24 hours)

2. Honey-colored, crusted lesions RATIONALE: In impetigo, honey-colored, crusted lesions develop once the pustules rupture. Small, brown, benign lesions are common in children with warts. Linear, threadlike burrows are typical in a child with scabies. Circular lesions that clear centrally characterize tinea corporis.

When assessing a child for impetigo, the nurse expects which assessment findings? 1. Small, brown, benign lesions 2. Honey-colored, crusted lesions 3. Linear, threadlike burrows 4. Circular lesions that clear centrally

4. behavioral patterns are passed from one generation to the next. RATIONALE: The nurse should keep in mind that a family's behavioral patterns and values are passed from one generation to the next. Cultural background commonly plays a major role in determining a family's health practices. Physical characteristics don't indicate a child's culture. Although heritage plays a role in culture, it doesn't dictate a group's shared values, and its effect on culture is weaker than that of behavioral patterns.

When assessing a child's cultural background, the nurse should keep in mind that: 1. cultural background usually has little bearing on a family's health practices. 2. physical characteristics mark the child as part of a particular culture. 3. heritage dictates a group's shared values. 4. behavioral patterns are passed from one generation to the next.

RATIONALE: The anterior fontanel is formed by the junction of the sagittal, frontal, and coronal sutures. It's shaped like a diamond and normally measures 4 to 5 cm at its widest point. A widened, bulging fontanel is a sign of increased ICP.

When assessing an infant for changes in intracranial pressure (ICP), a nurse must palpate the fontanels. Identify the area where the nurse should palpate to assess the anterior fontanel.

3. Trust versus mistrust RATIONALE: Freud defined the first 2 years of life as the oral stage and suggested that the mouth is the primary source of satisfaction for the developing child. Erikson posited that infancy (from birth to age 12 months) is the stage of trust versus mistrust, during which the infant learns to deal with the environment through the emergence of trustfulness or mistrust. Initiative versus guilt corresponds to Freud's phallic stage. Autonomy versus shame and doubt corresponds to Freud's anal/sensory stage. Industry versus inferiority corresponds to Freud's latency period.

When developing a care plan for a child, the nurse identifies which Eriksonian stage as corresponding to Freud's oral stage of psychosexual development? 1. Initiative versus guilt 2. Autonomy versus shame and doubt 3. Trust versus mistrust 4. Industry versus inferiority

2. establishing an identity. RATIONALE: According to Erikson, the primary psychosocial task during adolescence is to establish a personal identity while overcoming role or identity confusion. The adolescent attempts to establish a group identity by seeking acceptance and approval from peers, and strives to attain a personal identity by becoming more independent from his family. Becoming industrious is the developmental task of the school-age child; achieving intimacy is the task of the young adult; and developing initiative is the task of the preschooler.

When developing a care plan for an adolescent, the nurse considers the child's psychosocial needs. During adolescence, psychosocial development focuses on: 1. becoming industrious. 2. establishing an identity. 3. achieving intimacy. 4. developing initiative.

3. A rear-facing infant safety seat in the middle of the back seat RATIONALE: Infants from birth to 20 lb (9.1 kg) and younger than age 1 must be in a rear-facing infant or convertible seat in the back seat, preferably in the middle. Infants and small children should never be placed in the front seat because of the risk of injuries from a breaking front windshield and an expanding airbag. Positioning a car seat next to the window isn't preferred.

When discharging a 5-month-old infant from the hospital, the nurse checks to see whether the parent's car restraint system for the infant is appropriate. Which restraint system would be safest? 1. A front-facing convertible car seat in the middle of the back seat 2. A rear-facing infant safety seat in the front passenger seat 3. A rear-facing infant safety seat in the middle of the back seat 4. A front-facing convertible car seat in the back seat next to the window

1. provide beneficial care and avoid harming the child. RATIONALE: Nurses must provide beneficial care and avoid harming all clients. A nurse shouldn't base any decision solely on the desire to prevent legal trouble, on her own feelings for her loved ones, or what the physician says.

When making ethical decisions about caring for preschoolers, a nurse should remember to: 1. provide beneficial care and avoid harming the child. 2. make decisions that will prevent legal trouble. 3. do what she would do for her own child or loved ones. 4. be sure to do what the physician says.

3. Coordinate the multidisciplinary services and providing information about them. RATIONALE: Coordinating the multidisciplinary services and providing information about them demonstrate collaboration because the nurse will be explaining the functions of social service, case management, and so forth. Providing parents with information about financial assistance programs is the responsibility of social services, not a nursing role. Informing the family of the diagnosis and recently discovered findings is a physician's responsibility as is referring and consulting with other specialties.

When meeting with a family who'll learn that their 3-year-old is seriously ill, which action demonstrates the nurse's role as collaborator of care? 1. Providing the parents with information about financial assistance programs. 2. Informing the family of the diagnosis and recently discovered findings. 3. Coordinate the multidisciplinary services and providing information about them. 4. Referring and consulting with other specialties to help in treating the diagnosis.

2. Complaints of a stiff neck RATIONALE: The nurse should discuss complaints of a stiff neck because fever and a stiff neck indicate possible meningitis. Burning or pain with urination, fever that disappears for 24 hours then returns, and a history of febrile seizures should be addressed by the physician but can wait until office hours.

When teaching a parent of a school-age child about signs and symptoms of fever that require immediate notification of the physician, which description should the nurse include? 1. Burning or pain with urination 2. Complaints of a stiff neck 3. Fever disappearing for longer than 24 hours, then returning 4. History of febrile seizures

1. Potassium level of 6.5 mEq/L RATIONALE: A potassium level of 6.5 mEq/L requires immediate follow-up because it's considered critically high, making the child prone to cardiac arrhythmias. Whereas a blood pressure of 90/50 mm Hg should be recorded and monitored, it doesn't require immediate follow-up. Abdominal cramping may be caused by several conditions and can be observed over time.

Which finding in a 3-year-old child with acute renal failure requires immediate follow-up? 1. Potassium level of 6.5 mEq/L 2. Blood pressure in right leg of 90/50 mm Hg 3. Abdominal cramps 4. No albumin in the urine

4. Weighing the infant daily RATIONALE: Weighing an infant daily provides the most accurate information about the infant's hydration status. Vital signs, intake and output, and electrolyte levels provide helpful information about an infant's hydration status, but they aren't as accurate as weighing daily.

Which intervention provides the most accurate information about an infant's hydration status? 1. Monitoring the infant's vital signs 2. Accurately measuring intake and output 3. Monitoring serum electrolyte levels 4. Weighing the infant daily

4. Obtaining a history of the illness RATIONALE: Obtaining a history of the infant's illness takes priority because the history helps with developing a treatment plan. Getting a stool specimen and weighing the infant can follow taking the history. The nurse shouldn't offer clear liquids because they increase the risk of vomiting, which may worsen the infant's dehydration.

Which intervention takes priority when admitting an infant with acute gastroenteritis? 1. Obtaining a stool specimen 2. Weighing the infant 3. Offering the infant clear liquids 4. Obtaining a history of the illness

3. Arm restraints while asleep RATIONALE: The nurse should revise a care plan that includes restraints. Restraints should never be used on a child with a seizure disorder because they could harm him if a seizure occurs. Padded side rails will prevent the child from injuring himself during a seizure. The bag and mask system should be present in case the child needs oxygen during a seizure. Cardiopulmonary monitoring should be readily available for checking vital signs during a seizure.

Which item in the care plan for a toddler with a seizure disorder should a nurse revise? 1. Padded side rails 2. Oxygen mask and bag system at bedside 3. Arm restraints while asleep 4. Cardiorespiratory monitoring

3. Having the child take a deep breath and blow it out until told to stop RATIONALE: Having the child take a deep breath and blow it out is a form of distraction and will help the child cope better with the procedure. A child may prefer to keep his eyes open, not shut, during a procedure so he can see what is going on and can anticipate what is going to happen. Letting a child yell during a procedure is a form of helpful distraction. In addition, holding the breath isn't beneficial and could have adverse effects (such as feeling dizzy or faint). The nurse should prepare a child for a procedure by using nonpain descriptors and not suggesting pain. For example, the nurse might say, "Sometimes this feels like pushing or sticking, and sometimes it doesn't bother children at all."

Which relaxation strategy would be effective for a school-age child to use during a painful procedure? 1. Having the child keep his eyes shut at all times 2. Having the child hold his breath and not yell 3. Having the child take a deep breath and blow it out until told to stop 4. Being honest with the child and telling him the procedure will hurt a lot


Set pelajaran terkait

PTA 101: Biomechanics and Vitals

View Set

Social Media Marketing Chapter Quiz 8-11 Study Guide

View Set

Week 4 Chapter 9 Equine Clinical procedures

View Set

Exam 2 LDR alternative questions

View Set

Presentation: PHP and MySQL: Chapter 12 Cookies and Sessions [Slides 1-19]

View Set